You are on page 1of 98

lOMoARcPSD|15007663

Reading Vstep B1

Khoa Công Nghệ Sinh Học - Thực Phẩm (Trường Đại học Kỹ thuật - Công nghệ Cần
Thơ)

Scan to open on Studeersnel

Studocu is not sponsored or endorsed by any college or university


Downloaded by Nhi An (annhinguyendang@gmail.com)
lOMoARcPSD|15007663

ENGLISH PRACTICE GROUP WITH ERIC


MR. TRAN VAN HAU

READING
VSTEP B1

Compiled and Updated by Tran Van Hau

1/1/2019

Downloaded by Nhi An (annhinguyendang@gmail.com)


lOMoARcPSD|15007663

TRAN VAN HAU (ERIC TRAN) VSTEP – READING B1 UPDATED 2019


SESSION 1
READING THEORY
I. READING FOR TOPICS, MAIN IDEAS.
<How to recognize questions:
- What is the passage about?
- What is the main idea in this passage?
- What is the title for this passage?
- What is the best summarize for this passage?
- What is the topic of the paragraph?
a. How to answer:
Step 1: Read the opening sentence and the last sentence of the paragraph since they are
often the topic statements of that paragraph. Only focus on the main subject, the main verb,
and the main object.
Note: Sometimes, the topic sentence is presented in the middle of the text after some
introductory sentences.
Step 2: Read the answers and find the key words related to main idea.
Step 3: Eliminate wrong answers and choose the best answer.
Example:
Jogging is a popular form of exercise and recreation in which a person runs at a steady,
moderate pace. The actual pace depends on the individual?s ability, but it should be one at
which the jogger can talk without becoming breathless. Since the mid-1960?s, millions of
people in the United States and other countries have started to jog. The popularity of
jogging stems from its healthful benefits and its simplicity. Popular (a)
What is the main topic of the passage?
A. the history of jogging C. the introduction about jogging
B. people like jogging D. benefits of jogging

Jogging builds and maintains physical fitness by improving the function of the circulatory
and respiratory systems. It strengthens leg muscles and aids in weight control. Jogging also
helps relieve mental stress and provides an opportunity to enjoy the outdoors.
What is the main idea of the 2nd paragraph?
A. Jogging makes us healthier.
B. The physical and mental benefits of jogging.
C. Mental stress disappears when you jog.
D. Jogging is a good way to be out of door.
Practice 1
In the future, entertainment will be faster and easier to use. We?ll stream video and music
around the home and we?ll have unlimited access to TV programs. We won?t store media
files at home but in a <cloud= and we will interact naturally with technology using motion,
1
Downloaded by Nhi An (annhinguyendang@gmail.com)
lOMoARcPSD|15007663

TRAN VAN HAU (ERIC TRAN) VSTEP – READING B1 UPDATED 2019


voice and touch. Games, films, and TV programs will blend together and become
interactive. If we watch a film, we?ll able to choose the ending. Home cinemas will get
better and the experience will be like a real cinema. What?s the wildest prediction for future
entertainment? We will physically experience the action in a film and even feel the
characters? emotions!
What is the passage about?
a. Entertainment in the future
b. Reasons for fast entertainment in the future.
c. TV program in the future
d. The negative effects of entertainment.
Practice 3
Ocean water plays an indispensable role in supporting life. The great ocean basins hold
about 300 million cubic miles of water. From this vast amount, about 80,000 cubic miles of
water are sucked into the atmosphere each year by evaporation and returned by
precipitation and drainage to the ocean. More than 24,000 cubic miles of rain descend
annually upon the continents. This vast amount is required to replenish the lakes and
streams, springs and water tables on which all flora and fauna are dependent. Thus, the
hydrosphere permits organic existence.
What is the best title for this passage?
a. How to get water
b. The importance of conserving water.
c. The importance of treated water
d. How to maintain the fresh water
Practice 4
People who break the law go to prison. While they are in prison, they watch television,
read books, play some sports and usually do some boring jobs. Many of them stay there for
a long time.

It?s expensive because it costs almost $40,000 a year to keep each prisoner. To cut these
costs, prisons in Australia have started teaching prisoners new skills: how to make model
animals which tourists want to buy. Each prisoner can make ten models a day. They sell the
models for around forty dollars each.

It?s a great idea! The prisoners enjoy making these things. People like buying things made
by prisoners. The prisons make a lot of money from the sale of the models. Everyone wins!
Some of the prisoners have decided to start their own business when they get out prison.
Other countries are thinking of doing the same things as Australia, too.
What is the main topic of this passage?
a. The modern prison in Australia
b. The rebuilt prison in Australia
c. Reasons why people in Australia want to go in prison.

2
Downloaded by Nhi An (annhinguyendang@gmail.com)
lOMoARcPSD|15007663

TRAN VAN HAU (ERIC TRAN) VSTEP – READING B1 UPDATED 2019


d. The prison where people can enjoy their life.

II. READING FOR SPECIFIC INFORMATION


A - Find factual information
In the Reading section of the VSTEP test, you may be asked questions about factual information. The
answers to these multiple-choice questions are often restatements of what is given in the passage.
This means that the correct answer often expresses the same idea as what is written in the passage but
that the words are not exactly the same. The answers to these questions are generally given in order
in the passage, and the questions generally indicate which paragraph contains the answers, so the
answers are not too difficult to locate.
Questions:
- According to paragraph X………………..
- It is stated/indicated/mentioned in paragraph X……………………….
How to answer
1. Choose a key word or idea in the question.
2. Skim the appropriate paragraph for the key word or idea.
3. Read the sentence that contains the key word or idea carefully.
4. Eliminate any definitely wrong answers, and choose the best answer from the remaining
choices.
Example:
A ghost word is a nonexistent word that has made its way into a reference work by mistake.
One well-known example of a ghost word is the word Dord, which appeared in a 1934
American dictionary defined as density, as it is used in physics and chemistry. Dord was added
to the dictionary when a typesetter who was making entries into the dictionary misread the
entry 0 or d and typed it as Dord. In reality, the letter d (or its capitalized version) was used to
refer to density in physics or chemistry. When the error was discovered, the ghost word Dord
was removed from the dictionary.
Not all ghost words are recognized as errors, removed from reference works, and forgotten.
One example of a well-established ghost word is the word syllabus. The Roman writer Cicero
had correctly used the Latin word syllabus in his writings to refer to the title and author label
on a manuscript. In a 1470 edition of Cicero's works, syllabus was miswritten as syllabus; the
miswritten ghost word syllabus has now achieved status as a commonly used word referring to
an outline of the contents of a course.
According to paragraph 1, the word Dord:
a. has appeared in numerous physics and chemistry texts
b. was mistakenly added to a dictionary
c. can be used in physics and chemistry to refer to density
d. can still be found in dictionaries

3
Downloaded by Nhi An (annhinguyendang@gmail.com)
lOMoARcPSD|15007663

TRAN VAN HAU (ERIC TRAN) VSTEP – READING B1 UPDATED 2019


B- Understand negative facts
In the Reading section of the VSTEP test, you will sometimes be asked to find an answer that
is not stated, or not mentioned, or not true in the passage. This type of question really means
that three of the answers are stated, mentioned, or true in the passage, while one answer is not.
You should note that there are two kinds of answers to this type of question:
1. there are three answers that are true and one that is not true according to the passage, or
2. there are three true answers and one that is not stated or mentioned in the passage.
Questions:
It is NOT stated It is NOT mentioned
It is NOT discussed It is NOT true
It is NOT indicated All of the following are true EXCEPT
Find the answer: These answers are generally found in order in the passage, and the
paragraph where the answer is found is generally indicated in the question.
How to answer:
1. Choose a key word in the question.
2. Scan the appropriate place in the passage for the key word (or related idea).
3. Read the sentence that contains the key word carefully.
4. Look for the answers that are definitely true according to the passage. Eliminate those answers.
5. Choose the answer that is not true or not discussed in the passage.
Example:
Passage 1-
Computers changed life for all of us. Two computer inventions in particular changed the way
we communicate: email and the World Wide Web.
Email, or electronic mail, allows us to send messages without paper or a stamp. In an instant, a
person at a computer anywhere in the world can send and receive messages. Ray Tomlinson
sent the first email in 1971. He worked for a company that was doing projects for the U.S
military. Their goal was to create a new way for people to communicate with each other.
The company created a network called ARPANET. Computer programs helped organize email
on the network. Once only the military used email. Now anyone can send and receive email.
Today, there are about 600 million email accounts.
The invention of this network led to the development of the World Wide Web. Tim Berners –
Lee created the <WEB= in 1990. It allows us to connect to people and businesses around the
world. It lets us do things in seconds that used to take a long time. We can do research and
shop online. We can go different websites and compare prices. We can find maps or plan trips.
We can buy airplane tickets or get hotel rooms without making a phone call. Computers make
our life easier. Now, we can do almost anything without leaving our desks.
1. What is the main topic of this paragraph?
A. Instant messaging
B. Shopping in the Internet
4
Downloaded by Nhi An (annhinguyendang@gmail.com)
lOMoARcPSD|15007663

TRAN VAN HAU (ERIC TRAN) VSTEP – READING B1 UPDATED 2019


C. Changes in Technology
D. The importance of emails.
2. What is NOT true about the email?
A. Email doesn?t use paper
B. Email is sent and received worldwide.
C. Email was first used in 1971
D. Email has a stamp.
3. To whom was email useful before every one can use it?
a. A company
b. The military
c. Programmer
d. Tim Berners – Lee
4. When was the World Wide Web developed?
a. 30 years ago
b. 42 years ago
c. 28 years ago
d. 17 years ago
5. What is the benefit of shopping online?
A. It helps people find the cheapest price for something
B. It helps people find the location of the Shop
C. It helps create the Web
D. It helps people shop whenever the shops are closed
Passage 2
Sequoyah was a young Cherokee Indian, son of a white trader and an Indian woman. At an
early age, he became fascinated by <the talking leaf=, an expression that he used to describe the
white man?s written records. Although many believed this <talking leaf= to be the gift from the
Great Spirit, Sequoyah refused to accept that theory. Like other Indians of the period, he was
illiterate but his determination to change the situation led to the invention of a unique 86-
character alphabet based on the sound patterns that he heard.
His family and friends thought him mad, but while recovering from a hunting accident, he
diligently and independently set out to create a form of communication for his own people as
well as for other Indians. In 1821, after twelve years of work, he had successfully developed a
written language that would enable thousands of Indians to read and write.
Sequoyah?s desire to preserve words and events for later generations has caused him to be
remembered among the important inventors. The giant redwood trees of California, call
<sequoias= in his honor, will further imprint his name in history.
1) What is the most important reason that Sequoyah will be remembered?
A/ California redwoods were named in his honor.
B/ He was illiterate.
C/ He created a unique alphabet.
D/ He recovered from his madness and helped mankind.
2) How did Sequoyah?s family react to his idea of developing his own <talking leaf=?
A/ They arranged for his hunting accident.
B/ They thought he was crazy.

5
Downloaded by Nhi An (annhinguyendang@gmail.com)
lOMoARcPSD|15007663

TRAN VAN HAU (ERIC TRAN) VSTEP – READING B1 UPDATED 2019


C/ They decided to help him.
D/ They asked him to teach them to read and write.
3) What caused Sequoyah to develop his alphabet?
A/ People were writing things about him that he couldn?t read.
B/ He wanted to become famous.
C/ After his hunting accident, he needs something to keep him busy.
D/ He wanted the history of his people preserved for future generations.
4) How would you describe Sequoyah?
A/ determined B/ mad C/ backwards D/ gentle
5) Which of the following is NOT TRUE?
A/ Sequoyah developed a form of writing with the help of the Cherokee tribe.
B/ Sequoyah was a very clever young man.
C/ Sequoyah spent twelve years developing his alphabet.
D/ Sequoyah was honored by having some named trees after him.
Practice 3
Many people are quite familiar with rainbows, but few are as familiar with moonbows. Rainbows
are caused by sunlight hitting raindrops and bouncing back. You can see a rainbow when the sun
low in the sky behind you and it is raining ahead of you. Light from the Sun reflects off the inside
surfaces of raindrops and is bent as it travels through them. It appears as a band of colors because
each of the colors in sunlight is bent to a different angle.
Moonbows are far less common than are rainbows, but they are formed in much the same way.
They require a very specific set of circumstances to occur. When they do occur, they occur just
after a full Moon, a Moon at its brightest, has risen in the east and just after the Sun has set in the
west, and it must also be raining in the west. In this situation, a moonbow may be visible to you if
you are facing west and if the Moon is behind you. Light from the bright Moon reflects off the
inside surfaces of the raindrops in the west and bends the colors to create a moon bow.
1: According to paragraph 1, it is NOT true that you can see a rainbow when
a. the Sun is low in the sky
b. the Sun is in front of you
c. it is raining in front of you
d. the Sun's rays are reflected off the raindrops
2. It is NOT indicated in paragraph 2:
a. where the moon must be in the sky for a moon bow to occur
b. at what time of day moon bows occur
c. which direction you must be facing to see a moonbow
d. in which parts of the world moon bows occur
Practice 4
Members of the flatfish family, sand dabs and flounders, have an evolutionary advantage over
many colorfully decorated ocean neighbors in that they are able to adapt their body coloration
to different environments. These aquatic chameleons have flattened bodies that are well-suited
to life along the ocean floor in the shallower areas of the continental shelf that they inhabit.
6
Downloaded by Nhi An (annhinguyendang@gmail.com)
lOMoARcPSD|15007663

TRAN VAN HAU (ERIC TRAN) VSTEP – READING B1 UPDATED 2019


They also have remarkably sensitive color vision that registers the subtlest gradations on the
sea bottom and in the sea life around them. Information about the coloration of the
environment is carried through the nervous system to chromatophores, which are pigment-
carrying skin cells. These chromatophores13 are able to accurately reproduce not only the
colors but also the texture of the ocean floor. Each time that a sand dab or flounder finds itself
in a new environment, the patterns on the body of the fish adapts to fit in with the color and
texture around it.
1. It is NOT stated in the passage that sand dabs
A. are a type of flatfish
B. are in the same family as flounders
C. have evolved
D. are colorfully decorated
2. According to the passages, it is NOT true that sand dabs and flounders
A. have flattened bodies
B. live along the ocean floor
C. live in the deepest part of the ocean
D. live along the continental shelf
3. All of the following are stated about the vision of sand dabs and flounders EXCEPT that they
are…………………
A. overly sensitive to light
B. able to see colors
C. able to see the sea bottom
D aware of their surroundings
SESSION 2
III. READING – RECOGNIZE REFERENTS
You may be asked to determine the referent for a particular pronoun or adjective, or find the
referent for a variety of words. A referent generally precedes the pronoun or adjective in the
passage, so to answer this question type, you should study the content around that.

QUESTIONS ABOUT REFERENTS

HOW TO IDENTIFY THE


QUESTION
The word X refers to ...

The pronoun or adjective is highlighted in the passage. The


HOW TO FIND THE ANSWER
referent is generally in front of the highlighted word.
1. Locate the highlighted pronoun or adjective.
2. Look before the highlighted word for nouns that agree with the
HOW TO ANSWER THE highlighted word.
QUESTION
3. Try each of the nouns in the context around the highlighted
word.
4.. Eliminate any definitely wrong answers, and choose the
7
Downloaded by Nhi An (annhinguyendang@gmail.com)
lOMoARcPSD|15007663

TRAN VAN HAU (ERIC TRAN) VSTEP – READING B1 UPDATED 2019


best answer from remaining choices.
Ex:
A tornado is created when warm, moist air rises from the ground and comes into contact with
a mass of colder air at the bottom of a thundercloud. The rising air pushes against the colder
air, and the rotation of the earth causes the air to spin, in much the same way that water in a
sink spins as it goes down a drain.
The word <it= in line 4 refers to…………………….
A. Way B. water C. sink D. drain
Practice 1:
Scientists have proposed several explanations for the Flynn effect. Some suggest that the
improved test scores simply reflect an increased exposure to tests in general. Because we take
so many tests, we learn test-taking techniques that help us perform better. Others have
pointed to better nutrition, which results in babies being born larger, healthier, and with more
brain development than in the past. Another possible explanation is a change in educational
styles - children are encouraged to discover things for themselves rather than just memorizing
information. This could prepare people to do the kind of problem-solving that intelligence
tests require.
In line 3, Who does the word "_others_" refer to?
A: babies B: scientists C: people in general D: people who take tests
Practice 2:
A tornado is created when warm, moist air rises from the ground and comes into contact with a
mass of colder air at the bottom of a thundercloud. The rising air pushes against the colder air,
and the rotation of the earth causes the air to spin, in much the same way that water in a sink
spins as it goes down a drain.
The pressure at the center of a tornado is much lower than that in the air surrounding the
tornado. The low pressure creates a funnel in the middle of the tornado, which causes
destruction by acting much like a vacuum cleaner and sucking up whatever is in its path.
In line 1, the word < that= refers to…………………
A. Pressure B. center C. tornado D. air
Practice 3
Many types of animals combine the advantages of family association with those conferred by
membership of still larger groups. Bees congregate in hives; some fish move in schools; ants
gather in mounds, wolves live in packs; deer associate in herds. The main advantage of
membership in a mass community is the safety that it provides. A large group of pray may be
easier for a predator to find at any given point than is a small one, and a predator may think
twice before taking on such a group; if a predator does decide to challenge a large group, it
may merely encounter a confusing mass of moving bodies and possibly may not succeed in its
primary goal.
In line 1, The word <those= in the passage refers to………
A. Types B. animals C. advantages D. membership
8
Downloaded by Nhi An (annhinguyendang@gmail.com)
lOMoARcPSD|15007663

TRAN VAN HAU (ERIC TRAN) VSTEP – READING B1 UPDATED 2019


Practice 4
Horatio Alger, Jr. (1832-1899) was the author of more than 100 books for boys in the second
half of the nineteenth century that focused on the theme of success coming to those who work
hard to achieve it. The son of a minister, Alger came from a prominent Massachusetts family.
He graduated with honors from Harvard in 1852 and graduated from the Cambridge Divinity
school eight years later. He served as a minister for a short time before moving to New York
City in 1866 to devote his time to writing inspirational books for boys.
In many of his books, he wrote about the poor and homeless children of the slums of New
York City, seeing them as unfortunate pawns of society who, if only given the opportunity,
could improve their lot. A general plot line that he followed often was of a poor boy who
managed to achieve a respectable and successful life by working hard and taking advantage
of opportunities presented. Though his writing style was characterized by simplicity and
repetition, it was well received by his target audience; his books were enormously popular,
selling millions of copies well into the first few decades of the twentieth century.
1. The word < that= in paragraph 1 refers to ………………….
A. Author B. books C. Boys D. Half
2. The word < it= in paragraph 1 refers to …………..
A. The second half B. The nineteenth century C. 100 D. Success

IV. READING – GETTING MEANING FROM CONTEXT


You can find the meaning of the words or phrases in the context. The context is the other
words in the sentence or paragraph.
1. Definitions and Italics
- Definitions give the meaning of a word or phrase. It can be a short term. It often comes
in a sentence after the verbs > be= or < mean=
Ex: Stage is the area for actors to perform.
- The definitions or meaning explanations can be in a quotation marks <…=
ex: One of the meaning of the word abroad is < in a foreign place=
2. Punctuation and Phrase clues
Short definitions of new vocabulary items sometimes appear between or after punctuation
marks such as (), (,), (-) (;).
The pharse <in other words and that is, or i.e (That is) can be clues to the meaning of
vocabulary.
Ex:
- Many fast – food restaurants are part of fast food chains (eating places with the same
name or company owner)
- Some people buy vegetables, fruits without additives (chemical substances)
- The family is a social institution; in other words, it is an organization with a purpose
inside a human community – that is, among the people living together in a certain area.
HOW TO ANSWER
9
Downloaded by Nhi An (annhinguyendang@gmail.com)
lOMoARcPSD|15007663

TRAN VAN HAU (ERIC TRAN) VSTEP – READING B1 UPDATED 2019


1. Find the word or phrase in the passage.
2. Read the sentence that contains the word or phrase carefully.
3. Look for context clues to help you to understand the meaning.
4. Choose the answer that the context indicates.
Example:
Spiders are unique among animals in that many spin webs out of silk. The webs trap insects
and other prey to be eaten later. Larger spiders, such as wolf spiders, huntsman, and
tarantulas, attack prey rather than spin webs. One spider that was recently discovered even
flings itself at prey like a slingshot! Most spiders eat insects, but tarantulas and larger spiders
will eat small lizards and mammals as well.
What does "unique" mean in the following sentence:
a.) old b.) fun c.) similar to d.) different
Practice 1:
The Native American Trade is the trade between Europeans, their North American
descendants, and the indigenous people of North America who are now known as Native
Americans in the United States, First Nations in Canada, whom were formerly known as
Indians. Indian Trade is the term used to describe the people involved in the trade, which
began in the 1500s, and had various products in different regions and eras. In most of Canada
the term is synonymous with the fur trade such as beaver fur which, from the European point
of view, was the most valuable product of the trade.
The word "indigenous" in paragraph 1 is closest in meaning to___________
A. Intelligent B. Foreign C. Native D. Friendly
The word <eras= is clost in meaning to___________
A. Period b. money C. people C. product
Practice 2
Parasitic plants are plants that survive by using food produced by host plants rather than by
producing their own food from the Sun?s energy. Because they do not need sunlight to survive,
parasitic plants are generally found in umbrageous areas rather than in areas exposed to direct
sunlight. Parasitic plants attach themselves to host plants, often to the stems or roots, by means of
haustoria, which the parasite uses to make its way into the food channels of the host plant and
absorb the nutrients that it needs to survive from the host plant.
The worId?s heaviest flower, a species of rafflesia, is a parasite that flourishes among, and lives off
of, the roots of jungle vines. Each of these ponderous blooms can weigh up to 15 pounds (7 kg)
and can measure up to 3 feet (1 m) across
1. The word umbrageous in paragraph 1 is closest in meaning to:
a) moist b) well lit c) shaded d) buried
2. The phrase make its way into in paragraph 1 is closest in meaning to
a) outline b) eat c) develop d) penetrate
4. The word ponderous in paragraph 2 is closest in meaning to

10
Downloaded by Nhi An (annhinguyendang@gmail.com)
lOMoARcPSD|15007663

TRAN VAN HAU (ERIC TRAN) VSTEP – READING B1 UPDATED 2019


a) smelly b) hidden c) mature d) heavy
5. The word across in paragraph 2 could best be replaced by
a) in diameter b) on the other side c) at a distance d) inside and out
V.MAKE INFERENCES
You may be asked to answer a multiple choice question by drawing a conclusion from a
specific detail or details in the passage.
How to identify questions:
+ It is implied in paragraph X……………..
+ It can be inferred from paragraph X………….
+ What is most likely…………………..?
+ What probably happened…………….?
How to answer:
- Choose the key word or phrases in the question.
- Scan the passage for they key word or phrase
- Carefully read the sentence that contains they key word.
Example:
When I opened the first >Body Shop? in 1976 my only object was to earn enough to feed my children
. Today >The Body Shop? is an international company rapidly growing all around the world . In the
years since we began I have learned a lot. Much of what I have learned will be found in this book, for
I believe that we, as a company, have something worth saying about how to run a successful business
without giving up what we really believe in.
It?s not a normal business book, nor it is just about my life. The message is that to succeed in
business you have to be different. Business can be fun, a business can be run with love and it can do
well. In business, as in life, I need to enjoy myself, to have a feeling of family and to feel excited by
the unexpected. I have always wanted the people who work for > The Body Shop? to feel the same
way.
1. What would someone learn from this text?
A. what the writer?s family is like B. how to write a book about business
C. how to make a lot of money D. what the writer?s book is about
2. According to paragraph 2, what kind of workers does the writer like to employ?
A. workers who have the same attitudes as she does
B. workers who get on well with the public
C. workers who can explain her ideas
D. workers who have their own families
Practice
Long before coins were invented, metals such as gold, silver, copper, and bronze were used as
a medium of exchange for trade. However, each piece of metal had to be weighed each time it
was used in trade to establish its value. The Lydians of western Anatolia were the first to begin

11
Downloaded by Nhi An (annhinguyendang@gmail.com)
lOMoARcPSD|15007663

TRAN VAN HAU (ERIC TRAN) VSTEP – READING B1 UPDATED 2019


producing metal coins in standard weights, the seventh century B.C., impressing a seal into the
coin to indicate its value.
One such coin minted during the time of Lydian King Croesus, who ruled from 560 B.C. to
546 B.C., has been recovered by archeologists; this coin is imprinted with the heads and
forelegs of two animals, a bull and a lion, who are facing each other. It was not until 525 B.C.
that coins with images on both sides came into being.
1: It can be inferred from paragraph 1 that gold came into use as a medium of exchange
a. before 700 B.C.
b. after 700 B.C.
c. during the reign of King Croesus
d. in 525 B.C.
2: It is implied in paragraph 2 that coins from the time of King Croesus were all
a. imprinted on one side only
b. made of gold
c. imprinted with two animals
e. imprinted on both sides
VI.INFER RHERICAL PURPOSE
Rhetorical purpose questions may ask why the author:
+ Introduces a topic
+ Support an idea
+ Uses an example of
+ Makes a comparison
+ Mentions a specific piece of information
How to identify questions
+ Why does the author mention/ talk about………………?
+ The author mentions X in order to………..?
+ Why is X is mentioned in the passage?
How to answer:
- Study the bold information carefully.
- Study the context around the information which is bolded.
- Draw a conclusion and read the answer choices and choose the best answer.
- Read the answer choices, and eliminate any definitely wrong answers.
- Choose the best answer from the remaining choices.
Practice 1
One more familiar use of electrochemistry that has made its way into the mainstream is
xerography, a process for replicating documents that is dependent on photoconductive
materials. A photoconductive material is an insulator in the dark but becomes conductive
when exposed to bright light. When a photocopy is being made, an image of a document is
12
Downloaded by Nhi An (annhinguyendang@gmail.com)
lOMoARcPSD|15007663

TRAN VAN HAU (ERIC TRAN) VSTEP – READING B1 UPDATED 2019


projected onto the surface of a rotating drum, and bright light causes the photoconductive
material on the surface of the drum to become conductive.
As a result of the conductivity, the drum loses its charge in the lighted areas, and toner (small
grains to which dry ink adheres) attaches itself only to the darker parts of the image. The
grains are then carried to a sheet of paper and fused with heat. When a laser printer is used, the
image is projected by means of a laser beam, which creates a brighter light and a greater
contrast between lighter and darker areas and therefore results in sharper printed images.
1. The author begins the first paragraph with One more familiar use of electrochemistry in
order to………………
a. explain that xerography is one of the less familiar uses of electrochemistry
b. make it clear that electrochemistry requires photoconductive materials
c. show that xerography is the only known use for electrochemistry
d. indicate that other less familiar uses have already been discussed
2.The author places the phrase small grains to which dry ink adheres in parenthesis in order
to…………………………..
a. provide information that contradicts the previous statement
b. provide another example of conductivity
c. provide further detail information about toner
d. provide an alternate explanation for the effectiveness of toner
Practice 2
The Hudson River is one of the most significant rivers in America, though it is not the largest.
It was named after Henry Hudson, an English explorer who charted the river for the Dutch
East India Company. The Hudson goes for 315 miles, starting in the Adirondack Mountains in
New York State. It enters the ocean around New York City.
Washington Irving, an American writer, was one of the first to write about the Hudson River.
He lived in the Hudson River Valley and wrote about the unique characteristics of this area. He
developed the first kind of American folklore and told a story of a man named Rip Van
Winkle. Rip Van Winkle fell asleep one day while climbing the mountains around the ·
Hudson River. He did not wake up for twenty years. When he finally woke up, America
had become a free country.
1. Why does the author mention "Washington Irving" in paragraph 2?
a. To give an example of an American writer
b. To prove that many writers live in New York
c. To explain why Rip Van Winkle did not wake up
d. To show that writers wrote about the Hudson River
Practice 3:
Babe Ruth is one of the greatest and the most popular American baseball players that ever
lived. He was one of the first five players elected to the Baseball Hall of Fame. Though over
sixty years have passed since his death, almost every kid in the U.S. today grows up knowing
his name.
13
Downloaded by Nhi An (annhinguyendang@gmail.com)
lOMoARcPSD|15007663

TRAN VAN HAU (ERIC TRAN) VSTEP – READING B1 UPDATED 2019


He was born in 1895 as George Herman Ruth, Jr. in Maryland and grew up without a mother.
At the age of seven, his father sent him to St. Mary's, a Catholic school where Babe Ruth
lived for the next twelve years of his life. There he learned to play baseball but was isolated
from his family.
He started playing in the majors as a pitcher when he was only nineteen years old. He was very
successful and before long he was traded to the Boston Red Sox. There he won a World Series
in 1915 but fame as a hitter was still a long way off. In 1919, Babe Ruth was sold to the New
York Yankees for 125 ,000 dollars cash and three 25,000 dollar notes. He then started to
become the legend that everyone knows about today. By the end of his career, Babe Ruth had
hit 714 home runs and had scored 2,213 runs.
1. Why does the author talk about "St. Mary's"?
a. To explain why Ruth wasn't very nice
b. To give some background information about Ruth
c. To show why Ruth joined the New York Yankees
d. To provide an example of a place to learn baseball
2. Why does the author mention that almost all American kids today grow up knowing
Babe Ruth's name?
a. To show how admired he is in the U.S.
b. To explain the American kids' wishes
c. To prove that he was the greatest pitcher
d. To give an example of how Americans love baseball
VII. DETERMINE THE TONE, PURPOSE, OR COURSE
The following chart outlines the key information that you should remember about questions on
the tone, purpose, or course.

TONE, PURPOSE, OR COURSE

What is the tone of the passage?


HOW TO
IDENTIFY THE What is the author?s purpose in this passage?
QUESTION In which course would this reading be assigned?

1. Skim the passage looking for clues that the


author is showing some emotion.
Tone 2. Choose the answer that identifies the emotion.

1. Study the main idea in the topic sentence and the


details used to support the main idea.
Purpose 2. Draw a conclusion about the purpose.

HOW TO 1. Study the main idea in the topic sentence and the
ANSWER THE details used to support the main idea.
QUESTION course 2. Draw a conclusion about the course.

14
Downloaded by Nhi An (annhinguyendang@gmail.com)
lOMoARcPSD|15007663

TRAN VAN HAU (ERIC TRAN) VSTEP – READING B1 UPDATED 2019

Ex:
Truman Capote?s In Cold Blood (1966) is a well-known example of the <nonfiction novel,= a popular
type of writing based upon factual events in which the author attempts to describe the underlying
forces, thoughts, and emotions that lead to actual events. In Capote?s book, the Line author describes
the sadistic murder of a family on a Kansas farm, often showing the point of view of the killers. To
research the book, Capote interviewed the murderers, and he maintains that his book presents a
faithful reconstruction of the incident.
1. The purpose of this passage is to
(A) discuss an example of a particular literary genre
(B) tell the story of In Cold Blood
(C) explain Truman Capote's reasons for writing In Cold Blood
(D) describe how Truman Capote researched his nonfiction novel
2. Which of the following best describes the tone of the passage?
(A) Cold (B) Sadistic (C) Emotional (D) Descriptive
3. This passage would probably be assigned reading in which of the following courses?
(A) Criminal Law (B) American History
(C) Modern American Novels (D) Literary Research
Practice 1
The rate at which the deforestation of the world is proceeding is alarming. In 1950
approximately 25 percent of the earth?s land surface had been covered with forests, and less
than twenty-five years later the amount of forest land was reduced to 20 percent. This decrease
from 25 Line percent to 20 percent from 1950 to 1973 represents an astounding 20 million
square kilometers of forests. Predictions are that an additional 20 million square kilometers of
forest land will be lost by 2020.
The majority of deforestation is occurring in tropical forests in developing countries, fueled
by the developing countries? need for increased agricultural land and the desire on the part of
developed countries to import wood and wood products. More than 90 percent of the plywood
used in the United States, for example, is imported from developing countries with tropical
rain forests. By the mid-1980s, solutions to this expanding problem were being sought, in the
form of attempts to establish an international regulatory organization to oversee the use of
tropical forests.
1. The author?s main purpose in this passage is to………………………..
(A) cite statistics about an improvement on the earth's land surface
(B) explain where deforestation is occurring
(C) make the reader aware of a worsening world problem
(D) blame developing countries for deforestation
2. Which of the following best describes the tone of the passage?
(A) Concerned (B) Disinterested (C) Placid (D) Exaggerated
15
Downloaded by Nhi An (annhinguyendang@gmail.com)
lOMoARcPSD|15007663

TRAN VAN HAU (ERIC TRAN) VSTEP – READING B1 UPDATED 2019


3. This passage would probably be assigned reading in which of the following courses?
(A) Geology (B) Geography (C) Geometry (D) Marine Biology

Practice 2:
Military awards have long been considered symbolic of royalty, and thus when the United
States was a young nation just finished with revolution and eager to distance itself from
anything tasting of monarchy, there was strong sentiment against military decoration. For a
century, from the end of the Revolutionary War until the Civil War, the United States awarded
no military honors. The institution of the Medal of Honor in 1861 was a source of great
discussion and concern. From the Civil War until World War I, the Medal of Honor was the
only military award given by the United States government, and today it is awarded only in the
most extreme cases of heroism. Although the United States is still somewhat wary of granting
military awards, several awards have been instituted since World War I.
The tone of this passage is_____________________
(A) angered
(B) humorous
(C) outraged
(D) informational

VIII. READING – RECOGNIZE ORGANIZATION


How to identify questions:
Dạng 1
Questions about organization of ideas
How to identify the How is the information in the passage organized?
question - Present a definition + examples
- Present advantages + disadvantages
- Causes + effects
- Agreement + Disagreement
- Popularity + Reasons

How to answer the - Read the first or second line of each paragraph
question - Look for words that show the relationship between the
paragraph
- Choose the answer that best express the relationship.

Ex:
If asked who invented the game of baseball, most Americans would probably reply that it was
Abner Doubleday. At the beginning of this century, there was some disagreement over how the
game of baseball had actually originated, so sporting goods manufacturer Spaulding
inaugurated a commission to research the question. In 1908 a report was published by the
commission in which Abner Doubleday, a U.S. Army officer from Cooperstown, New York,
16
Downloaded by Nhi An (annhinguyendang@gmail.com)
lOMoARcPSD|15007663

TRAN VAN HAU (ERIC TRAN) VSTEP – READING B1 UPDATED 2019


was given credit for the inventionof the game. The National Baseball Hall of Fame was
established in Cooperstown in honor of Doubleday.
Today, most sports historians are in agreement that Doubleday really did not have much to do
with the development of baseball. Instead, baseball seems to be a close relation to the English
game of rounders and probably has English rather than American roots.

The second paragraph


(A) provides examples to support the ideas in the first paragraph
(B) precedes the ideas in the first paragraph
(C) outlines the effect of the idea in the first paragraph
(D) refutes the idea in the first paragraph
Dạng 2:
+ The paragraph preceding the passage probably?
+ What is the most likely in the passage following the passage?
How to answer the question
- Read the first line for a preceding
- Read the last line for a following
- Draw a conclusion about what comes before or after.
- Choose the answer that is reflected in the first or last line of the passage.
Ex:
Another myth of the oceans concerns Davy Jones, who in folklore is a mean-spirited sovereign of
the ocean?s depths. The name <Jones= is thought by some etymologists to have been derived from
the name <Jonah=, the Hebrew prophet who spent three days in a whale?s belly.
According to tradition, any object that goes overboard and sinks to the bottom of the ocean is said
to have gone to Davy Jones?s locker, the ocean-sized, mythical receptacle for anything that falls
into the water. Needless to say, any sailor on the seas is not so eager to take a tour of Davy Jones?s
locker, although it might be a rather interesting trip considering all the treasures located there.
The paragraph preceding this passage most probably discusses
(A) the youth of Davy Jones
(B) Davy Jones?s career as a sailor
(C) A different traditional story from the sea
(D) preparing to travel on the ocean
Practice 1
Conflict within an organization is not always viewed as undesirable. In fact, various managers
have widely divergent ideas on the value that conflict can have.
According to the traditional view of conflict, conflict is harmful to an organization. Managers with
this traditional view of conflict see it as their role in an organization to rid the organization of any
possible sources of conflict.
The interactionist view of conflict, on the other hand, holds that conflict can serve an
17
Downloaded by Nhi An (annhinguyendang@gmail.com)
lOMoARcPSD|15007663

TRAN VAN HAU (ERIC TRAN) VSTEP – READING B1 UPDATED 2019


important function in an organization by reducing complacency among workers and causing
positive changes to occur. Managers who hold an interactionist view of conflict may actually take
steps to stimulate conflict within the organization.
How is the information in the passage organized?
a. The origin of ideas about conflict is presented.
b. Contrasting views of conflict are presented.
c. Two theorists discuss the strengths and weaknesses of their views on conflict.
d. Examples of conflict within organizations are presented.
Practice 2
IQ or intelligence quotient, is defined as the ratio of a person's mental age to chronological age,
with the ratio multiplied by 100 to remove the decimal. Chronological age is easily determined;
mental age is generally measured by some kind of standard test and is not so simple to define.

In theory, a standardized IQ test is set up to measure an individual's ability to perform intellectual


operations such as reasoning and problem solving. These intellectual operations are considered to
represent intelligence.

In practice, it has been impossible to arrive at consensus as to which types of intellectual


operations demonstrate intelligence. Furthermore, it has been impossible to devise a test without
cultural bias, which is to say that any IQ tests so far proposed have been shown to reflect the
culture of the test makers. Test takers from that culture would, it follows, score higher on such a
test than test takers from a different culture with equal intelligence.
What type of information is included in the first paragraph?
a. An argument
b. A definition
c. An opinion
d. A theory
Practice 3
Another program instrumental in the popularization of science was Cosmos. This series, broadcast
on public television, dealt with topics and issues from varied fields of science. The principal writer
and narrator of the program was Carl Sagan, a noted astronomer and Pulitzer Prize-winning
author.
The paragraph preceding this passage most probably discusses
a. a different scientific television series
b. Carl Sagan?s scientific achievements
c. the Pulitzer Prize won by Carl Sagan
d. public television
Practice 4
When a strong earthquake occurs on the ocean floor rather than on land, a tremendous force is
exerted on the seawater and one or more large, destructive waves called tsunamis can be formed.
18
Downloaded by Nhi An (annhinguyendang@gmail.com)
lOMoARcPSD|15007663

TRAN VAN HAU (ERIC TRAN) VSTEP – READING B1 UPDATED 2019


Tsunamis are commonly called tidal waves in the United States, but this is really an inappropriate
name in that the cause of the tsunami is an underground earthquake rather than the ocean's tides.

Far from land, a tsunami can move through the wide open vastness of the ocean at a speed of 600
miles (900 kilometers) per hour and often can travel tremendous distances without losing height
and strength. When a tsunami reaches shallow coastal water, it can reach a height of 100 feet (30
meters) or more and can cause tremendous flooding and damage to coastal areas.

1. The paragraph preceding the passage most probably discusses


a. tsunamis in various parts of the world
b. the negative effects of tsunamis
c. Land-based earthquakes
d. the effect of tides on tsunamis
2. Which of the following is most likely the topic of the paragraph following the passage?
a. The causes of tsunamis
b. The destructive effects of tsunamis on the coast
c. The differences between tsunamis and tidal waves
d. The distances covered by tsunamis

Practice 5
While draft laws are federal laws, marriage laws are state laws rather than federal; marriage
regulations are therefore not uniform throughout the country. The legal marriage age serves as an
example of this lack of conformity. In most states, both the man and the woman must be at least
eighteen years old to marry without parental consent; however, the states of Nebraska and
Wyoming require the couple to be at least nineteen, while the minimum age in Mississippi is
twenty-one.

If parental permission is given, then a couple can marry at sixteen in some states, and a few states
even allow marriage before the age of sixteen, though a judge's permission, in addition to the
permission of the parents, is sometimes required in this situation. Some states which allow couples
to marry at such a young age are now considering doing away with such early marriages because
of the numerous negative effects of these young marriages.
1. The paragraph preceding the passage most probably discusses
a. state marriage laws
b. the lack of uniformity in marriage laws
c. federal draft laws
d. the minimum legal marriage age
2. The topic of the paragraph following the passage is most likely to be

19
Downloaded by Nhi An (annhinguyendang@gmail.com)
lOMoARcPSD|15007663

TRAN VAN HAU (ERIC TRAN) VSTEP – READING B1 UPDATED 2019


a. disadvantages of youthful marriages
b. reasons why young people decide to marry
c. the age when parental consent for marriage is required
d. a discussion of why some states allow marriages before the age of sixteen

Practice 6
The locations of stars in the sky relative to one another do not appear to the naked eye to change,
and as a result stars are often considered to be fixed in position. Many unaware stargazers falsely
assume that each star has its own permanent home in the nightmare sky.

In reality, though, stars are always moving, but because of the tremendous distances between stars
themselves and from stars to earth, the changes are barely perceptible here. An example of a rather
fast moving star demonstrates why this misconception prevails; it takes approximately 200 years
for a relatively rapid star like Bernard?s star to move a distance in the skies equal to the diameter
of the earth?s moon. When the apparently negligible movement of the stars is contrasted with the
movement of the planets, the stars are seemingly unmoving.

The paragraph following the passage most probably discusses


a. The movement of the planets
b. Bernard?s star
c. The distance from earth to the moon
d. Why stars are always moving
Practice 7
Sunlight appears to have no colors. We call in white light. It is really made up of different colors.
When you see a rainbow, you see white light that has traveled through millions of falling
raindrops.
White light is a mixture of all the colors of the rainbow, orange, yellow, green, blue and violet.
When white light in a drop of rain, it changes direction is called refraction. Some orange colors in
the light change direction more than other and so the white light separate into its different colors.
The paragraph following the passage most probably discusses
a. White light
b. Falling raindrops
c. The change direction
d. The mixture of colour
Practice 8
Skiing has become a way of life for many people. From the moment the first snowflake falls until
the spring thaw, skiers put their skis on their cars and head for the slopes. There are many reasons
behind the popularity of this winter sport.

20
Downloaded by Nhi An (annhinguyendang@gmail.com)
lOMoARcPSD|15007663

TRAN VAN HAU (ERIC TRAN) VSTEP – READING B1 UPDATED 2019


Skiing is a true family sport that can be enjoyed by all people, whether 3 or 93 years old. Being
able to descend a hill, to turn at will, and enjoy nature at its loveliest is a thrill for all age groups.
Skiing is also interesting because it provides a variety of experiences. Snow conditions change
hourly as the temperature and weather conditions change during the day. Moreover, every trail is
different. Rarely does one pass over the same spot twice.

Improvements in ski equipment, clothing, and ski areas have made the sport more pleasurable,
comfortable, and available. Warm, light, down-filled clothing has replaced layers of heavy
sweaters. Ski equipment made with modern materials has made skis and poles lighter, more
flexible, and suited to people of all ages and abilities. The availability of skiing has also been
improved by snow-making equipment. Even in areas of the country that have every little snowfall,
snow can be made if the temperature is below 32 degrees.

For many people, skiing is an opportunity to enjoy the beauty of the out-of-doors, to challenge
their physical abilities, and, finally, to simply have fun. It is a sport enjoyed worldwide and
appears to be gaining in popularity constantly.
1. Which is the following the best organization of the passage?
A. The author presents the advantages and disadvantages of skiing.
B. The author describes skiing.
C. The author explains the popularity of skiing and gives reasons.
D. The author classifies skiing.
2. The paragraph following this passage probably deals with……
A. Snow – making C. Skiing accidents
B. The cost of skiing D. Ski resorts around the world.
IX. Essential Information (Sentence Simplification) Question
In an <Essential Information question=, you will see the question you see an entire sentence
highlighted in the reading passage. The question will ask you to choose which of the 4 answer
option sentences is equal to the highlighted sentence. The correct sentence will be paragraphed
so it is different than the highlighted one, but still convey all the important information.
HOW TO IDENTIFY THE QUESTION
Which of the sentences below best expresses the essential information ... ?
HOW TO FIND THE ANSWER
The targeted sentence is highlighted in the passage. Information to answer question is in the
highlighted sentence and may also be in the context around highlighted sentence.
HOW TO ANWER THE QUESTION
1. Study the highlighted sentence carefully.
2. Break the sentence down into meaningful parts by looking for punctuation transition
expressions.
3. If the highlighted sentence makes references to information outside of the highlighted
sentence, read the context around the highlighted sentence.

21
Downloaded by Nhi An (annhinguyendang@gmail.com)
lOMoARcPSD|15007663

TRAN VAN HAU (ERIC TRAN) VSTEP – READING B1 UPDATED 2019


4. Study the answer choices, and eliminate definitely wrong answers.
5. Choose the best answer from the remaining choices.

EX:
Dissatisfaction with conventional explanations for dinosaur extinctions led to a surprising
observation that, in turn, has suggested a new hypothesis. Many plants and animals
disappear abruptly from the fossil record as one moves from layers of rock documenting
the end of the Cretaceous up into rocks representing the beginning of the Cenozoic (the
era after the Mesozoic). Between the last layer of Cretaceous rock and the first layer of
Cenozoic rock, there is often a thin layer of clay. Scientists felt that they could get an idea of
how long the extinctions took by determining how long it took to deposit this one centimeter
of clay and they thought they could determine the time it took to deposit the clay by
determining the amount of the element iridium (Ir) it contained.

Which of the sentences below best expresses the essential information in the highlighted
sentence in paragraph 4? Incorrect choices change the meaning in important ways or leave
out essential information.
A. The fossil record suggests that there was an abrupt extinction of many plants and
animals at the end of the Mesozoic era.
B. Few fossils of the Mesozoic era have survived in the rocks that mark the end of the
Cretaceous.
C. Fossils from the Cretaceous period of the Mesozoic up to the beginning of the Cenozoic
era have been removed from the layers of rock that surrounded them.
D. Plants and animals from the Mesozoic era were unable to survive in the Cenozoic era.

Practice 1
The whale retained a tail and lacked a fluke, the major means of locomotion in modern cetaceans.
The structure of the backbone shows, however, that Ambulocetus swam like modern whales by
moving the rear portion of its body up and down, even though a fluke was missing. The large
hind legs were used for propulsion in water. On land, where it probably bred and gave birth,
Ambulocetus may have moved around very much like a modern sea lion. It was undoubtedly a whale
that linked life on land with life at sea.

Which of the sentences below best expresses the essential information in the highlighted sentence
in the passage? Incorrect choices change the meaning in important ways or leave out essential
information.
a. Even though Ambulocetus swam by moving its body up and down, it did not have a backbone.
b. The backbone of Ambulocetus, which allowed it to swim, provides evidence of its missing fluke.
c. Although Ambulocetus had no fluke, its backbone structure shows that it swam like modern
whales.

22
Downloaded by Nhi An (annhinguyendang@gmail.com)
lOMoARcPSD|15007663

TRAN VAN HAU (ERIC TRAN) VSTEP – READING B1 UPDATED 2019


d. By moving the rear parts of their bodies up and down, modern whales swim in a different way
from the way Ambulocetus swam.
X. READING – INSERT SENTENCE INTO THE PASSAGE
How to answer the questions:
1. Look at the sentence given for a key words or ideas at the beginning or the end of the
sentence.
2. Read the context before and after the insertion squares for any ideas that relate to the
sentence to be inserted. \
3. Choose the insertion square suitably.
Ex: Examine the four █ in the selection below and indicate at which block the following sentence
could be inserted into the passage:

█ [A] Darwin was born February 12, 1809 in England. █ [B] Although initially entering into
medicine, Darwin chose to pursue his interest in natural science and embarked on 12
www.bestmytest.com 12 a five-year journey aboard the H.M.S. Beagle, a British sloop belonging
to the Royal Navy █ [C] Because of his experience aboard the Beagle, he laid the foundation for
his Theory of Evolution while also establishing himself within the scientific community. █ [D]
The five-year voyage proved to be a major turning point in his life.
A. [A] B. [B] C. [C] D. [D]
Practice 1

The origins of the game of chess are not known with certainty, and traditional stories in a number
of cultures claim credit for developing the game. [A] One legend claims that chess was invented
during the Trojan Wars. [B] According to another legend, chess was developed to depict the
battle between two royal brothers for the crown of Persia. [C] In a third legend, chess was the
creation of the mythical Arab philosopher Sassa. [D]

When one brother was killed, the remaining brother had the game invented to explain the
tragic events to his mother.
A. [A] B. [B] C. [C] D. [D]
Practice 2
Whatever its origins, chess was known to exist in India as early as 500 B.C., and it eventually
spread from India to Persia, where it took on much of the terminology that today is part of the
game. [A] Foot soldiers in the Persian army were called piyadah, which became the pawns of
today's game, and the Persian chariot was a rukh, which became the rook. [B]The Persian king
was the shah, which evolved into the name chess. [C] Shahmat, which means "the king is dead"
became the expression checkmate. [D].

This expression is used during the game to indicate that one player's king is on the
verge of being captured.
A. [A] B. [B] C. [C] D. [D]

SESSION 3
PRACTICE TEST 1
23
Downloaded by Nhi An (annhinguyendang@gmail.com)
lOMoARcPSD|15007663

TRAN VAN HAU (ERIC TRAN) VSTEP – READING B1 UPDATED 2019


PASSAGE 1: QUESTIONS 1 -10
1____ Niagara Falls, one of the most famous North American natural wonders, has long been a
popular tourist destination. Tourists today flock to see the two falls that actually constitute Niagara
Falls: the 173-foot Horseshoe Falls on the Canadian side of the Niagara River in the Canadian
province of Ontario and the 182-foot high American Falls on the U.S. side of the river in the state of
New York. Approximately 85 percent of the water that goes over the falls actually goes over
Horseshoe Falls, with the rest going over American Falls.
7____ Most visitors come between April and October, and it is quite a popular activity to take a
steamer out onto the river and right up to the base of the falls for a close-up view. It is also possible
to get a spectacular view of the falls from the strategic locations along the Niagara River, such as
Prospect Point or Table Rock, or from one of the four observation towers which have heights up to
500 feet.
11____ Tourists have been visiting Niagara Falls in large numbers since the 1800s; annual visitation
now averages above 10 million visitors per year. Because of concern that all these tourists would
inadvertently destroy the natural beauty of this scenic wonder, the state of New York in 1885 created
Niagara Falls Park in order to protect the land surrounding American Falls. A year later Canada
created Queen Victoria Park on the Canadian side of the Niagara, around Horseshoe Falls. With the
area surrounding the falls under the jurisdiction of government agencies, appropriate steps could be
taken to preserve the pristine beauty of the area.
1. What is the major point that the author is making in this passage?
A. Niagara Falls can be viewed from either the American side or the Canadian side.
B. A trip to the United States isn't complete without a visit to Niagara Falls.
C. Niagara Falls has had an interesting history.
D. It has been necessary to protect Niagara Falls from the many tourists who go there.
2. The word "flock" in the first paragraph could best be replaced by ___.
A. come by plane B. come in large numbers
C. come out of boredom D. come without knowing what they will see
3. According to the passage, which of the following best describes Niagara Falls?
A. Niagara Falls consists of two rivers, one Canadian and the other American.
B. American Falls is considerably higher that Horseshoe Falls.
C. The Niagara River has two falls, one in Canada and one in the United States.
D. Although the Niagara River flows through the United States and Canada, the falls are only in the
United States.
4. A "steamer" in the second paragraph is probably
A. a bus B. a boat C. a walkway D. a park
5. The expression "right up" in the second paragraph could best be replaced by
A. turn to the right B. follow correct procedures C. travel upstream D. all the way up
6. The passage implies that tourists prefer to
A. visit Niagara Falls during warmer weather B. see the falls from a great distance
24
Downloaded by Nhi An (annhinguyendang@gmail.com)
lOMoARcPSD|15007663

TRAN VAN HAU (ERIC TRAN) VSTEP – READING B1 UPDATED 2019


C. take a ride over the falls D. come to Niagara Falls for a winter vacation
7. According to the passage, why was Niagara Park created?
A. To encourage tourists to visit Niagara Falls
B. To show off the natural beauty of Niagara Falls
C. To protect the area around Niagara Falls
D. To force Canada to open Queen Victoria Park
8. The word "jurisdiction" in the last paragraph is closest in meaning to
A. view B. assistance C. taxation D. control
9. The word "pristine" in the last paragraph is closest in meaning to
A. pure and natural B. highly developed C. well-regulated D. overused
10. The paragraph following the passage most probably discusses
A. additional ways to observe the falls
B. steps taken by government agencies to protect the falls
C. a detailed description of the division of the falls between the United States and Canada
D. further problems that are destroying the area around the falls
PASSAGE 2: QUESTIONS 11- 19
The first birds appeared during late Jurassic times. These birds are known from four very good
skeletons, two incomplete skeletons, and an isolated feather, all from the Solnhofen limestone of
Bavaria, Germany. This fine-grained rock, which is extensively quarried for lithographic stone, was
evidently deposited in a shallow coral lagoon of a tropical sea, and flying vertebrates occasionally
fell into the water and were buried by the fine limy mud, to be preserved with remarkable detail in
this way, the late Jurassic bird skeletons, which have been named Archaeopteryx, were fossilized.
And not only were the bones preserved in these skeletons, but so also were imprints of the feathers.
If the indications of feathers had not been preserved in association with Archaeopteryx, it is likely
that these fossils would have been classified among the dinosaurs, for they show numerous theropod
characteristics.
Archaeopteryx were animals about the size of a crow, with an archeosaurian type of skull, a long
neck, a compact body balanced on a pair of strong hind limbs, and a long tail. The forelimbs were
enlarged and obviously functioned as wings. Modern birds, who are the descendants of these early
birds, are highly organized animals, with a constant body temperature and a very high rate of
metabolism. In addition, they are remarkable for having evolved extraordinarily complex behavior
patterns such as those of nesting and song, and the habit among many species of making long
migrations from one continent to another and back each year.
Most birds also have very strong legs, which allow them to run or walk on the ground as well as to
fly in the air. Indeed, some of the water birds, such as ducks and geese, have the distinction of being
able to move around proficiently in the water, on land, and in the air, a range in natural locomotor
ability that has never been attained by any other vertebrate.
11: According to the author, all of-the following evidence relating to the first birds was found
EXCEPT

25
Downloaded by Nhi An (annhinguyendang@gmail.com)
lOMoARcPSD|15007663

TRAN VAN HAU (ERIC TRAN) VSTEP – READING B1 UPDATED 2019


(A) nesting materials (B) four skeletons in good condition
(C) two fragmented skeletons (D) a single feather.
12: The word "preserved" in line 6 is closest in meaning to
(A) confused with others (B) gradually weakened
(C) protected from destruction (D) lost permanently
13: It can be inferred from the passage that the Archaeopteryx were classified as birds on the basis
of…………………..
(A) imprints of bones (B) imprints of feathers
(C) the neck structure (D) skeletons
14: The word "they" in line 10 refers to………………
(A) indications (B) fossils (C) dinosaurs (D) characteristics
15: It can be inferred from the passage that theropods were
(A) dinosaurs (B) birds (C) Archaeopteryx (D) crows
16: The word "constant" in line 14 is closest in meaning to
(A) comfortable (B) combined (C) consistent (D) complementary
17: The word "attained" in line 22 is closest in meaning to
(A) required (B) achieved (C) observed (D) merited
18. Why does the author mention <a crow=in line 11?
a. to indicate the size of Archaeopteryx.
b. to specify the age of Archaeopteryx fossils
c. to explain the evolutionary history of Archaeopteryx
d. to demonstrate the superiority of the theropod to Archaeopteryx
19. The author mentions all of the following as examples of complex behavior patterns evolved by
birds EXCEPT:
A. Migrating B. nesting C. singing D. running
PASSAGE 3: Questions 20 – 30
In the American colonies there was little money. England did not supply the colonies with coins and
did not allow the colonies to make their own coins, except for the Massachusetts Bay Colony, which
received permission for a short period in 1652 to make several kinds of silver coins. England wanted
to keep money out of America as a means of controlling trade: America was forced to trade only
with England if it did not have the money to buy products from other countries. The result during
this pre-revolutionary period was that the colonists used various goods in place of money: beaver
pelts, Indian wampum, and tobacco leaves were all commonly used substitutes for money. The
colonists also made use of any foreign coins they could obtain. Dutch, Spanish, French, and English
coins were all in use in the American colonies.
During the Revolutionary War, funds were needed to finance the world, so each of the individual
states and the Continental Congress issued paper money. So much of this paper money was printed

26
Downloaded by Nhi An (annhinguyendang@gmail.com)
lOMoARcPSD|15007663

TRAN VAN HAU (ERIC TRAN) VSTEP – READING B1 UPDATED 2019


that by the end of the war, almost no one would accept it. As a result, trade in goods and the use of
foreign coins still flourished during this period.
By the time the Revolutionary War had been won by the American colonists, the monetary system
was in a state of total disarray. To remedy this situation, the new Constitution of the United States,
approved in 1789, allowed Congress to issue money. The individual states could no longer have their
own money supply. A few years later, the Coinage Act of 1792 made the dollar the official currency
of the United States and put the country on a bimetallic standard. In this bimetallic system, both gold
and silver were legal money, and the rate of exchange of silver to gold was fixed by the government
at sixteen to one.
20. The passage mainly discusses
A. the effect of the Revolution on American money.
B. American money from past to present.
C. the American monetary system of the seventeenth and eighteenth centuries.
D. the English monetary policies in colonial America.
21. The passage indicates that during the colonial period, money was
A. used extensively for trade. B. scarce.
C. supplied by England. D. coined by colonists.
22. The Massachusetts Bay Colony was allowed to make coins
A. for a short time during one year.
B. throughout the seventeenth century.
C. continuously from the inception of the colonies.
D. from 1652 until the Revolutionary War.
23: The expression <a means of= in paragraph 1 could be best replaced by.
A. a result of B. a method of C. a punishment for D. an example of
24. Which of the following is NOT mentioned in the passage as a substitute for money during the
colonial period?
A. Wampum B. Cotton C. Beaver furs D. Tobacco
25. The pronoun <it= in paragraph 2 refers to which of the following
A. The Continental Congress B. Trade in goods
C. The War D. Paper money
26: It is implied in the passage that at the end of the Revolutionary War, a paper dollar was worth
A. exactly one dollar B. just over one dollar
C. just under one dollar D. almost nothing
27. The word <remedy= in paragraph 3 is closest in meaning to
A. resolve B. medicate C. renew D. understand
28. How was the monetary system arranged in the Constitution?
A. The US officially went on a bimetallic monetary system.

27
Downloaded by Nhi An (annhinguyendang@gmail.com)
lOMoARcPSD|15007663

TRAN VAN HAU (ERIC TRAN) VSTEP – READING B1 UPDATED 2019


B. The dollar was made official currency of the US.
C. Only the US Congress could issue money.
D. Various state governments, including Massachusetts, could issue money.
29. According to the passage, which of the following is NOT true about the bimetallic monetary
system?
A. Either gold or silver could be used as official money.
B. It was established in 1792.
C. Gold could be exchanged for silver at the rate of sixteen to one.
D. The monetary system was based on two matters.
30. The word < fixed= in paragraph 3 is closed in meaning to
A. discovered B. set C. repaired D. valued
PASSAGE 4: Questions 31-40
There are many theories about the beginning of drama in ancient Greece. The one most widely
accepted today is based on the assumption that drama evolved from ritual. The argument for this view
goes as follows. In the beginning, human beings viewed the natural forces of the world, even the
seasonal changes, as unpredictable, and they (5) sought through various means, to control these
unknown and feared powers. Those measures which appeared to bring the desired results were then
retained and repeated until they hardened into fixed rituals. Eventually stories arose which explained
or veiled the mysteries of the rites. As time passed some rituals were abandoned, but the stories, later
called myths, persisted and provided material for art and drama.
(10) Those who believe that drama evolved out of ritual also argue that those rites contained the seed
of theater because music, dance, masks, and costumes were almost always used. Furthermore, a
suitable site had to be provided for performances, and when the entire community did not participate,
a clear division was usually made between the "acting area" and the "auditorium." In addition, there
were performers, (15) and since considerable importance was attached to avoiding mistakes in the
enactment of rites, religious leaders usually assumed that task. Wearing masks and costumes, they
often impersonated other people, animals, or supernatural beings, and mimed the desired effect-
success in hunt or battle, the coming rain, the revival of the Sun-as an actor might. Eventually such
dramatic representations were separated from religious (20)activities. Another theory traces the
theater's origin from the human interest in storytelling. According to this view, tales (about the hunt,
war, or other feats) are gradually elaborated at first through the use of impersonation, action, and
dialogue by a narrator and then through the assumption of each of the roles by a different person. A
closely
(25) related theory traces theater to those dances that are primarily rhythmical and gymnastic or that
are imitations of animal movements and sounds.
31. What does the passage mainly discuss?
(A) The origins of theater (B) The role of ritual in modern dance
(C) The importance of storytelling (D) The variety of early religious activities
32. The word "they" in line 4 refers to
(A) seasonal changes (B) natural forces (C) theories (D) human beings
28
Downloaded by Nhi An (annhinguyendang@gmail.com)
lOMoARcPSD|15007663

TRAN VAN HAU (ERIC TRAN) VSTEP – READING B1 UPDATED 2019


33. What aspect of drama does the author discuss in the first paragraph?
(A) The reason drama is often unpredictable
(B) The seasons in which dramas were performed
(C) The connection between myths and dramatic plots
(D) The importance of costumes in early drama
34. Which of the following is NOT mentioned as a common element of theater and ritual?
(A) Dance (B) Costumes (C) Music (D) Magic
35. The word "considerable" in line 15 is closest in meaning to
(A) thoughtful B) substantial (C) relational (D) ceremonial
36. The word "enactment" in line 15 is closest in meaning to
(A) establishment (B) performance (C) authorization (D) season
37. The word "they" in line 16 refers to
(A) mistakes (B) costumes (C) animals (D) performers
38. According to the passage, what is the main difference between ritual and drama?
(A) Ritual uses music whereas drama does not.
(B) Ritual is shorter than drama.
(C) Ritual requires fewer performers than drama.
(D) Ritual has a religious purpose and drama does not.
39. The passage supports which of the following statements?
(A) No one really knows how the theater began.
(B) Myths are no longer represented dramatically.
(C) Storytelling is an important part of dance.
(D) Dramatic activities require the use of costumes.
40. Where in the passage does the author discuss the separation of the stage and the audience?
(A) Lines 8-9 (B) Lines 12-14 (C) Lines 19-20 (D) Lines 22-24

SESSION 4
PRACTICE TEST 2
PASSAGE 1: QUESTIONS 1 -10
Mutualism is a type of symbiosis that occurs when two unlike organism live together in a state that
is mutually beneficial. It can exist between two animals, between two plants, or between a plant and
an animal. Mutualism is unlike the symbiotic state of commensalism in that commensalism is a one-
sided state in which a host gives a guest takes, while in mutualism both partners live on a give-and-
take basis.
In the African wilds, the zebra and the ostrich enjoy a symbiotic relationship that enhances the
ability of each of these large land animals to survive. Both serve as prey for the lion and neither has
the capability alone to withstand an attack from this fierce hunter. However, when the zebra and the
29
Downloaded by Nhi An (annhinguyendang@gmail.com)
lOMoARcPSD|15007663

TRAN VAN HAU (ERIC TRAN) VSTEP – READING B1 UPDATED 2019


ostrich collaborate in their defense by alerting each other to possible danger from an approaching
predator, the lion is rarely able to capture more than the oldest or feeblest of the herd.
The complementary physical strengths and weaknesses of the ostrich and the zebra allow them to
work in coordination to avoid succumbing to the lion. The ostrich possesses great speed and keen
eyesight, which enable it to spot large predatory animals long before they are able to position
themselves to attack. The zebra, with a running speed equal to that of the ostrich, has excellent
hearing and a good sense of smell but lacks the sharp eyesight of the ostrich. When ostriches and
zebras intermix for grazing, each animal benefits from the ability of the other to detect approaching
danger. If either animal senses danger, both animals are alerted and take off. With the running speed
that both of these animals possess, they are able to outrun any predator except the cheetah.
1. How is the information in the passage organized?
A. A concept is explained through an extended example.
B. a series of chronological events is presented.
C. Two examples are compared and contrasted.
D. Two opposing theories are explained.
2. The word <unlike= in line 1 is closest in meaning to ________
A. dissimilar B. unfriendly C. hated D. potential
3. Where in the passage does the author mention the one animal that is faster than both the ostrich and the
zebra?
A. Line 15-16 B. Line 17 -18 C. Line 8-10 D. Line 14-15
4. The word <beneficial= in line 2 is closest in meaning to _______
A. understood B. helpful C. meaningful D. distinctive
5. Which of the following is NOT stated in the passage?
A. The ostrich is unable to fly.
B. The ostrich is able to see better than the zebra.
C. The zebra hears and smells well.
D. The zebra is able to run faster than the ostrich.
6. The word <collaborate= in line 9 is closest in meaning to _______
A. make observation B. make a stand C. run and hide D. work together
7. The pronoun <it= in line 14 refers to _______
A. eyesight B. world C. speed D. ostrich
8. What is implied in the passage about the zebra and the ostrich?
A. Their relationship is not symbiotic B. They share a mutualistic relationship.
C. The lion is prey for both of them. D. They have a commensalist relationship.
9. What is <commensalisms= in line3?
A. A relationship in which both partners are hurt.
B. A relationship that is beneficial for both partners.
C. A specific kind of mutualistic relationship.
30
Downloaded by Nhi An (annhinguyendang@gmail.com)
lOMoARcPSD|15007663

TRAN VAN HAU (ERIC TRAN) VSTEP – READING B1 UPDATED 2019


D. A relationship that is beneficial to only one partner.
10. What is stated in the passage about the lion?
A. It is usually able to catch only weaker zebras and ostriches.
B. It does not hurt old or feeble zebras and ostriches.
C. It is easily able to capture zebras and ostriches.
D. It never hunts zebras and ostriches
PASSAGE 2: QUESTIONS 11 -20
Basic to any understanding of Canada in the 20 years after the Second World War is
the country's impressive population growth. For every three Canadians in 1945, there
were over five in 1966. In September 1966 Canada's population passed the 20 million
Line mark. Most of this surging growth came from natural increase. The depression of the
(5) 1930's and the war had held back marriages, and the catching-up process began after
1945. The baby boom continued through the decade of the 1950's, producing a
population increase of nearly fifteen percent in the five years from 1951 to 1956. This
rate of increase had been exceeded only once before in Canada's history, in the decade
before 1911, when the prairies were being settled. Undoubtedly, the good economic
(10) conditions of the 1950's supported a growth in the population, but the expansion also
derived from a trend toward earlier marriages and an increase in the average size of
families. In 1957 the Canadian birth rate stood at 28 per thousand, one of the highest in
the world.
After the peak year of 1957, the birth rate in Canada began to decline. It continued
(15) falling until in 1966 it stood at the lowest level in 25 years. Partly this decline reflected
the low level of births during the depression and the war, but it was also caused by
changes in Canadian society. Young people were staying at school longer; more
woman were working; young married couples were buying automobiles or houses
before starting families; rising living standards were cutting down the size of families.
(20) It appeared that Canada was once more falling in step with the trend toward smaller
families that had occurred all through the Western world since the time of the Industrial
Revolution.
Although the growth in Canada's population had slowed down by 1966 (the
increase in the first half of the 1960's was only nine percent), another large population
(25) wave was coming over the horizon. It would be composed of the children of the
children who were born during the period of the high birth rate prior to 1957.
11. What does the passage mainly discuss?
(A) Educational changes in Canadian society. (B) Canada during the Second World War
(C) Population trends in postwar Canada (D) Standards of living in Canada
31
Downloaded by Nhi An (annhinguyendang@gmail.com)
lOMoARcPSD|15007663

TRAN VAN HAU (ERIC TRAN) VSTEP – READING B1 UPDATED 2019


12. According to the passage, when did Canada's baby boom begin?
(A) In the decade after 191 (B) After 1945
(C) During the depression of the 1930's (D) In 1966
13. The word "five" in line 3 refers to
(A) Canadian (B) years (C) decades (D) marriages
14. The word "surging" in line 4 is closest in meaning to
(A) new (B) extra (C) accelerating (D) surprising
15. The author suggests that in Canada during the 1950's,……………………………….
(A) the urban population decreased rapidly (B) fewer people married
(C) economic conditions were poor (D) the birth rate was very high
16. The word "trend" in line 11 is closest in meaning to
(A) tendency (B) aim (C) growth (D) directive
17. The word "peak" in line 14 is closest in meaning to
(A) pointed (B) dismal (C) mountain (D) maximum
18. When was the birth rate in Canada at its lowest postwar level?
(A) 1966 (B) 1957 (C) 1956 (D) 1951
19. The author mentions all of the following as causes of declines in population growth after
1957 EXCEPT
(A) people being better educated (B) people getting married earlier
(C) better standards of living (D) couples buying houses
20. It can be inferred from the passage that before the Industrial Revolution
A) families were larger (B) population statistics were unreliable
(C) the population grew steadily (D) economic conditions were bad
21. The word "It" in line 25 refers to
(A) horizon (B) population wave (C) nine percent (D) first half
PASSAGE 3: QUESTIONS 22-30
Accustomed though we are to speaking of the films made before 1927 as "silent,"
the film has never been, in the full sense of the word, silent. From the very beginning,
music was regarded as an indispensable accompaniment ; when the Lumiere films were
Line shown at the first public film exhibition in the Unites States in February 1896, they
(5) were accompanied by piano improvisations on popular tunes. At first, the music played
bore no special relationship to the films ; an accompaniment of any kind was sufficient.
Within a very short time, however, the incongruity of playing lively music to a solemn
film became apparent, and film pianists began to take some care in matching their
pieces to the mood of the film.
(10) As movie theaters grew in number and importance, a violinist, and perhaps a cellist,
32
Downloaded by Nhi An (annhinguyendang@gmail.com)
lOMoARcPSD|15007663

TRAN VAN HAU (ERIC TRAN) VSTEP – READING B1 UPDATED 2019


would be added to the pianist in certain cases, and in the larger movie theaters small
orchestras were formed. For a number of years the selection of music for each film
program rested entirely in the hands of the conductor or leader of the orchestra, and
very often the principal qualification for holding such a position was not skill or taste
(15) so much as the ownership of a large personal library of musical pieces. Since the
conductor seldom saw the films until the night before they were to be shown (if,
indeed, the conductor was lucky enough to see them then), the musical arrangement
was normally improvised in the greatest hurry.
To help meet this difficulty, film distributing companies started the practice of
(20) publishing suggestions for musical accompaniments. In 1909, for example, the Edison
Company began issuing with their films such indications of mood as "pleasant," "sad,"
"lively." The suggestions became more explicit, and so emerged the musical cue sheet
containing indications of mood, the titles of suitable pieces of music, and precise
directions to show where one piece led into the next.
(25) Certain films had music especially composed for them. The most famous of these
early special scores was that composed and arranged for D.W. Griffith's film Birth of
a Nation, which was released in 1915.
22. The passage mainly discusses music that was
(A) performed before the showing of a film (B) played during silent films
(C) specifically composed for certain movie theaters (D) recorded during film exhibitions
23. What can be inferred from the passage about the majority of films made after 1927?
(A) They were truly "silent."
(B) They were accompanied by symphonic orchestras.
(C) They incorporated the sound of the actors' voices.
(D) They corresponded to specific musical compositions.
24. The word "solemn" in line 7 is closest in meaning to
(A) simple (B) serious (C) short (D) silent
25. It can be inferred that orchestra conductors who worked in movie theaters needed to
(A) be able to play many instruments (B) have pleasant voices
(C) be familiar with a wide variety of music. (D) be able to compose original music
26. The word "them" in line 17 refers to
(A) years (B) hands (C) pieces (D) films
27. According to the passage, what kind of business was the Edison Company?
(A) It produced electricity. (B) It distributed films.
(C) It published musical arrangements. (D) It made musical instruments.
28. It may be inferred from the passage that the first musical cue sheets appeared around

33
Downloaded by Nhi An (annhinguyendang@gmail.com)
lOMoARcPSD|15007663

TRAN VAN HAU (ERIC TRAN) VSTEP – READING B1 UPDATED 2019


(A) 1896 (B) 1909 (C) 1915 (D) 1927
29. The word " composed" in line 26 is closest in meaning to
(A) selected (B) combined (C) played (D) created
30. The word "scores" in line 26 is closest in meaning to
(A) totals (B) successes (C) musical compositions (D) groups of musicians
PASSAGE 4: QUESTIONS 31-40
Glass is a remarkable substance made from the simplest raw materials. It can be colored or colorless,
monochrome or polychrome, transparent, translucent, or opaque. It is lightweight impermeable to
liquids, readily cleaned and reused, durable yet fragile, and often very beautiful Glass can be
decorated in multiple ways and its optical properties are exceptional. In all its myriad forms - as table
ware, containers, in architecture and design – glass represents a major achievement in the history of
technological developments.
Since the Bronze Age about 3,000 B.C., glass has been used for making various kinds of objects. It
was first made from a mixture of silica, line and an alkali such as soda or potash, and these remained
the basic ingredients of glass until the development of lead glass in the seventeenth century. When
heated, the mixture becomes soft and malleable and can be formed by various techniques into a vast
array of shapes and sizes. The homogeneous mass thus formed by melting then cools to create glass,
but in contrast to most materials formed in this way (metals, for instance), glass lacks the crystalline
structure normally associated with solids, and instead retains the random molecular structure of a
liquid. In effect, as molten glass cools, it progressively stiffens until rigid, but does so without setting
up a network of interlocking crystals customarily associated with that process. This is why glass
shatters so easily when dealt a blow. Why glass deteriorates over time, especially when exposed to
moisture, and why glassware must be slowly reheated and uniformly cooled after manufacture to
release internal stresses induced by uneven cooling.
Another unusual feature of glass is the manner in which its viscosity changes as it turns from a cold
substance into a hot, ductile liquid. Unlike metals that flow or "freeze" at specific temperatures glass
progressively softens as the temperature rises, going through varying stages of malleability until it
flows like a thick syrup. Each stage of malleability allows the glass to be manipulated into various
forms, by different techniques, and if suddenly cooled the object retains the shape achieved at that
point. Glass is thus amenable to a greater number of heat-forming techniques than most other
materials.
31: Why does the author list the characteristics of glass in paragraph 1?
A. To demonstrate how glass evolved B. To show the versatility of glass
C. To explain glassmaking technology D. To explain the purpose of each component of glass
32: The word <durable= in paragraph 1 is closest in meaning to .
A. lasting B. delicate C. heavy D. plain
33: What does the author imply about the raw materials used to make glass?
A. They were the same for centuries. B. They are liquid.
C. They are transparent. D. They are very heavy.

34
Downloaded by Nhi An (annhinguyendang@gmail.com)
lOMoARcPSD|15007663

TRAN VAN HAU (ERIC TRAN) VSTEP – READING B1 UPDATED 2019


34: According to the passage, how is glass that has cooled and become rigid different from most other
rigid substances?
A. It has an interlocking crystal network. B. It has an unusually low melting temperature.
C. It has varying physical properties. D. It has a random molecular structure.
35: The word <customarily= in paragraph 2 could best be replaced by < =.
A. naturally B. necessarily C. usually D. certainly
36: The words <exposed to= in paragraph 2 most likely mean .
A. hardened by B. chilled with C. subjected to D. deprived of
37: What must be done to release the internal stresses that build up in glass products during
manufacture?
A. The glass must be reheated and evenly cooled.
B. The glass must be cooled quickly.
C. The glass must be kept moist until cooled.
D. The glass must be shaped to its desired form immediately
38: The word <induced= in paragraph 2 is closest in meaning to .
A. joined B. missed C. caused D. lost
39: The word <it= in paragraph 3 refers to .
A. feature B. glass C. manner D. viscosity
40: According to the passage, why can glass be more easily shaped into specific forms than can metals?
A. It resists breaking when heated
B. It has better optical properties.
C. It retains heat while its viscosity changes.
D. It gradually becomes softer as its temperature rises.

SESSION 5
PRACTICE TEST 3
PASSAGE 1: QUESTIONS 1 -10
A rather surprising geographical feature of Antarctica is that a huge freshwater lake ,one of the
world?s largest and deepest , lies hidden there under four kilometers of ice .Now known as Lake
Vostok, this huge body of water is located under the ice block that comprises Antarctica . The lake is
able to exist in its unfrozen state beneath this block of ice because its waters are warmed by
geothermal heat from the earth?s core .The thick glacier above Lake Vostok actually insulates it from
the frigid temperatures on the surface(Earth).

The lake was first discovered in the 1970s while a research team was conducting an aerial survey of
the area .Radio waves from the survey equipment penetrated the ice and revealed a body of water of
indeterminate size. It was not until much more recently that data collected by satellite made scientists
aware of the tremendous size of the lake ; the satellite the lowest ever recorded on the –borne radar
35
Downloaded by Nhi An (annhinguyendang@gmail.com)
lOMoARcPSD|15007663

TRAN VAN HAU (ERIC TRAN) VSTEP – READING B1 UPDATED 2019


detected an extremely flat region where the ice remains level because it is floating on the water of the
lake .

The discovery of such a huge freshwater lake trapped under Antarctica is of interest to the scientific
community because of the potential that the lake contains ancient microbes that have survived for
thousands upon thousands of years, unaffected by factors such as nuclear fallout and elevated
ultraviolet light that have affected organism in more exposed areas .The downside of the discovery,
however, lies in the difficulty of conducting research on the lake in such a harsh climate and in the
problems associated with obtaining uncontaminated samples from the lake without actually exposing
the lake to contamination. Scientists are looking for possible way to accomplish this.
1: The word <hidden= in paragraph 1 is closest in meaning to____

A. unexploitable B. untouched C. undiscovered D. undrinkable

2: What is true of Lake Vostok ?

A. It is beneath a thick slab of ice B. It is heated by the sun

C. It is a saltwater lake. D. It is completely frozen.

3: Which of the following is closest in meaning to <frigid= in paragraph 1?

A. easily broken B. quite harsh C. lukewarm D. extremely cold

4: All of the following are true about the 1970 survey of Antarctica EXCEPT that it ____

A. could not determine the lake?s exact size B. made use of radio waves

C. was conducted by air D. was controlled by a satellite

5: It can be inferred from the passage that the ice would not be flat if ______ .

A. there were no lake underneath B. Antarctica were not so cold

C. radio waves were not used D. the lake were not so big

6: The word <microbes= in paragraph 3 could be best replaced by which of the following ?

A. Rays of light B. Pieces of dust C. Tiny bubbles D. Tiny organisms

7: Lake Vostok is potentially important to scientists because it ________ .

A. may have elevated levels of ultraviolet light B. has already been contaminated

C. can be studied using radio waves D. may contain uncontaminated microbes

8: The word <downside= in paragraph 3 is closest in meaning to_______ .

A. buried section B. bottom level C. underside D. negative aspect

9: The last paragraph suggests that scientists should be aware of _______ .

A. further discoveries on the surface of Antarctica


36
Downloaded by Nhi An (annhinguyendang@gmail.com)
lOMoARcPSD|15007663

TRAN VAN HAU (ERIC TRAN) VSTEP – READING B1 UPDATED 2019


B. problems with satellite – borne radar equipment

C. the harsh climate of Antarctica

D. ways to study Lake Vostok without contaminating it

10: The purpose of the passage is to _______ .

A. present an unexpected aspect of Antarctica?s geography

B. provided satellite data concerning Antarctica

C. explain how Lake Vostok was discovered

D. discuss future plans for Lake Vostok

PASSAGE 2: QUESTIONS 11 -20


Crows are probably the most frequently met and easily identifiable members of the native fauna of the
United States. The great number of tales, legends, and myths about these birds indicates that people
have been exceptionally interested in them for a long time. On the other hand, when it comes
to substantive -- particularly behavioral -- information, crows are less well known than many
comparably common species and, for that matter, not a few quite uncommon ones: the endangered
California condor, to cite one obvious example. There are practical reasons for this.

Crows are notoriously poor and aggravating subjects for field research. Keen observers and quick
learners, they are astute about the intentions of other creatures, including researchers, and adapt
at avoiding them. Because they are so numerous, active, and monochromatic, it is difficult to
distinguish one crow from another. Bands, radio transmitters, or other identifying devices can be
attached to them, but this of course requires catching live crows, who are among the wariest and most
untrappable of birds.
Technical difficulties aside, crow research is daunting because the ways of these birds are so complex
and various. As preeminent generalists, members of this species ingeniously exploit a great range
of habitats and resources, and they can quickly adjust to changes in their circumstances. Being so
educable, individual birds have markedly different interests and inclinations, strategies
and scams. For example, one pet crow learned how to let a dog out of its kennel by pulling the pin on
the door. When the dog escaped, the bird went into the kennel and ate its food.
11. What is the main topic of the passage?
(A) The ways in which crows differ from other common birds
(B) The myths and legends about crows
(C) The characteristics that make crows difficult to study
(D) The existing methods for investigating crow behavior
12. The word "comparable" in line 5 is closest in meaning to
(A) interestingly (B) similar (C) otherwise (D) sometimes
13. In line 6, the author mention the endangered California condor as an example of a species that
is
37
Downloaded by Nhi An (annhinguyendang@gmail.com)
lOMoARcPSD|15007663

TRAN VAN HAU (ERIC TRAN) VSTEP – READING B1 UPDATED 2019


(A) smaller than the crow (B) easily identifiable (C) featured in legends (D) very rare
14. The word "them" in line 10 refers to
(A) crow (B) subjects (C) intentions (D) researchers
15.According to the second paragraph, crows are poor subjects for field research for all of the
following reasons EXCEPT
(A) They can successfully avoid observers.
(B) They are hard to distinguish from one another
(C) They can be quite aggressive.
(D) They are difficult to catch.
16. In the second paragraph, the author implies that using radio transmitters would allow a researcher
who studies crow to
(A) identify individual crows (B) follow flocks of crows over long distances
(C) record the times when crows are most active (D) help crows that become sick or injured
17. According to the third paragraph, which of the following is true about crows?
(A) They seldom live in any one place for very long.
(B) They thrive in a wide variety of environments.
(C) They have marked preferences for certain kinds of foods.
(D) They use up the resources in one area before moving to another.
18. In line 19, the word "inclinations" is closest in meaning to
(A) tricks (B) opportunities (C) preferences (D) experiences
19. In lines 19-21, the author mentions a pet crow to illustrate which of the following?
(A) The clever ways that crows solve problems
(B) The differences between pet crows and wild crows
(C) The ease with which crows can be tamed
(D) The affection that crows show to other creatures
20. Which of the following statements is supported by the passage?
(A) Crows have relatively long lives. (B) Crows have keen vision
(C) Crows are usually solitary (D) Crows are very intelligent.
PASSAGE 3: QUESTIONS 21 - 30
Whereas literature in the first half of the eighteenth century in America had been largely religious
and moral in tone, by the latter half of the century the revolutionary fervor that was coming to life
in the colonies began to be reflected in the literature of the time, which in turn served to further
influence the population. Although not all writers of this period supported the Revolution, the two
best-known and most influential writers, Ben Franklin and Thomas Paine, were both strongly
supportive of that cause.

38
Downloaded by Nhi An (annhinguyendang@gmail.com)
lOMoARcPSD|15007663

TRAN VAN HAU (ERIC TRAN) VSTEP – READING B1 UPDATED 2019


Ben Franklin first attained popular success through his writings in his brother's newspaper, the
New England Current. In these articles he used a simple style of language and common sense
argumentation to defend the point of view of the farmer and the Leather Apron man. He continued
with the same common sense practicality and appeal to the common man with his work on Poor
Richard's Almanac from 1733 until 1758. Firmly established in his popular acceptance by the
people, Franklin wrote a variety of extremely effective articles and pamphlets about the colonists'
revolutionary cause against England.

Thomas Paine was an Englishman working as a magazine editor in Philadelphia at the time of the
Revolution. His pamphlet Common Sense, which appeared in 1776, was a force in encouraging
the colonists to declare their independence from England. Then throughout the long and desperate
war years he published a series of Crisis papers (from 1776 until 1783) to encourage the colonists
to continue on with the struggle. The effectiveness of his writing was probably due to his
emotional yet oversimplified depiction of the cause of the colonists against England as a classic
struggle of good and evil.
22. The paragraph preceding this passage most likely discusses
(A) how literature influences the population
(B) religious and moral literature
(C) literature supporting the cause of the American Revolution
(D) what made Thomas Paine's literature successful
23. The word "fervor" in line 2 is closest in meaning to
(A) war (B) anxiety (C) spirit (D) action
24. The word "time" in line 3 could best be replaced by
(A) hour (B) period (C) appointment (D) duration
25. It is implied in the passage that
(A) some writers in the American colonies supported England during the Revolution
(B) Franklin and Paine were the only writers to influence the Revolution
(C) because Thomas Paine was an Englishman, he supported England against the colonies
(D) authors who supported England did not remain in the colonies during the Revolution
26. The pronoun "he" in line 7 refers to
(A) Thomas Paine (B) Ben Franklin (C) Ben Franklin's brother (D) Poor Richard
27. The expression "point of view" in line 8 could best be replaced by
(A) perspective (B) sight (C) circumstance (D) trait
28. According to the passage, the tone of Poor Richard's Almanac is
(A) pragmatic (B) erudite (C) theoretical (D) scholarly
29. The word "desperate" in line 14 could best be replaced by
(A) unending (B) hopeless (C) strategic (D) combative
30. The purpose of the passage is to

39
Downloaded by Nhi An (annhinguyendang@gmail.com)
lOMoARcPSD|15007663

TRAN VAN HAU (ERIC TRAN) VSTEP – READING B1 UPDATED 2019


(A) discuss American literature in the first half of the eighteenth century
(B) give biographical data on two American writers
(C) explain which authors supported the Revolution
(D) describe the literary influence during revolutionary America
PASSAGE 4: QUESTIONS 31-40
Sharks have gained an unfair reputation for being fierce predators of large sea animals.
Humanity's unfounded fear and hatred of these ancient creatures is leading to a worldwide
slaughter that may result in the extinction of many coastal shark species. The shark is the
victim of a warped attitude of wildlife protection; we strive only to protect the beautiful, non-
threatening parts of our environment. And, in our efforts to restore only non-threatening parts
of our earth, we ignore other important parts.

A perfect illustration of this attitude is the contrasting attitude toward another large sea
animal, the dolphin. During the 1980s, environmentalists in the United States protested the
use of driftnets for tuna fishing in the Pacific Ocean since these nets also caught dolphins. The
environmentalists generated enough political and economic pressure to prevent tuna
companies from buying tuna that had been caught in driftnets. In contrast to this effort on
behalf of the dolphins, these same environmentalists have done very little to help save the
Pacific Ocean sharks whose population has decreased nearly to the point of extinction. Sharks
are among the oldest creatures on earth, having survived in the seas for more than 350 million
years. They are extremely efficient animals, feeding on wounded or dying animals, thus
performing an important role in nature of weeding out the weaker animals in a species. Just
the fact that species such as the Great White Shark have managed to live in the oceans for so
many millions of years is enough proof of their efficiency and adaptability to changing
environments. It is time for us humans, who may not survive another 1,000 years at the rate we
are damaging the planet, to cast away our fears and begin considering the protection of sharks
as an important part of a program for protection of all our natural environment.
31. With which of the following topics is this passage primarily concerned?
(A) Sharks are efficient creatures with bad reputations.
(B) Sharks are some of the oldest creatures on earth.
(C) Sharks illustrate a problem in wildlife protection.
(D) The campaign to save dolphins was not extended to save sharks.
32. Which of the following is most similar to the meaning of the word "warped" in line 4?
(A) distorted (B) wasteful (C) extravagant (D) wanton
33. In line 8, the word "protested" is closest in meaning to which of the following?
(A) prescribed (B) objected to (C) protected (D) reflected on
34. In line 10, the word "generated" could be best replaced by
(A) consumed (B) absorbed (C) designated (D) produced
35. How did environmentalists manage to protect dolphins?

40
Downloaded by Nhi An (annhinguyendang@gmail.com)
lOMoARcPSD|15007663

TRAN VAN HAU (ERIC TRAN) VSTEP – READING B1 UPDATED 2019


(A) They prevented fishermen from selling them for meat.
(B) They pressured fishermen into protecting dolphins by law.
(C) They brought political pressure against tuna companies.
(D) They created sanctuaries where dolphin fishing was not allowed.
36. About how long have sharks lived on the planet?
(A) 25 million years (B) 150 million years
(C) 350 million years (D)500 million years
37. The author uses the phrase "weeding out" in line 16 to mean
(A) strengthening something that is weak (B) feeding something that is hungry
(C) encouraging something that is efficient (D) getting rid of something that is unwanted
38. In line 18, the phrase "managed to live" is used to infer that
(A) surviving was difficult (B) migration was common
(C) procreation was expanding (D) roaming was necessary
39. The word "proof" in line 18 could be best replaced by which of the following?
(A) characteristic (B) evidence (C) praise (D) customary
40. What is the author's tone in this passage?
(A) explanatory (B) accusatory (C) gentle (D) proud
SESSION 6
PRACTICE TEST 4
PASSAGE 1:
Esperanto is what is called a planned, or artificial, language. It was created more than a century ago by
Polish eye doctor Ludwik Lazar Zamenhof. Zamenhof believed that a common language would help to
alleviate some of the misunderstandings among cultures.

In Zamenhof?s first attempt at a universal language, he tried to create a language that was as
uncomplicated as possible. This first language included words such as ab, ac, ba, eb, be, and ce. This
did not result in a workable language in that these monosyllabic words, though short, were not easy to
understand or to retain.

Next, Zamenhof tried a different way of constructing a simplified language. He made the words in his
language sound like words that people already knew, but he simplified the grammar tremendously. One
example of how he simplified the language can be seen in the suffixes: all nouns in this language end in
o, as in the noun amiko, which means <friend=, and all adjectives end in -a, as in the adjective bela,
which means <pretty=. Another example of the simplified language can be seen in the prefix mal-,
which makes a word opposite in meaning; the word malamiko therefore means <enemy=, and the word
malbela therefore means <ugly= in Zamenhof?s language.

In 1887, Zamenhof wrote a description of this language and published it. He used a pen name, Dr.
Esperanto, when signing the book. He selected the name Esperanto because this word means <a person
who hopes= in his language. Esperanto clubs began popping up throughout Europe, and by 1950,
Esperanto had spread from Europe to America and Asia.
41
Downloaded by Nhi An (annhinguyendang@gmail.com)
lOMoARcPSD|15007663

TRAN VAN HAU (ERIC TRAN) VSTEP – READING B1 UPDATED 2019

In 1905, the First World Congress of Esperanto took place in France, with approximately700 attendees
from 20 different countries. Congresses were held annually for nine years, and 4,000 attendees were
registered for the Tenth World Esperanto Congress scheduled for 1914, when World War I erupted and
forced its cancellation.

Esperanto has had its ups and downs in the period since World War I. Today, years after it was
introduced, it is estimated that perhaps a quarter of a million people are fluent in it. This may seem like
a large number, but it is really quite small when compared with the billion English speakers and billion
Mandarin Chinese speakers in today?s world. Current advocates would like to see its use grow
considerably and are taking steps to try to make this happen.
1.The topic of this passage is
A. one man?s efforts to create a universal language
B. how language can be improved
C. using language to communicate internationally
D. a language developed in the last few years
2. According to the passage, Zamenhof wanted to create a universal language
A. to build a name for himself
B. to provide a more complex language
C. to resolve cultural differences
D. to create one world culture
3: It can be inferred from the passage that the Esperanto word malespera means
A. hopeless B. hope C. hopelessness D. hopeful
4. The expression <popping up= in line 17 could best be replaced by
A. shouting B. opening C. hiding D. leaping
5. It can be inferred from the passage that the Third World Congress of Esperanto took place
A. in 1905 B. in 1909 C. in 1907 D. in 1913
6. According to the passage, what happened to the Tenth World Esperanto Congress?
A. It had attendees from20 countries
B. It never took place
C. It had 4,000 attendees
D. It was scheduled for 1915
7: The expression <ups and downs= in line 23 is closest in meaning to
A. tops and bottoms B. floors and ceilings
C. takeoffs and landings D. highs and lows
8: Which paragraph describes the predecessor to Esperanto?
A. The first paragraph B. The second paragraph
C. The third paragraph D. The fourth paragraph
9: The passage would most likely be assigned reading in a course on
A. European history B. English grammar C. world government D. applied linguistics
10. The paragraph following the passage most likely discusses

42
Downloaded by Nhi An (annhinguyendang@gmail.com)
lOMoARcPSD|15007663

TRAN VAN HAU (ERIC TRAN) VSTEP – READING B1 UPDATED 2019


A. how current supporters of Esperanto are encouraging its growth
B. another of Zamenhof?s accomplishments
C. the disadvantages of using an artificial language
D. attempts to reconvene the World Congress of Esperanto in the 1920s
PASSAGE 2: QUESTIONS 11 -20
Just two months after the flight of Apollo 10, the Apollo 11 astronauts made their historic landing on
the surface of the Moon. This momentous trip for humanity also provided scientists with an
abundance of material for study; from rock and soil samples brought back from the Moon, scientists
have been able to determine much about the composition of the Moon (as well as to draw) inferences
about the development of the Moon from its composition.
The Moon soil that came back on Apollo 11 contains small bits of rock and glass which were
probably ground from larger rocks when meteors impacted with the surface of the Moon. The bits of
glass are spherical in shape and constitute approximately half of the Moon soil. Scientists found no
trace of animal or plant life in this soil.
In addition to the Moon soil, astronauts gathered two basic types of rocks from the surface of the
Moon: Basalt and breccia. Basalt is cooled and hardened volcanic lava common to the Earth. Since
basalt is formed under extremely high temperatures, the presence of this type of rock is an indication
that the temperature of the Moon was once extremely hot. Breccia, the other kind of rock brought
back by the astronauts, was formed during the impact of falling objects on the surface of the Moon.
This second type of rock consists of small pieces of rock compressed together by the force of
impact. Gases such as hydrogen and helium were found in some of the rocks, and scientists believe
that these gases were carried to the Moon by the solar wind, the streams of gases that are constantly
emitted by the atmosphere.
11: The paragraph preceding the passage most likely discusses
A. astronaut training. B. The inception of the Apollo space program.
C. a different space trip. D. previous Moon landings.
12: What is the subject of this passage?
A. The Apollo astronauts. B. Soil on the Moon.
C. What the Moon is made of. D. Basalt and breccia.
13: According to the passage, what does Moon soil consist of?
A. Hydrogen and helium. B. Large chunks of volcanic lava.
C. Tiny pieces of stones and glass. D. Streams of gases.
14: Which of the following was NOT brought back to the Earth by the astronauts?
A. Basalt B. Soil C. Breccia D. Plant life
15: According to the passage, breccia was formed…………………………….
A. when objects struck the Moon.
B. from volcanic lava.
C. When streams of gases hit the surface of the Moon.

43
Downloaded by Nhi An (annhinguyendang@gmail.com)
lOMoARcPSD|15007663

TRAN VAN HAU (ERIC TRAN) VSTEP – READING B1 UPDATED 2019


D. from the interaction of helium and hydrogen.
16: It is implied in the passage that scientists believe that the gases found in the Moon rocks.
A. were not originally from the Moon. B. were created inside the rocks.
C. traveled from the Moon to the Sun . D. caused the Moon's temperature to rise.
17: The word 'emitted' in the last paragraph is closest in meaning to
A. set off B. vaporized C. sent out D. separated
18: The author's purpose in this passage is to………………………….
A. describe some rock and soil samples.
B. explain some of the things learned from space flights.
C. propose a new theory about the creation of the Moon.
D. demonstrate the difference between basalt and breccia.
19: It can be interred from the passage that………………………………
A. the only items of importance that astronauts brought back from the Moon were rock and soil
samples.
B. scientists learned relatively little from the Moon rock and soil samples.
C. scientists do not believe that it is necessary to return to the Moon.
D. rock and soil samples were only some of a myriad of significant items from the Moon.
20: Which of the following would serve as the best title of the passage:
A. Apollo 11 B. things from the space flights
C. rock on the Moon D. Astronauts
READING 3: QUESTIONS 21 - 30
Rent control is the system whereby the local government tells building owners how much they
can charge their tenants in rent. In the United States, rent controls date back to at least World War
II.

In 1943 the federal government imposed rent controls to help solve the problem of housing
shortages during wartime. The federal program ended after the war, but in some locations,
including New York City, controls continued. Under New York's controls, a landlord generally
cannot raise rents on apartments as long as the tenants continue to renew their leases. In places
such as Santa Monica, California, rent controls are more recent. They were spurred by the
inflation of the 1970's, which, combined with California's rapid population growth, pushed
housing prices, as well as rents, to record levels. In 1979 Santa Monica's municipal government
ordered landlords to roll back their rents to the levels charged in 1978. Future rents could only go
up by two-thirds as much as any increase in the overall price level.

In any housing market, rental prices perform three functions: (1) promoting the efficient
maintenance of existing housing and stimulating the construction of new housing, (2) allocating
existing scarce housing among competing claimants, and (3) rationing use of existing housing by
potential renters.

44
Downloaded by Nhi An (annhinguyendang@gmail.com)
lOMoARcPSD|15007663

TRAN VAN HAU (ERIC TRAN) VSTEP – READING B1 UPDATED 2019


One result of rent control is a decrease in the construction of new rental units. Rent controls have
artificially depressed the most important long-term determinant of profitability - rents. Consider
some examples. In a recent year in Dallas, Texas, with a 16 percent rental vacancy rate but no rent
control laws, 11,000 new housing units were built. In the same year, in San Francisco, California,
only 2,000 units were built. The major difference? San Francisco has only a 1.6 percent vacancy
rate but stringent rent control laws. In New York City, except for government-subsidized
construction, the only rental units being built are luxury units, which are exempt from controls. In
Santa Monica, California, new apartments are not being constructed. New office rental space and
commercial developments are, however. They are exempt from rent controls.

20. What does the passage mainly discuss?


(A) The construction of apartments in the United States.
(B) Causes and effects of rent control
(C) The fluctuations of rental prices
(D) The shortage of affordable housing in the United States.
21. The word "They" in line 7 refers to
(A) the tenant (B) their leases (C) places (D) rent controls.
22. Which of the following was NOT a reason for the introduction of rent controls in Santa Monica,
California?
(A) Rapid population growth (B) Inflation
(C) Economic conditions during wartime (D) Record-high housing prices
23. The phrase "roll back" in lines 9 is closest in meaning to
(A) credit (B) measure (C) vary (D) reduce
24. The word "stimulating" in line 12 is closest in meaning to
(A) experimenting with (B) identifying (C) estimating (D) encouraging
25. It can be inferred that the purpose of rent control is to
(A) protect tenants (B) promote construction
(C) increase vacancy rates (D) decrease sales of rental units
26. The word "depressed" in line 15 is closest in meaning to
(A) saddened (B) created (C) lowered (D) defeated
27. The information in the last paragraph supports which of the following statements?
(A) San Francisco has eliminated its rent control laws.
(B) Rent control leads to a reduction in the construction of housing units
(C) Luxury apartments are rarely built when there is rent control
(D) There is a growing need for government-subsidized housing.
28. According to the passage, which of the following cities does NOT currently have rent controls?
(A) Santa Monica (B) Dallas
(C) San Francisco (D) New York City
29. The word "stringent" in line 18 is closest in meaning to
(A) straightforward B) strict (C) expanded (D) efficient
30. According to the passage, which of the following is NOT exempt from rent control?
(A) Luxury apartments
45
Downloaded by Nhi An (annhinguyendang@gmail.com)
lOMoARcPSD|15007663

TRAN VAN HAU (ERIC TRAN) VSTEP – READING B1 UPDATED 2019


(B) Commercial development
(C) Moderately priced apartments
(D) Office space.
PARAGRAPH 4: QUESTIONS 31 -40
A useful definition of an air pollutant is a compound added directly or indirectly by humans to the
atmosphere in such quantities as to affect humans, animals, vegetation, or materials adversely. Air
pollution requires a very flexible definition that permits continuous change. When the first air
pollution laws were established in England in the fourteenth century, air pollutants were limited to
compounds that could be seen or smelled-a far cry from the extensive list of harmful substances
known today. As technology has developed and knowledge of the health aspects of various chemicals
has increased, the list of air pollutants has lengthened. In the future, even water vapor might be
considered an air pollutant under certain conditions.

Many of the more important air pollutants, such as sulfur oxides, carbon monoxide, and nitrogen
oxides, are found in nature. As the Earth developed, the concentrations of these pollutants were
altered by various chemical reactions; they became components in biogeochemical cycle. These serve
as an air purification scheme by allowing the compounds to move from the air to the water or soil on a
global basis, nature's output of these compounds dwarfs that resulting from human activities.
However, human production usually occurs in a localized area, such as a city.

In this localized regions, human output may be dominant and may temporarily overload the natural
purification scheme of the cycle. The result is an increased concentration of noxious chemicals in the
air. The concentrations at which the adverse effects appear will be greater than the concentrations that
the pollutants would have in the absence of human activities. The actual concentration need not be
large for a substance to be a pollutant; in fact the numerical value tells us little until we know how
much of an increase this represents over the concentration that would occur naturally in the area. For
example, sulfur dioxide has detectable health effects at 0.08 parts per million (ppm), which is about
400 times its natural level. Carbon monoxide, however, has a natural level of 0.1 ppm and is not
usually a pollutant until its level reaches about 15 ppm.
31. What does the passage mainly discuss?
A. The economic impact of air pollution B. What constitutes an air pollutant
C. How much harm air pollutants can cause D. The effects of compounds added to the atmosphere
32. The word "adversely" is closest in meaning to
A. negatively B. quickly C. admittedly D. considerably
33. It can be inferred from the first paragraph that
A. water vapor is an air pollutant in localized areas
B. most air pollutants today can be seen or smelled
C. the definition of air pollution will continue to change
D. a substance becomes an air pollutant only in cities
34. The word "altered" is closest in meaning to
46
Downloaded by Nhi An (annhinguyendang@gmail.com)
lOMoARcPSD|15007663

TRAN VAN HAU (ERIC TRAN) VSTEP – READING B1 UPDATED 2019


A. eliminated B. caused C. slowed D. changed
35. Natural pollutants can play an important role in controlling air pollution for which of the following
reasons?
A. They function as part of a purification process.
B. They occur in greater quantities than other pollutants.
C. They are less harmful to living beings than are other pollutants.
D. They have existed since the Earth developed.
36. According to the passage, which of the following is true about human-generated air pollution in
localized regions?
A. It can be dwarfed by nature's output of pollutants in the localized region.
B. It can overwhelm the natural system that removes pollutants.
C. It will damage areas outside of the localized regions.
D. It will react harmfully with naturally occurring pollutants.
37. The word "noxious' is closest in meaning to
A. harmful B. noticeable C. extensive D. weak
38. According to the passage, the numerical valued of the concentration level of a substance is only
useful if
A. the other substances in the area are known
B. it is in a localized area
C. the naturally occurring level is also known
D. it can be calculated quickly
39. The word "detectable" is closest in meaning to
A. beneficial B. special C. measurable D. separable
40. Which of the following is best supported by the passage?
A. To effectively control pollution local government should regularly review their air pollution laws.
B. One of the most important steps in preserving natural lands is to better enforce air pollution laws.
C. Scientists should be consulted in order to establish uniform limits for all air pollutants.
D. Human activities have been effective in reducing air pollution.
SESSION 7
PRACTICE TEST 5
PART 1: QUESTIONS 1 -10
The brain of the average human weighs approximately 14 kilograms and consists of three main parts-
the cerebrum, the cerebellum, and the brain stem. It is the control center of the body. It receives
information from the senses, processes the information, and rapidly sends out responses; it also stores
the information that is the source of human thoughts and feelings.

The cerebrum is by far the largest of the three parts, taking up 85% of the brain by weight. The outside
layer of the cerebrum, the cerebral cortex, is a grooved and bumpy surface covering the nerve cells
47
Downloaded by Nhi An (annhinguyendang@gmail.com)
lOMoARcPSD|15007663

TRAN VAN HAU (ERIC TRAN) VSTEP – READING B1 UPDATED 2019


beneath. The various sections of the cerebrum are the sensory cortex, which is responsible for receiving
and decoding sensory messages from throughout the body; the motor cortex, which sends action
instructions to the skeletal muscles; and the association cortex, which receives, monitors, and processes
information. It is in the association cortex that the processes that allow humans to think take place.

The cerebellum, located below the cerebrum in the back part of the skull, is the section of the brain that
controls balance and posture. The brain stem which is a lengthening stem connects the cerebrum and the
spinal cord. It controls various body processes such as breathing and heartbeat. It is the major motor and
sensory pathway connecting the body and the cerebrum.

1. What is the author's main purpose?


(A) To describe the functions of the parts of the brain
(B) To explain how the brain processes information
(C) To demonstrate the physical composition of the brain
(D) To give examples of human body functions
2. The passage states that the most massive part of the brain is the ………………………
(A) cerebrum (B) cerebellum (C) cerebral cortex (D) brain stem
3. How does the passage describe the appearance of the cerebral cortex?
(A) As smooth (B) As 85% of the brain by weight
(C) As a layer of the cerebellum (D) As ridged of the layer
4. According to the passage, which part of the brain analyzes information?
(A) The sensory cortex (B) The association cortex
(C) The cerebellum (D) The brain stem
5. The sensory cortex………………………..
(A) senses that messages should be sent out to the muscles.
(B) provides a surface covering for nerve cells.
(C) is where the human process of thinking occurs.
(D) receives and processes information from the senses.
6. Which of the following is true about the cerebellum?
(A) It is located above the cerebrum. (B) It controls breathing.
(C) It is responsible for balance. (D) It is the outside layer of the cerebrum.
7. What shape does the brain stem most likely have?
(A) Small and round (B) Long and thin
(C) Large and formless (D) Short and flat
8. The word <stores= in paragraph 1 is closet in meaning to:
A. shops B. Processes C. Releases D. Stockpiles
9. The word <pathway= in paragraph 3 can best replaced:
A. driveway B. roadway C. route D. street
PASSAGE 2: QUESTIONS 10 -20

48
Downloaded by Nhi An (annhinguyendang@gmail.com)
lOMoARcPSD|15007663

TRAN VAN HAU (ERIC TRAN) VSTEP – READING B1 UPDATED 2019


(1) Though Edmund Halley was most famous because of his achievements as an astronomer, he was
a scientist of diverse interests and great skill. In addition to studying the skies, Halley was also
deeply interested in exploring the unknown depths of the oceans. One of his lesser-known
accomplishments that were quite remarkable was his design for a diving bell that facilitated
exploration of the watery depths.

(2) The diving bell that Halley designed had a major advantage over the diving bells that were in use
prior to his. Earlier diving bells could only make use of the air contained within the bell itself, so
divers had to surface when the air inside the bell ran low. Halley?s bell was an improvement in that
its design allowed for an additional supply of fresh air that enabled a crew of divers to remain
underwater for several hours.

(3) The diving contraption that Halley designed was in the shape of a bell that measured three feet
across the top and five feet across the bottom and could hold several divers comfortably; it was open
at the bottom so that divers could swim in and out at will. The bell was built of wood, which was
first heavily tarred to make it water repellent and was then covered with a half-ton sheet of lead to
make the bell heavy enough to sink in water. The bell shape held air inside for the divers to breathe
as the bell sank to the bottom.

(4) The air inside the bell was not the only source of air for the divers to breathe, and it was this
improvement that made Halley?s bell superior to its predecessors. In addition to the air already in the
bell, air was also supplied to the divers from a lead barrel that was lowered to the ocean floor close to
the bell itself. Air flowed through a leather pipe from the lead barrel on the ocean floor to the bell.
The diver could breathe the air from a position inside the bell, or he could move around outside the
bell wearing a diving suit that consisted of a lead bell-shaped helmet with a glass viewing window
and a leather body suit, with a leather pipe carrying fresh air from the diving bell to the helmet.
10.The subject of the preceding passage was most likely Halley?s……..
A. childhood B. work as an astronomer
C. invention of the diving bell D. many different interests
11. Halley?s bell was better than its predecessors because it
A. was bigger B. provided more air
C. weighed less D. could rise more quickly
12. The expression <ran low= in paragraph 2 is closest in meaning to
A. sank to the bottom B. was almost exhausted
C. had been replenished D. move slowly
13. Which of the following best expresses the subject of this passage?
A. Halley?s work as an astronomer B. Halley?s many different interests
C. Halley?s invention of a contraption for diving D. Halley?s experiences as a diver
14. How long could divers stay underwater in Halley?s bell?

49
Downloaded by Nhi An (annhinguyendang@gmail.com)
lOMoARcPSD|15007663

TRAN VAN HAU (ERIC TRAN) VSTEP – READING B1 UPDATED 2019


A. Just a few seconds B. Only a few minutes
C. For days on end D. For hours at a time
15. It is NOT stated in the passage that Halley?s bell
A. was completely enclosed B. was wider at the top than at the bottom
C. could hold more than one diver D. was made of tarred wood
16. The expression <at will= in paragraph 3 could best be replaced by………………….
A. in the future B. upside down C. as they wanted D. with great speed
17. It can be inferred from the passage that, were Halley?s bell not covered with lead, it would
A. float B. get wet C. trap the divers D. suffocate the divers
18. Where in the passage does the author indicate how air traveled from the barrel to the bell?
A. Lines 8-10 B. Lines 11-13 C. Lines 16-17 D. line 19
19. In which paragraph does the author describe the diving bells that preceded Halley?s?
A. The first paragraph B. The second paragraph
C. The third paragraph D. The fourth paragraph
20. This passage would most likely be assigned reading in a course on
A. astronomy B. physiology C. oceanography D. recreation
PASSAGE 3: QUESTIONS 21- 30
The tides are a natural phenomenon involving the alternating rise and fall in the large fluid bodies of
the earth caused by the combined gravitational attraction of the sun and moon. The combination of
these two variable forces influences produce the complex recurrent cycle of the tides. Tides may
occur in both oceans and seas, to a limited extent in large lakes, the atmosphere, and, to a very
minute degree, in the earth itself. The period between succeeding tides varies as the result of many
factors.

The tide-generating force represents the difference between the centrifugal force produced by the
revolution of the earth around the common center-of-gravity of the earth-moon system and the
gravitational attraction of the moon acting upon the earth's overlying waters. Since, on the average,
the moon is only 238,852 miles from the earth compared with the sun's greater distance of
92,956,000 miles, this closer distance outranks the much smaller mass of the moon compared to the
sun, and the moon?s tide raising force is more than twice that of the sun.

The effect of the tide-generating forces of the moon and sun acting tangentially to the earth's surface
(the so-called "tractive force") tends to cause a maximum accumulation of the waters of the oceans at
two diametrically opposite positions on the surface of the earth and to withdraw compensating
amounts of water from all points 90 degrees removed from the positions of these tidal bulges. As the
earth rotates beneath the maxima and minima of these tide-generating forces, a sequence of two high
tides, separated by two low tides, ideally is produced each day.

50
Downloaded by Nhi An (annhinguyendang@gmail.com)
lOMoARcPSD|15007663

TRAN VAN HAU (ERIC TRAN) VSTEP – READING B1 UPDATED 2019


Twice in each lunar month, when the sun, moon, and earth are directly aligned, with the moon
between the earth and sun (at new moon) or on the opposite of the earth from the sun (at full moon),
the sun and the moon exert their gravitational force in a mutual or additive fashion. Higher high tides
and lower low tides are produced. These are called spring tides. At two positions 90 degrees in
between, the gravitational forces of the moon and sun - imposed at right angles - then to counteract
each other to the greatest extent, and the range between high and low tides is reduced. These are
called neap tides.

The actual range of tide in the waters of the open ocean may amount to only one or two feet,
However, in Nova Scotia along the narrow channel of the Bay of Fundy, the range of tides may reach
43 feet or more (under spring tide conditions) due to resonant amplification. In every case, actual
high or low tide can vary considerably from the average due to weather conditions such as strong
winds, abrupt barometric pressure changes, or prolonged periods of extreme high or low pressure.
21. The passage supports all of the following statements EXCEPT:
A. high tides occur everywhere on earth at the same time.
B. tides are largely the result of the moon?s gravity
C. tides occur in all of earth?s large bodies of water
D. the distance between high and low tides varies throughout the year
22. The phrase= these two variable forces= in paragraph 1refers to………
A. high and low tides B. the alternating rise and fall
C. the gravity of the moon and sun D. oceans and seas
23. The word < recurrent= in paragraph 1 refers to…………….
A. simultaneous B. repeated C. interrupted D. resistant
24. According to the passage, tides occur………………………..
A. in the earth?s gravity B. in the earth?s atmosphere
C. in large rivers D. on the moon
25. The word <outranks= in paragraph 2 is closet in meaning to ……………
A. multiples B. reflects C. overcomes D. neutralizes
26. According to the passage, the moon………………………
A. has more mass than the sun B. is full at least twice a month
C. is farther from the earth than the sun D. affects tides more than the sun
27. The word <bulges= in paragraph 3 is closet in meaning to………………
A. basins B. ridges C. swellings D. pools
28. At full moon, the position of the earth is…………………
A. between the moon and the sun
B. on the opposite side of the moon from the sun
C. on the opposite side of the sun from the moon
D. determined by the Sun
51
Downloaded by Nhi An (annhinguyendang@gmail.com)
lOMoARcPSD|15007663

TRAN VAN HAU (ERIC TRAN) VSTEP – READING B1 UPDATED 2019


29. What is the purpose of the paragraph 5?
A. to discuss the range of tide in the open ocean
B. to show examples of actual high and low tides
C. to explain tide – generating forces
D. to explain the effect of weather on tides
30. It can be concluded from the three last sentences of final paragraph that
A. high and low tides remain stable throughout the year
B. high tides can cause extreme weather conditions
C. changes in atmosphere pressure influence tides
D. low pressure causes lower tides
PASSAGE 4: QUESTIONS 31 -40
Harvard University, today recognized as part of the top echelon of the world's universities, came
from very inauspicious and humble beginning.
This oldest of American universities was founded in 1636, just sixteen years after the Pilgrims
landed at Plymouth. Included in the Puritan emigrants to the Massachusetts colony during this period
were more than 100 graduates of England's prestigious Oxford and Cambridge universities, and these
universities graduates in the New Word were determined that their sons would have the same
educational opportunities that they themselves had had. Because of this support in the colony for an
institution of higher learning, the General Court of Massachusetts appropriated 400 pounds for a
college in October of 1636 and early the following year decided on a parcel of land for the school;
this land was in an area called New town, which was later renamed Cambridge after its English
cousin and is the site of the present-day university.
When a young minister named John Harvard, who came from the neighboring town of
Charlestown, died from tuberculosis in 1638, he willed half of his estate of 1,700 pounds to the
fledgling college. In spite of the fact that only half of the bequest was actually paid, the General
Court named the college after the minister in appreciation for what he had done. The amount of the
bequest may not have been large, particularly by today's standard, but it was more than the General
Court had found it necessary to appropriate in order to open the college.
Henry Dunster was appointed the first president of Harvard in 1640, and it should be noted that in
addition to serving as president, he was also the entire faculty, with an entering freshmen class of
four students. Although the staff did expand somewhat, for the first century of its existence the entire
teaching staff consisted of the president and three or four tutors.
31. The main idea of this passage is that ______________ .
A. Harvard University developed under the auspices of the General Court of Massachusetts
B. What is today a great university started out small
C . John Harvard was key to the development of a great university
D. Harvard is one of the world's most prestigious universities.
32. The passage indicates that Harvard is _______________
A. one of the oldest universities in the world

52
Downloaded by Nhi An (annhinguyendang@gmail.com)
lOMoARcPSD|15007663

TRAN VAN HAU (ERIC TRAN) VSTEP – READING B1 UPDATED 2019


B . the oldest university in the world
C . one of the oldest universities in America
D. the oldest university in America
33. It can be inferred from the passage that the Puritans who traveled to the Massachusetts colony
were _________________
A. rather rich B. Rather well educated
C . rather supportive of the English government D. rather undemocratic
34. The pronoun "they" in the second paragraph refers to _______________
A. son B. university graduates
C . Oxford and Cambridge universities D. educational opportunities
35. The "pounds" in the second paragraph are probably ______________
A. units of money B. college students
C. types of books D. school campuses
36.Which of the following is NOT mentioned about John Harvard?
A. What he died of B. Where he came from
C. Where he was buried D. How much he bequeathed to Harvard
37. The passage implies that __________________ .
A. Someone else really served as president of Harvard before Henry Dunster
B. Henry Dunster was an ineffective president
C . Henry Dunster spent much of his time as president managing the Harvard faculty
D. The position of president of Harvard was not merely an administrative position in the early ears
38. The < English cousin= in paragraph 2 refers to a…………….
A. city B. relative .C. person D. court
39. The word < fledgling= in paragraph 3 could be best replaced by…………….
A. newborn B. flying C. winged D. established
40. The word "somewhat" in the last paragraph could best be replaced by _______
A. to and from B. Back and forth C. side by side D. more or less
SESSION 8
PRACTICE TEST 6:
PASSAGE 1 – Questions 1 - 10
The final battle of the War of 1812 was the Battle of New Orleans. This battle gave a clear
demonstration of the need for effective communication during wartime; it also showed the
disastrous results that can come to pass when communication is inadequate.
The War of 1812 was fought between Great Britain and the very young country of the United
States only a relatively few years after the United States had won its independence from Britain.
The United States had declared war against Britain in June of 1812, mostly because of
interference with U.S. shipping by the British and because of the shanghaiing of U.S. sailors for
53
Downloaded by Nhi An (annhinguyendang@gmail.com)
lOMoARcPSD|15007663

TRAN VAN HAU (ERIC TRAN) VSTEP – READING B1 UPDATED 2019


enforced service on British vessels. The war lasted for a little more than two years, when a peace
treaty was signed at Ghent, in Belgium, on the 24th of December, 1814.
Unfortunately, the news that the Treaty of Ghent had been signed and that the war was officially
over was not communicated in a timely manner over the wide distance to where the war was
being contested. Negotiations for the treaty and the actual signing of the treaty took place in
Europe, and news of the treaty had to be carried across the Atlantic to the war front by ship. A
totally unnecessary loss of life was incurred as a result of the amount of time that it took to
inform the combatants of the treaty.
Early in January of 1815, some two weeks after the peace treaty had been signed, British troops
in the southern part of the United States were unaware that the war had officially ended. Over
5,000 British troops attacked U.S. troops. During the ensuing battle, known as the Battle of New
Orleans, the British suffered a huge number of casualties, around 2,000, and the Americans lost
71 , all in a battle fought only because news of the peace treaty that had already been signed in
Ghent had not yet reached the battlefield.
1. The main idea of this passage is that ………………………
A. The War of Independence was unnecessary
B. The War of 1812 was unnecessary
C. The Treaty of Ghent was unnecessary
D. The Battle of New Orleans was unnecessary
2. The word 'it' in paragraph 1 represents ……………………
A. battle B. demonstrate C. communication D. wartime
3. Look at the expression come to pass in paragraph 1 . This expression could best be replaced
by
A. happen B. overthrow C. self – destruct D. circumvent
4. According to the passage, when did the United States win its independence from Britain?
A. Shortly before the War of 1812 C. During the War of 1812
B. Just after the War of 1812 D. Long after the War of 1812
5. According to the passage, some U.S. sailors were……………
A. taken forcibly to Shanghai B. made to go to Ghent
C. forced to work on British ships D. responsible for causing the War of 1812
6. It is NOT stated in the passage that Ghent was ……….
A. where negotiations took place B. The Site of the final battle
C. Where the treaty was signed D. far from the battle field
7. The word contested in paragraph 4 is the closest in meaning to……………………
A. played B. fought C. discussed D. examined
8. It can be determined from the passage that, of the following dates, the Battle of New Orleans
was most probably fought ………………..
A. on December 10, 1814 B. on December 24, 1814
C. on January 1, 1815 D. on January 8, 1815
9. Where in the passage does the author indicate when the War of 1812 officially ended?
54
Downloaded by Nhi An (annhinguyendang@gmail.com)
lOMoARcPSD|15007663

TRAN VAN HAU (ERIC TRAN) VSTEP – READING B1 UPDATED 2019


A. lines 1 -4 B. lines 5 -7 C. lines 8 -9 D. lines 12 -14
10. Which paragraph best describes the battle that took place after the signing of the treaty?
A. Paragraph 1 B. Paragraph 2 C. Paragraph 3 D. Paragraph 4
PASSAGE 2 – QUESTIONS 11- 20
Carbon dating can be used to estimate the age of any organic natural material; it has been used
successfully in archeology to determine the age of ancient artifacts or fossils as well as in a
variety of other fields. The principle underlying the use of carbon dating is that carbon is a part
of all living things on Earth. Since a radioactive substance such as carbon-14 has a known half-
life, the amount of carbon-14 remaining in an object can be used to date that object.
Carbon-14 has a half-life of 5,570 years, which means that after that number of years, half of
the carbon- 14 atoms have decayed into nitrogen-14. It is the ratio of carbon-14 in that substance
that indicates the age of the substance. If, for example, in a particular sample the amount of
carbon-14 is roughly equivalent to the amount of nitrogen-14, this indicates that around half of
the carbon-14 has decayed into nitrogen-14, and the sample is approximately 5,570 years old.
Carbon dating cannot be used effectively in dating objects that are older than 80,000 years.
When objects are that old, much of the carbon-14 has already decayed into nitrogen-14, and the
molecule amount that is left doesn?t provide a reliable measurement of age. In the case of older
objects, other age-dating methods are available, methods which use radioactive atoms with
longer half-lives than carbon has.
11. This passage is mainly about______.
A. archeology and the study of ancient artifacts.
B. one method of dating old objects.
C. various uses for carbon.
D. the differences between carbon-14 and nitrogen-14.
12. The word < estimate= in line 1 is closet in meaning to……………..
A. understand B. hide C. rate D. approximate
13. Which of the following is NOT true about carbon-14?
A. It and nitrogen always exist in equal amounts in any substance.
B. Its half-life is more than 5,000 years.
C. It can decay into nitrogen-14.
D. It is radioactive.
14. The word <it= in paragraph 1 refers to______.
A. carbon dating B. the age
C. any organic natural material D. archeology
15. The word <underlying= could best be replaced by______.
A. below B. requiring C. serving as a basis for D. being studied
through
16. It can be inferred from the passage that if an item contains more carbon-14 than nitrogen-14,
then the item is______.
55
Downloaded by Nhi An (annhinguyendang@gmail.com)
lOMoARcPSD|15007663

TRAN VAN HAU (ERIC TRAN) VSTEP – READING B1 UPDATED 2019


A. not as much as 5,570 years old B. too old to be age-dated with carbon-14
C. too radioactive to be used by archeologists D. more than 5.570 years old
17. The word <roughly= in paragraph 2 is closest in meaning to______.
A. precisely B. harshly C. approximately D. coarsely
18. The paragraph following the passage most probably discusses______.
A. what substances are part of all living things. B. how carbon-14 decay intonitrogen.-
14.
C. why carbon-14 has such a long half-life. D. various other age-dating methods.
19. It is implied in the passage that______.
A. carbon dating has no known uses outside of archeology.
B. fossils cannot be age-dated using carbon-14.
C. carbon dating could not be used on an item containing nitrogen.
D. carbon-14 does not have the longest known half-life.
20. The expression= is left= in the last paragraph could be best replaced
by…………………
A. remains B. has turned C. changes D. is gone
PASSAGE 3: QUESTIONS 21- 30
Mount Rushmore is a well-known monument in the Black Hills of South Dakota that features
the countenances of four U.S. presidents: Washington, Jefferson, Roosevelt, and Lincoln. What
is not so well known is that the process of creating this national treasure was not exactly an
uneventful one.
Mount Rushmore was the project of the visionary sculptor John Gutzen de la Mothe Borglum,
who was born in Idaho but studied sculpture in Paris in his youth and befriended the famous
French sculptor Auguste Rodin. In 1927 Borglum was granted a commission by the federal
government to create the sculpture on Mount Rushmore. Though he was nearly sixty years old
when he started, he was undaunted by the enormity of the project and the obstacles that it
engendered. He optimistically asserted that the project would be completed within five years, not
caring to recognize the potential problems that such a massive project would involve, the
problems of dealing with financing, with government bureaucracy, and with Mother Nature
herself. An example of what Mother Nature had to throw at the project was the fissure that
developed in the granite where Jefferson was being carved. Jefferson had to be moved to the
other side of Washington, next to Roosevelt, because of the break in the stone. The work that
had been started on the first Jefferson had to be dynamited away.
Mount Rushmore was not completed within the five years predicted by Borglum and was in fact
not actually completed within Borglum's lifetime, although it was almost finished. Borglum died
on March 6, 1941, at the age of seventy-four, after fourteen years of work on the presidents. His
son, Lincoln Borglum, who had worked with his father throughout the project, completed the
monument within eight months of his father's death.
21. Which of the following best expresses the main idea of the passage?
A. Mount Rushmore was a huge project filled with numerous obstacles.
56
Downloaded by Nhi An (annhinguyendang@gmail.com)
lOMoARcPSD|15007663

TRAN VAN HAU (ERIC TRAN) VSTEP – READING B1 UPDATED 2019


B. Mount Rushmore is a famous American monument.
C. Mount Rushmore has sculptures of four U.S. presidents on it.
D. John Gutzen dela Mothe Borglum created Mount Rushmore.
22. The word <countenances= could be best replaced by………..
A. museums B. faces C. graves D. relatives
23. Which of the following best describes the relationship between Borglum and Rodin?
A. Borglum studied about Rodin in Paris.
B. Borglum was far more famous than Rodin as a sculptor.
C. Borglum and Rodin were born and raised in the same place.
D. Borglum and Rodin were friends.
24. Which of the following is not true about Borglum?
A. He began Mount Rushmore around the age of sixty.
B. He predicted that Mount Rushmore would be finished around 1932.
C. Mount Rushmore was finished when Borglum predicted it would be.
D. Borglum worked on Mount Rushmore for more than a decade.
25. It can be inferred from the passage that Borglum was someone who…………
A. expected the best to happen
B. set realistic goals
C. Never tried anything to challenging
D. was always afraid that bad things were going to happen
26. The word <fissure= could be best replaced by………..
A. discoloration B. crack C. unevenness D. softness
27. Why does the author mention the fact that the carving of Tomas Jefferson was removed?
A. To show what a perfectionist Borglum was.
B. It demonstrates Borglum?s artistic style.
C. It gives insight into Jefferson?s character.
D. It is an example of a problem caused by nature.
28. The pronoun= It= refers which of the following?
A. The first Jefferson B. Mount Rushmore C. Borglum?s lifetime D. 14 years of work
29. The word <nearly= could be best replaced by………..
A. Over B. Closely C. Almost D. Barely
30. Which of the following is closest in meaning to the expression within eight months of his
father9s death=?
A. More than eight months before his father?s death
B. Less than eight months before his father?s death
C. Less than eight months after his father?s death
D. More than eight months after his father?s death
PASSAGE 4 – QUESTIONS 31 – 40
The Fall of the Roman Empire

57
Downloaded by Nhi An (annhinguyendang@gmail.com)
lOMoARcPSD|15007663

TRAN VAN HAU (ERIC TRAN) VSTEP – READING B1 UPDATED 2019


At its peak in the year 106 AD, the Roman Empire consisted of 52 provinces and influenced or
controlled some 2.3 million square miles of territory around the Mediterranean Sea. The
Roman army was efficient and well-organized, and the ruling classes in Rome, including
the Emperor, were the richest and most powerful men in the Western world. Yet within
three hundred years the Empire was on the verge of collapse, and the reasons for its decline and
fall have interested historical analysts for centuries.

(A) The seeds of the destruction of the Empire may have been sown in 44 BC, when Julius
Caesar appointed himself >dictator for life?. (B) The rapid growth of the Empire under this
famous general was based on conquest and plunder, and its economy depended on the taxes,
slaves and treasure that could be extorted from the new provinces..(C) In the later years of the
Empire, the army was spread too thinly to defend the Empire?s borders against attacks from
Germanic barbarians to the north and Persian invaders to the east, and relied more and more
upon locally recruited mercenary soldiers who lacked the patriotic pride and discipline of the
Roman legionaries.(D)

The leadership of the Empire was often a cause of insurgency and discontent among the Roman
citizens and the Empire?s subjects in other countries. There were some good Emperors over the
centuries; but there were also a substantial number who were cruel, weak, insane or greedy.
The worst of these rulers imposed heavy taxes, passed insupportable laws and executed their
enemies; so there were always rebellions in the provinces, and the army was always dealing
with a problem somewhere in the Empire. Even at home, long periods of civil war were
common, usually caused by arguments over the right to the title of Emperor – in one year there
were no fewer that 25 soldier-emperors appointed as the army joined in the struggle for power.

The Empire eventually became too large to be ruled directly from Rome, and the Emperor
Diocletian divided his realm into two parts in 250 AD, each with its own Emperor and its own
capital city; Milan and Nicomedia, and later Ravenna and Constantinople. Partly as a result of
epidemics of plague and other diseases in the West, most of the population of the Empire lived
in the East, so the old problems of finding recruits for the Western army on its Germanic and
Gallic borders continued.

In addition, the Western Empire had economic problems as gold became scarcer to find,
making it difficult to pay the soldiers and causing further unrest in the army. Meanwhile, the
aristocracy of the Empire continued to live their extravagant lifestyles which the failing
economy could not support. One interesting theory holds that a major cause of illness among
the aristocracy could be traced to their wine bottles, water pipes and even their cosmetics, all of
which were made from lead and other heavy metals and simply caused unexplained deaths
which we now know were due to lead poisoning on a massive scale.

It is possible that the gradual spread of Christianity made the true Roman citizens more tolerant,
less belligerent and less inclined to force their way of life upon others, paving the way for the
58
Downloaded by Nhi An (annhinguyendang@gmail.com)
lOMoARcPSD|15007663

TRAN VAN HAU (ERIC TRAN) VSTEP – READING B1 UPDATED 2019


withdrawal of their forces from the lands they had conquered. It is more likely that the army
became so badly-organized that they could not withstand the strength of the rebellions on all
their borders, and the true Roman soldierly spirit was diluted by the influx of barbarian or non-
Roman mercenary recruits. One of these mercenary chiefs, Odoacer, led a revolt in the year 476
caused by the Emperor?s refusal to pay his men in land for their services, and met no opposition
as he swept into Italy and overran what remained of the Western Empire, deposing the true
Emperor who fled and never returned. This event is generally thought of as the end of the
Roman Empire, although the Eastern Empire continued in Constantinople for another thousand
years.
31. According to paragraph 1, which was true of the Roman Empire?
A. it declined and fell apart in the course of the 2nd century
B. its government ruled the provinces ineffectively
C. it was a union of territories in central Europe.
D. it prospered during the early 2nd centuries A.D
32. The word= recruited= in paragraph 2 is closed in meaning to…………………..
A. qualified B. dominated C. fired D. hired
33. Look at the four letters (A, B, C and D) indicate where the following sentence could be
added to the passage in paragraph 2.
This one – way traffic had to come to an end when there were no more land to conquer
within the reach of Rome, and the cost of protecting and administrating the provinces
became too high.
Where would be the sentence best fit? Choose the place to add it.
A. (A) B. (B) C. (C) D. (D)
34. Which of the following best expresses the essential information in the highlighted sentence
in the passage? Incorrect choices change the meaning in important ways leave out essential
information.
A. Rome had one of the largest and most useful armies in the world
B. Rich and powerful men usually relocated from Western Europe to Rome in order to satisfy
their urge for Roman cuisine.
C. Rome has a formidable army that helped to make its citizens among the wealthiest in the
world, while other countries were weaker.
D. Roman soldiers were generally paid in land that was used for agriculture.
35. The author mentions < The worst of these rulers= in paragraph 3 in order to……………..
A. detail how the empire expanded rapidly
B. provide an reason for further discussion of emperors
C. demonstrate the unique characteristics of Roman emperors
D. illustrate one possible reason for the fall of the Roman Empire
36. The word <his= in line 21 refers to……………….
A. Diocletian B. Caesar C. money D. Rome
59
Downloaded by Nhi An (annhinguyendang@gmail.com)
lOMoARcPSD|15007663

TRAN VAN HAU (ERIC TRAN) VSTEP – READING B1 UPDATED 2019


37. According to the passage, which of the following was NOT true of the Roman Empire?
A. It was divided into two parts
B. It was ruled by Julius Caesar in 44 B.C
C. It hosted many games and concerts
D. it battled with the Germanic tribes in the West
38. The word <extravagant = in lines 28 is closet in meaning to…………….
A. boring B. prodigal C. austere D. simple
39. All of the following were the effects of the gradual spread of Christianity on Roman citizens
EXCEPT………..
A. power B. belligerence C. withdrawal D. tolerance
30. It can be inferred from the passage that the author most likely believes which of the
following about the fall of the Roman Empire?
A. there were many possible reasons
B. the introductions of Christianity was the key reason for the decline.
C. it was inevitable because of the cultural diversity.
D. the Roman Empire fell because it was bankrupted.

SESSION 9
PRACTICE TEST 7
PASSAGE 1 – QUESTIONS 1 -10
Staggering tasks confronted the people of the United States, North and South, when
the Civil War ended. About a million and a half soldiers from both sides had to be
demobilized, readjusted to civilian life, and reabsorbed by the devastated economy.
Line Civil government also had to be put back on a peacetime basis and interference from
(5) the military had to be stopped.

The desperate plight of the South has eclipsed the fact that reconstruction had to be
undertaken also in the North, though less spectacularly. Industries had to adjust to
peace time conditions; factories had to be retooled for civilian needs.

Financial problems loomed large in both the North and the South. The national debt
(10) had shot up from a modest $65 million in 1861, the year the war started, to nearly $3
billion in 1865, the year the war ended. This was a colossal sum for those days but one
that a prudent government could pay. At the same time, war taxes had to be reduced to
less burdensome levels.

Physical devastation caused by invading armies, chiefly in the South and border
(15) states, had to be repaired. This herculean task was ultimately completed, but with
60
Downloaded by Nhi An (annhinguyendang@gmail.com)
lOMoARcPSD|15007663

TRAN VAN HAU (ERIC TRAN) VSTEP – READING B1 UPDATED 2019


discouraging slowness.

Other important questions needed answering. What would be the future of the four
million black people who were freed from slavery? On what basis were the Southern
states to be brought back into the Union?

(20) What of the Southern leaders, all of whom were liable to charges of treason? One
of these leaders, Jefferson Davis, president of the Southern Confederacy, was the
subject of an insulting popular Northern song, "Hang Jeff Davis from a Sour Apple
Tree," and even children sang it. Davis was temporarily chained in his prison cell
during the early days of his two-year imprisonment. But he and the other Southern
(25) leaders were finally released, partly because it was unlikely that a jury from Virginia, a
Southern Confederate state, would convict them. All the leaders were finally pardoned
by President Johnson in 1868 in an effort to help reconstruction efforts proceed with as
little bitterness as possible.
1. What does the passage mainly discuss?
(A) Wartime expenditures
(B) Problems facing the United States after the war
(C) Methods of repairing the damage caused by the war
(D) The results of government efforts to revive the economy
2. The word "Staggering" in line 1 is closest in meaning to
(A) specialized (B) confusing (C) various (D) overwhelming
3. The word "devastated" in line 3 is closest in meaning to
(A) developing (B) ruined (C) complicated (D) fragile
4. According to the passage, which of the following statements about the damages in the South is
correct?
(A) It was worse than in the North. (B) The cost was less than expected.
(C) It was centered in the border states. (D) It was remedied rather quickly.
5. The passage refers to all of the following as necessary steps following
the Civil War EXCEPT
(A) helping soldiers readjust (B) restructuring industry
(C) returning government to normal (D) increasing taxes
6. The word "task" in line 15 refers to
(A) raising the tax level (B) sensible financial choices
(C) wise decisions about former slaves (D) reconstructions of damaged areas
7. Why does the author mention a popular song in lines 22-23?
(A) To give an example of a Northern attitude towards the South
(B) To illustrate the Northern love of music
(C) To emphasize the cultural differences between the North and South
(D) To compare the Northern and Southern presidents
61
Downloaded by Nhi An (annhinguyendang@gmail.com)
lOMoARcPSD|15007663

TRAN VAN HAU (ERIC TRAN) VSTEP – READING B1 UPDATED 2019


8. The word "them" in line 26 refers to
(A) charges (B) leaders (C) days (D) irons
9. Which of the following can be inferred from the phrase "...it was unlikely
that a jury from Virginia, a Southern Confederate state, would convict
them"(lines 25-26)?
(A) Virginians felt betrayed by Jefferson Davis.
(B) A popular song insulted Virginia.
(C) Virginians were loyal to their leaders.
(D) All of the Virginia military leaders had been put in chains.
PASSAGE 2: QUESTIONS 11 -20
These stories of killer bees in the news in recent years have attracted a lot of attention as the bees
have made their way from South America to North America. Killer bees are reputed to be
extremely aggressive in nature, although experts say that their aggression may have been
somewhat inflated.

The killer bee is a hybrid- or combination- of the very mild European strain of honeybee and the
considerably more aggressive African bee, which was created when the African strain was
imported into Brazil in 1955. The African bees were brought into Brazil because their aggression
was considered an advantage: they were far more productive than their European counterparts in
that they spent a higher percentage of their time working and continued working longer in
inclement weather than did the European bees.

These killer bees have been known to attack humans and animals, and some fatalities have
occurred. Experts point out, however, that the mixed breed known as the killer bee is actually not
at all as aggressive as the pure African bee. They also point out that the attacks have a chemical
cause. A killer bee stings only when it has been disturbed; it is not aggressive by nature. However,
after a disturbed bee stings and flies away; it leaves its stinger embedded in the victim. In the
viscera attached to the embedded stinger is the chemical isoamyl acetate, which has an odor that
attracts other bees. As other bees approach the victim of the original sting, the victim tends to
panic, thus disturbing other bees and causing them to sting. The new stings create more of the
chemical isoamyl acetate which attracts more bees and increases the panic level of the victim.
Killer bees tend to travel in large clusters or swarms and thus respond in large numbers to the
production of isoamyl acetate.
11: The subject of the preceding paragraph was most likely
A. stories in the media about killer bees. B. ways of producing honey.
C. the creation of the killer bee. D. the chemical nature of killer bee attacks.
12. The main idea of this passage is that killer bees……..
A. have been moving unexpectedly rapidly through the Americas.
B. have been in the news a lot recently.
C. are a hybrid rather than a pure breed.

62
Downloaded by Nhi An (annhinguyendang@gmail.com)
lOMoARcPSD|15007663

TRAN VAN HAU (ERIC TRAN) VSTEP – READING B1 UPDATED 2019


D. are not as aggressive as their reputation suggests.
13. The word „inflated‟ in paragraph 1could be best replaced by
A. exaggerated B. blown C. burst D. aired
14: It can be inferred from the passage that the killer bees………………..
A. was carried from Africa to Brazil in 1955. B. did not exist early in the twentieth century.
C. travelled from Brazil to Africa in 1955. D. was a predecessor of the African bee.
15: Why were African bees considered beneficial?
A. They produced an unusual type of honey. B. They were very aggressive.
C. They didn?t work hard from inclement weather. D. They spent their time traveling.
16: It is stated in the passage that killer bees……………
A. are more deadly than African bees. B. never attack animals.
C. always attack African bees. D. are less aggressive than African bees.
17: What is NOT mentioned in the passage as a contributing factor in an attack by killer bees?
A. Inclement weather B. Panic by the victim
C. An odorous chemical D. Disturbance of the bees
18. The word < their= in paragraph 2 refers to………
A. the killer bees B. The African bees C. the honey bees D. the European bees
19. Which paragraph does the author describe the size of the groups in which killer bees move?
A. Paragraph 1 B. Paragraph 2 C. Paragraph 3 D. Paragraph 4
20. The pronoun <They= in paragraph 3 refers to:………..
A. human and animals B. experts C. fatalities D. killer bees
PASSAGE 3: QUESTIONS 21 -30
That night as Easton walked home through the rain he felt very depressed. It had been a very bad
summer for most people and he had not fared better than the rest. A few weeks with one firm, a
few days with another, then out of job, then on again for a month perhaps, and so on.

William Easton was a man of medium height, about 23 years old, with fair hair and moustache and
blue eyes. His clothes, though shabby, were clean and neat but the holes in his shoes made it
painful to walk.

He was married: his wife was a young woman whose acquaintance he had made when he
happened to be employed with other painting the outside of the house where she was a general
servant. Easton had been in no hurry to marry for he knew that, taking good times with bad, his
wages did not average a pound a week. However, after going out for 18 months they were finally
married.
That was a year ago.

63
Downloaded by Nhi An (annhinguyendang@gmail.com)
lOMoARcPSD|15007663

TRAN VAN HAU (ERIC TRAN) VSTEP – READING B1 UPDATED 2019


As a single man he had never troubled much if he happened to be out of work. He always had
enough to live on and pocket money besides, but now that he was married it was different; the
fear of being >out? haunted him all the time.

He had started for Rushton and Co. on the previous Monday after having been idle for three weeks
and, as the house where he was working had to be done right through, he had congratulated
himself on having secured a job that would last till Christmas; but he now began to fear that what
had happened to Jack Linden – a master craftsman – might also happen to himself at any time. He
would have to be very careful not to offend Bill Crass in any way. He was afraid that the latter
did not like him very much as it was. He knew that Crass could get him the sack at any time and
would not scruple to do so if he wanted to make room for some pal of his.

Crass, the foreman, was quite without special abilities; he was if anything inferior to the majority
of the men he supervised. Even so, he pretended to know everything, and the vague references he
was in the habit of making to >tones? and >shades? and >harmony? had so impressed Frederick
Hunter that the latter was completely taken in. it was by pushing himself forward in this way that
Crass had managed to get himself put in charge of the work.

Although Crass did as little as possible himself, he took care to work the others hard. Any man
who failed to satisfy him was reported to Hunter as being >no good? or >too slow for a funeral? and
was then dispensed with at the end of the week. Knowing this, all the workers feared and hated the
wily Crass.

Some, by giving him pipefuls of tobacco and pints of beer, managed to stay in Crass?s favour and
often kept their jobs when better men were dismissed. As he walked home through the rain
thinking of these things, Easton realized that it was not possible to foresee what a day or even an
hour might bring.
21.As he walked home, Easton felt depressed because
A it had been a bad summer for most people, including him.
B he was afraid of losing his job.
C he had recently got married, despite his low wages.
D his shoes were worn out and his feet were hurting.
22 The fifth paragraph mentions Easton?s fear of being 8out9. Is there a fear of
A being unemployed? B not having any money?
C having nowhere to live? D falling out with his wife?
23. The word= haunted= in line 16 is closet in meaning to……………
A. impressed B. obsessed C. pressed D. depressed
24. The word < the later= in line 22 refers to…
A Jack Linden. B Frederick Hunter. C Bill Crass. D William Easton.

64
Downloaded by Nhi An (annhinguyendang@gmail.com)
lOMoARcPSD|15007663

TRAN VAN HAU (ERIC TRAN) VSTEP – READING B1 UPDATED 2019


25. The word < pal= in line 24 is closet in meaning to………
A. craftsman B. relative C. staff D. friend
26. The most senior person mentioned in the passage is
A Jack Linden. B Frederick Hunter. C Bill Crass. D William Easton.
27. Crass got his position because Hunter thought he was good at
A using language. B managing other people.
C understanding colour schemes. D repairing or decorating houses.
28. To keep his job, anyone working under Crass had to
A work hard. B give him presents.
C take care not to offend him. D make room for his >pals? (friends).
29. Crass was…………………
A a skillful worker but lazy. B not very skillful but hard working.
C not very skillful but also lazy. D a skillful man and a hard worker.
30. A good title for the passage would be
A Foreman Crass. B An Uncertain Future.
C Too Slow for a Funeral. D A Miserable Walk.
PASSAGE 4- QUESTIONS 31 -40
The tides are a natural phenomenon involving the alternating rise and fall in the large fluid bodies
of the earth caused by the combined gravitational attraction of the sun and moon. The combination
of these two variable forces influences produce the complex recurrent cycle of the tides. Tides
may occur in both oceans and seas, to a limited extent in large lakes, the atmosphere, and, to a very
minute degree, in the earth itself. The period between succeeding tides varies as the result of many
factors.

The tide-generating force represents the difference between the centrifugal force produced by the
revolution of the earth around the common center-of-gravity of the earth-moon system and the
gravitational attraction of the moon acting upon the earth's overlying waters. Since, on the average,
the moon is only 238,852 miles from the earth compared with the sun's greater distance of
92,956,000 miles, this closer distance outranks the much smaller mass of the moon compared to
the sun, and the moon?s tide raising force is more than twice that of the sun.

The effect of the tide-generating forces of the moon and sun acting tangentially to the earth's
surface (the so-called "tractive force") tends to cause a maximum accumulation of the waters of the
oceans at two diametrically opposite positions on the surface of the earth and to withdraw
compensating amounts of water from all points 90 degrees removed from the positions of these
tidal bulges. As the earth rotates beneath the maxima and minima of these tide-generating forces, a
sequence of two high tides, separated by two low tides, ideally is produced each day.

65
Downloaded by Nhi An (annhinguyendang@gmail.com)
lOMoARcPSD|15007663

TRAN VAN HAU (ERIC TRAN) VSTEP – READING B1 UPDATED 2019


Twice in each lunar month, when the sun, moon, and earth are directly aligned, with the moon
between the earth and sun (at new moon) or on the opposite of the earth from the sun (at full
moon), the sun and the moon exert their gravitational force in a mutual or additive fashion. Higher
high tides and lower low tides are produced. These are called spring tides. At two positions 90
degrees in between, the gravitational forces of the moon and sun - imposed at right angles - then to
counteract each other to the greatest extent, and the range between high and low tides is reduced.
These are called neap tides.

The actual range of tide in the waters of the open ocean may amount to only one or two feet,
However, in Nova Scotia along the narrow channel of the Bay of Fundy, the range of tides may
reach 43 feet or more (under spring tide conditions) due to resonant amplification. In every case,
actual high or low tide can vary considerably from the average due to weather conditions such as
strong winds, abrupt barometric pressure changes, or prolonged periods of extreme high or low
pressure.
31. The passage supports all of the following statements EXCEPT:
A. high tides occur everywhere on earth at the same time.
B. tides are largely the result of the moon?s gravity
C. tides occur in all of earth?s large bodies of water
D. the distance between high and low tides varies throughout the year
32. The phrase= these two variable forces= in paragraph 1refers to………
A. high and low tides B. the alternating rise and fall
C. the gravity of the moon and sun D. oceans and seas
33. The word < recurrent= in paragraph 1 refers to…………….
A. simultaneous B. repeated C. interrupted D. resistant
34. According to the passage, tides occur………………………..
A. in the earth?s gravity B. in the earth?s atmosphere
C. in large rivers D. on the moon
35. The word <outranks= in paragraph 2 is closet in meaning to ……………
A. multiples B. reflects C. overcomes D. neutralizes
36. According to the passage, the moon………………………
A. has more mass than the sun B. is full at least twice a month
C. is farther from the earth than the sun D. affects tides more than the sun
37. The word <bulges= in paragraph 3 is closet in meaning to………………
A. basins B. ridges C. swellings D. pools
38. At full moon, the position of the earth is…………………
A. between the moon and the sun
B. on the opposite side of the moon from the sun
C. on the opposite side of the sun from the moon
66
Downloaded by Nhi An (annhinguyendang@gmail.com)
lOMoARcPSD|15007663

TRAN VAN HAU (ERIC TRAN) VSTEP – READING B1 UPDATED 2019


D. determined by the Sun
39. What is the purpose of the paragraph 5?
A. to discuss the range of tide in the open ocean
B. to show examples of actual high and low tides
C. to explain tide – generating forces
D. to explain the effect of weather on tides
40. It can be concluded from the three last sentences of final paragraph that
A. high and low tides remain stable throughout the year
B. high tides can cause extreme weather conditions
C. changes in atmosphere pressure influence tides
D. low pressure causes lower tides
SESSION 10
PRACTICE TEST 8
PASSAGE 1 – QUESTIONS 1 – 10
Clipper ships were the swiftest sailing ships that were ever put to sea and the most beautiful.
These ships had their days of glory in the 1840s and 1850s. The first were built in Baltimore,
but most was constructed in the shipyards of New England. It was Chinese tea that brought
them into existence. Tea loses its flavour quickly when stored in the hold of a vessel, and
merchants were willing to pay top prices for fast delivery. American ship-builders designed
clippers to fill this need. Then came the California Gold Rush of 1849, when clippers took gold
seekers from the East Coast to the West by way of Cape Horn.

Clippers were built for speed, and considerations of large carrying capacity and economical
operation were sacrificed for this purpose. They had long, slender hulls with sharp bows.
Their three slanted masts carried a huge cloud of canvas sail, including topgallants and royal
sails, and sometimes skysails and moonrakers, to capture the power of the winds. They
required a hard driving captain and a large, experienced crew.

Many records were set by clippers. Sovereign of the Seas made it from San Francisco to New
York in eighty-two days. Flying cloud did 374 miles in one day. Lightning travelled from New
York to Liverpool in thirteen days, and Ino made it from New York to Singapore in eighty-six
days.

Some 500 clippers were built in American shipyards. British yards turned out some twenty-
seven tea clippers, as the British ships were called. Unlike the wooden American ships, British
clippers were >composites? with iron frames and wooden planking. The most famous tea
clipper was the Cutty Sark.

By 1860, the age of the clippers was fading Gold diggings in California were nearly exhausted.
American investors found railroad building more profitable than clippers. Most importantly,
there was a technological innovation that doomed the clipper, and in fact, the entire age of sail;
the development of the steamship.
1. What is the author?s main purpose in writing?
A. To describe the tea trade in 1840s
B. To contrast clipper ships and steamships
67
Downloaded by Nhi An (annhinguyendang@gmail.com)
lOMoARcPSD|15007663

TRAN VAN HAU (ERIC TRAN) VSTEP – READING B1 UPDATED 2019


C. To discuss nineteen-century shipbuilding techniques
D. To provide a brief history of clipper ships
2. Which of the following is closest in meaning to the word swiftest in line 1?
A. Fastest B. Best armed C. Largest D. Most
expensive
3. According to the passage, where were the majority of clipper ships built?
A. California B. Baltimore C. New England D. Great Britain
4. In line 4, the word vessel could be best be replaced by which of the following?
A. Container B. Ship C. Cargo D. Merchant
5. According to the passage, how did the California Gold Rush affect clipper ships?
A. It encouraged the development of railroads, which completed directly with clipper ships.
B. The newly discovered gold was used to finance the construction of new ships.
C. It simulated the demand for tea on the West Coast.
D. People who wanted to participate in the Gold Rush became passengers on clipper ships.
6. According to the passage, which of the following considerations was the most importance
to the owners of clipper ships?
A. Maximum speed B. Reduced operating costs
C. Increased cargo capacity D. Small crews
7. Which of the following is closest in meaning to the word slender in line 9?
A. Titled B. Slanted C. Strengthened
D. Towered
8. What can be inferred from the passage about skysails and moonrakers?
A. Skysails were the highest sails on the mast, and moonrakers were the lowest.
B. They were not always used on clipper ships.
C. They were much larger than royal sails and topgallants.
D. They were never used on clipper ship.
9. All the following are given in the passage as reasons for the decline of clipper ships
EXCEPT
A. the end of the California Gold Rush B. competition with British tea clippers
C. the development of steamships D. investment in railroad
10. In the next paragraph, the author will most likely discuss_______________
A. the beginnings of the age of steam
B. railroad travel in the United States
C. further developments in sailing ships
D. the relationship between speed and ship design
PASSAGE 2 – QUESTIONS 11 – 20
Sometimes it's hard to figure out if you have a food allergy, since it can show up so many
different ways. Your symptoms could be caused by many other problems. You may have
rashes, hives, joint pains mimicking arthritis, headaches, irritability, or depression. The most
common food allergies are to milk, eggs, seafood, wheat, nuts, seeds, chocolate, oranges, and
tomatoes. Many of these allergies will not develop if these foods are not fed to an infant until
her or his intestines mature at around seven months. Breast milk also tends to be protective.
Migraines can be set off by foods containing tyramine, phenathylamine, monosodium
glutamate, or sodium nitrate. Common foods which contain these are chocolate, aged
cheeses, sour cream, red wine, pickled herring, chicken livers, avocados, ripe bananas, cured
meats, many Oriental and prepared foods (read the labels!).

68
Downloaded by Nhi An (annhinguyendang@gmail.com)
lOMoARcPSD|15007663

TRAN VAN HAU (ERIC TRAN) VSTEP – READING B1 UPDATED 2019


Some people have been successful in treating their migraines with supplements of B-
vitamins, particularly B6 and niacin. Children who are hyperactive may benefit from
eliminating food additives, especially colorings, and foods high in salicylates from their
diets. A few of these are almonds, green peppers, peaches, tea, grapes. This is the diet made
popular by Benjamin Feingold, who has written the book <Why your Child is Hyperactive=.
Other researchers have had mixed results when testing whether the diet is effective.
11. The topic of this passage is
A. reactions to foods B. food and nutrition
C. infants and allergies D. a good diet
12. According to the passage, the difficulty in diagnosing allergies to foods is due to
A. the vast number of different foods we eat
B. lack of a proper treatment plan
C. the similarity of symptoms of the allergy to other problems
D. the use of prepared formula to feed babies
13. The word <symptom= in paragraph 1 is closet in meaning to
A. indicates B. Diet
C. diagnosis D. prescription
14. The phrase "set off" in lines 7 is closest in meaning to
A. relieved B. identified
C. avoided D. triggered
15. What can be inferred about babies from this passage?
A. They can eat almost anything.
B. They should have a carefully restricted diet as infants.
C. They gain little benefit from being breast fed.
D. They may become hyperactive if fed solid food too early.
16. The word <hyperactive= is closet in meaning to
A. overly active B. unusually low activity
C. excited D. inquisitive
17. The author states that the reason that infants need to avoid certain foods related to
allergies has to do with the infant's
A. lack of teeth B. poor metabolism
C. underdeveloped intestinal tract D. inability to swallow solid foods
18. The word "these" in paragraph 2 refers to
A. food additives B. food colorings
C. innutritious foods D. foods high in salicylates
19. Which of the following was a suggested treatment for migraines in the passage?
A. Eating more ripe bananas
69
Downloaded by Nhi An (annhinguyendang@gmail.com)
lOMoARcPSD|15007663

TRAN VAN HAU (ERIC TRAN) VSTEP – READING B1 UPDATED 2019


B. Avoiding all Oriental foods
C. Getting plenty of sodium nitrate
D. Using Vitamin B in addition to a good diet
20. According to the article the Feingold diet is NOT…………
A. verified by researchers as being consistently effective
B. available in book form
C. beneficial for hyperactive children
D. designed to eliminate foods containing certain food additives

PASSAGE 3 – QUESTIONS 21 – 30
If we believe that clothing has to do with covering the body, and costume with the choice of
a particular form of garment for a particular use, then we can say that clothing depend
primarily on such physical conditions as climate, health, and textile manufacture, whereas
costume reflect social factors such as religious beliefs, aesthetics, personal status, and the
wish to be distinguished from or to emulate our fellows.

The ancient Greeks and the Chinese believed that we first covered our bodies for some
physical reason such as protecting ourselves from the weather elements. Ethnologists and
psychologists have invoked psychological reasons: modesty in the case of ancients, and
taboo, magical influence and the desire to please for the moderns.

In early history, costume must have fulfilled a function beyond that of simple utility,
perhaps through some magical significance, investing primitive man with the attributes of
other creatures. Ornaments identified the wearer with animals, gods, heroes or other men.
This identification remains symbolic in more sophisticated societies. We should bear in mind
that the theater has its distant origins in sacred performances, and in all period children at
play have worn disguises, so as to adapt gradually to adult life.

Costume helped inspire fear or impose authority. For a chieftain, costume embodied
attributes expressing his power, while a warrior?s costume enhanced his physical superiority
and suggested he was superhuman. In more recent times, professional or administrative
costume has been devised to distinguish the wearer and express personal or delegated
authority; this purpose is seen clearly in the judge?s robes and the police officer?s uniform.
Costume denotes power, and since power is usually equated with wealth, costume came to be
an expression of social caste and material prosperity. Military uniform denotes rank and is
intended to intimidate, to protect the body and to express membership in a group. At the
bottom of the scale, there are such compulsory costumes as the convict?s uniform. Finally,
costume can possess a religious significance that combines various elements: an actual or
symbolic identification with a god, the desire to express this in earthly life, and the desire to
enhance the wearer?s position of respect.

70
Downloaded by Nhi An (annhinguyendang@gmail.com)
lOMoARcPSD|15007663

TRAN VAN HAU (ERIC TRAN) VSTEP – READING B1 UPDATED 2019


21. The passage mainly discusses costumes in term of its
A. physical protection
B. religious significance
C. social function
D. beauty and attractiveness
22. What is the purpose of the paragraph 1?
A. To describe the uses of costume
B. To contrast costume with the clothing
C. To trade the origins of costume
D. To point out that clothing developed before costume
23. Psychological reasons for wearing garments include
A. protection from cold
B. availability of materials
C. prevention of illness
D. wishing to give pleasure
24. The word <investing= in paragraph 3 is closet meaning to
A. endowing B. creating
C. wrapping D. frightening
25. The word <Ornaments= in paragraph 3 is closet in meaning to _____.
A. garments B. representation
C. details D. decorations
26. It can be inferred from paragraph 3 that
A. The function of costume has become very sophisticated
B. Children like to identify with other creature by wearing costumes
C. Primitive people wore cloths only for sacred performances
D. Costume no longer fulfills a function beyond simple utility
27. Why does the author mention the police officer9s uniform?
A. To illustrate the aesthetic function of costume
B. To identify the wearer with a hero
C. To suggest that police are superhuman
D. To show how costume signifies authority
28. The word <denotes= in paragraph 4 is closest in meaning to _____.
A. disguises B. describes
C. indicates D. denigrates
29. The word <scale= in paragraph 4 refers to
A. symbolic identification B. military rank
71
Downloaded by Nhi An (annhinguyendang@gmail.com)
lOMoARcPSD|15007663

TRAN VAN HAU (ERIC TRAN) VSTEP – READING B1 UPDATED 2019


C. social position D. the balance
30. Which of the following would most likely NOT be reflected in a person?s costume, as it
is defined in the passage?
A. Having a heart condition
B. Playing in a baseball game
C. Working in a hospital
D. Participating in a religious ceremony
PASSAGE 4 – QUESTIONS 31 - 40
Students read for a variety of reasons, but probably the most important reason students read
is to gain academic knowledge. Their goal is to learn and comprehend what they read. To be
successful, they must develop certain reading skills. However, some students meet difficulty
in comprehending and processing information taught by their teachers. This may be for a
variety of reasons. Often it is that they have never developed the proper learning strategies
necessary for comprehension. Therefore, their ability to read, learn, memorize and recall
information is impaired. Students then become discouraged from learning. There are,
however, a number of effective measures for those with reading comprehension difficulties.
Visual
Students respond well when oral or written information is presented to them in a visual way.
For example, when a skill or topic is both described and demonstrated at the same time, it
provides students with the means to associate the demonstration with the description. When
students are provided with a combination of visual, written and oral instruction, they can
better comprehend and process the information.
Participation
Passive listening and reading often result in the inability to process information because
students are not engaged or stimulated. One of the best remedies for this is for students to
participate in the topics they read about. Teachers can ask students to demonstrate what they
have learned, and give them feedback and praise.
Thinking Aloud
Students are encouraged in reading comprehension skills when they "think aloud." The
think-aloud strategy helps them to vocally express the material they are reading or the subject
they are learning about. It helps if students go through each step or take the subject one step
at a time. Talking while they think helps them to verbalize and better process the
information. Students can move from thinking aloud to whispering softly, and eventually
they will no longer require thinking aloud to comprehend. This is a gradual process that
equips students with the ability to think and process silently through a series of gradual
techniques.
Practice
Practice will help students give meaning to what they have been hearing or reading. Once the
teacher has provided them with the information, students can demonstrate what they have
learned through homework and in-class assignments. It is up to the teacher to assess the
difficulty of these assignments for students. Start off with easy assignments to complete so
72
Downloaded by Nhi An (annhinguyendang@gmail.com)
lOMoARcPSD|15007663

TRAN VAN HAU (ERIC TRAN) VSTEP – READING B1 UPDATED 2019


students are encouraged, then gradually increase the complexity and difficulty over time.
Encourage students to read over their work and process what they have read to ensure they
are grasping all the pertinent information.

31. Which of the following is true of readers who are reading to gain academic knowledge?
A. Reading increases their alertness.
B. Their goal is to learn and understand the materials they are reading.
C. They physically learn what they are reading.
D. They read for a variety of reasons.
32. The phrase >a variety of reasons? in the passage is closest in meaning to _______.
A. some reasons B. few reasons
C. many reasons D. no reasons
33. The word <goal= in the passage is closest in meaning to _______.
A. score B. desire C. guide D. choice
34. Many students have difficulty in reading comprehension because ______________.
A. they do not have enough time to read
B. they haven?t practiced reading enough
C. they do not have the appropriate approaches to reading
D. they do not have the right attitudes
35. In helping students better comprehend the reading, visual reading _________________.
A. combines the oral, visual and written information
B. associates the meanings with the descriptions
C. processes the information for students
D. provides a description of the learning process
36. One possible reason why students do not read well is that ___________________.
A. they cannot visualize the information
B. they listen and read passively
C. they cannot demonstrate what they have read
D. they do not receive praise and feedback from teachers
37. Thinking aloud is a ________________.
A. quick and noisy process of comprehension
B. step by step process towards reading
C. conversational method of reading comprehension
D. process in which students express their thoughts in spoken words
38. One advantage of thinking aloud is ______________.
A. students do not have to write down their thoughts

73
Downloaded by Nhi An (annhinguyendang@gmail.com)
lOMoARcPSD|15007663

TRAN VAN HAU (ERIC TRAN) VSTEP – READING B1 UPDATED 2019


B. students? ability to think and process information silently is developed
C. students can whisper softly in this technique
D. students can talk while studying in class
39. In order to demonstrate students? comprehension, ________________.
A. homework and in-class assignments should be given
B. students must hear and read the information well beforehand
C. the tasks? level of difficulty must be maintained consistently
D. all the related information must be provided
40. It can be inferred that in order to improve students? ability in reading _____________.
A. only one technique is enough
B. the teachers have to work very hard
C. the students have to read a lot
D. one technique is not enough, a combination is recommended

SESSION 11
PRACTICE TEST 9
PASSAGE 1 – QUESTIONS 1 – 10
While the bald eagle is one national symbol of the United States, it is not the only one. Uncle
Sam, the bearded gentleman costumed in the red, white, and blue stars and stripes of the nation's
flag, is another well-known national symbol. According to legend, this character is based on
Samuel Wilson, the owner of a meat packing business in Troy, New York. During the War of
1812, Sam Wilson's company was granted a government contract to supply meat to the nation's
soldiers; this meat was supplied to the army in barrels stamped with the initials U.S., which
stood for United States. However, the country was at that time relatively young, and the initials
U.S. were not commonly used. Many people questioned what the initials represented, and the
standard reply became "Uncle Sam", for the owner of the barrels. It is now generally accepted
that the figure of Uncle Sam is based on Samuel Wilson, and the U.S. Congress has made it
official by adopting a resolution naming Samuel Wilson as the inspiration for Uncle Sam.
1. The paragraph preceding this passage most probably discusses...
a. the war of 1812.
b. the bald eagle, which symbolizes in the United States.
c. Sam Wilson's meat-packing company.
d. the costume worn by Uncle Sam.
2. Which of the following is the most appropriate title for this passage?
a. The Bald Eagle.
b. The symbols of the United States.

74
Downloaded by Nhi An (annhinguyendang@gmail.com)
lOMoARcPSD|15007663

TRAN VAN HAU (ERIC TRAN) VSTEP – READING B1 UPDATED 2019


c. Samuel Wilson
d. Uncle Sam-Symbols of the Nation.
3. Which of the following is NOT mentioned about Uncle Sam's appearance?
a. He wears facial hair
b. The is some blue in his clothing
c. He is bald.
d. His clothes have stripes in them.
4. The word "costumed" in line 2 could most easily be replaced by
a. dressed b. nationalized
c. hidden d. seen
5. Sam Wilson was the proprietor of what type of business?
a. A costume company b. A meat-packing company.
c. A military clothier. d. A barrel-making company.
6. The word <granted', in line 5 means...
a. Refused. b. Underbid for.
c. Told about d. Given
7. According to the passage, what was in the barrels stamped U.S.?
a. Sam Wilson. b. Food for the army.
c. Weapons to be used in the war. d. Company contracts
8. The word "initials" in line 7 means...
a. Nicknames b. Family names.
c. First letter of the words. d. Company names.
9. The word "official" in line 11 is closest in meaning to...
a. Authorized. b. Professional.
C. Dutiful. d. Accidental.
10. In 1812, people most probably answered that the letters <U.S." " written on the barrels stood
for "Uncle Sam" because...
a. congress required it.
b. Samuel Wilson was their favorite uncle.
c. Sam Wilson preferred it.
d. they were not exactly sure what the letters meant.

PASSAGE 2 – Questions 11 – 20
When Christopher Columbus landed on America's shores, he encountered copper-shinned people
whom he promptly called "Indians". Current estimates indicate that there were over a million
Indians inhabiting North America then. There are approximately 800,000 Indians today, of
whom about 250,000 live on reservations.
75
Downloaded by Nhi An (annhinguyendang@gmail.com)
lOMoARcPSD|15007663

TRAN VAN HAU (ERIC TRAN) VSTEP – READING B1 UPDATED 2019

The early settlers had an amicable relationship with Indians, who shared their knowledge of
hunting, fishing, and farming with their uninvited guests. The stereotyped stealthy, wicked
Indian of modern Western movies was created by indifferent, faithless white men; the Indian
was born friendly.

Disgust developed between the Indians and the settlers, whose encroachment on Indian lands
provoked an era of turbulence. As early as 1745, Indian tribes joined together to drive the French
off their land. The French and Indian War did not end until 1763. The Indians had succeeded in
destroying many of the Western settlements. The British, superficially submissive to the Indians,
promised that further migrations west would not extend beyond a specified boundary.

Evicted from their lands or, worse still, frankly ceding their property to the whites for a few
baubles, Indians were ruthlessly pushed west. The battle in 1876 at Little Big Horn River in
Montana, in which Sitting Bull and the Sioux tribes massacred General Custer's cavalry, caused
the whites to intensify their campaign against the redmen. The battle at Wounded Knee, South
Dakota, in 1890 put an end to the last vestige of hope for amity between Indians and whites.

Although the Bureau of Indian Affairs has operated since 1842, presumably for the purpose of
guarding Indians' interests, Indians on reservations lead notoriously deprived lives. In recent
times Indians have taken a militant stand and have appealed to the courts and the American
people to improve their substandard living conditions.

11. You can infer the author of this passage_____


A. works for the Bureau of Indian Affairs
B. thinks the Indians were ferocious savages
C. admired the settlers for their endurance
D. sympathizes with the Indian
12. According to the passage, the early settlers in America______.
A.had to fight Indians
B.went hunting and fishing
C.found the Indians very helpful
D.thought the Indians were mean
13. The passage suggests that the French and Indian War______.
A.lasted eighteen years B.was quickly terminated
C.led to westward migration D.made the Indians leave their land
14. According to the passage, which of the following statements is TRUE?
A.There are approximately 250,000 Indians today.
76
Downloaded by Nhi An (annhinguyendang@gmail.com)
lOMoARcPSD|15007663

TRAN VAN HAU (ERIC TRAN) VSTEP – READING B1 UPDATED 2019


B.The French and Indian War did not end until 1763.
C.In 1842, Indian tribes tried to drive the French off their land.
D.In 1890, Indians succeeded in destroying many of the Western settlements.
15. The Britain made an agreement with the Indian to
A. food the Indians B. get the Indians? land
C. stop westward migration D. send Daniel Boone across the continent
16. The Indians sold their land _____________-
A. for huge profits B. for a few trinkets
C. because they didn?t understand the language D. thinking it was infertile
17. At Little Big Horn River, the Indians were _______________
A. defeated B. the victors
C. obliged to retreat D. massacred
18. The battle at Wounded Knee _________________
A. marks the end of the Indian wars
B. occurred on the Indian reservation
C. caused great hope among the Indians
D. was won by the Indians
19. Apparently the author feels that the Bureau of Indian Affairs______________
A. has been of great help to the Indians
B. was established in the 19th century
C. deprived the Indians
D. has never done much for the Indians
20. The Indians have recently begun to ______________
A. rebel against the Bureau of Indian Affairs
B. Stand up for their rights as a people
C. suffer from extreme deprivation
D. live on reservations

PASSAGE 3 – Questions 21 – 30
Certainly no creature in the sea is stranger than the common sea cucumber. All living creatures,
especially human beings, have their features, but everything about the little sea cucumber seems
unusual. What else can be said about a bizarre animal that eats mud, feeds almost continuously
day and night but can live without eating for long periods, and can be poisonous but is
considered eatable?
For some fifty million years, in spite of all its strange features, the sea cucumber has lived on its
diet of mud. It is adaptable enough to live attached to rocks by its tube feet, under rocks in
shallow water, or on the surface of mud flats. Common in cool water on both Atlantic and
77
Downloaded by Nhi An (annhinguyendang@gmail.com)
lOMoARcPSD|15007663

TRAN VAN HAU (ERIC TRAN) VSTEP – READING B1 UPDATED 2019


Pacific shores, it has the ability to suck up mud or sand and digest whatever nutrients are
present.
Sea cucumbers come in a variety of colors, ranging from black to reddish-brown to sand-color
and nearly white. One form even has vivid purple tentacles. Usually the creature are cucumber-
shaped----where their name comes from----and because they live in rocks, this shape, combined
with flexibility, enables them to press into crevices where they are safe from animal eater and
ocean currents.
Although they have greedy desire for food, eating day and night, sea cucumbers have the ability
to become still and live at a low metabolic rate----feeding a little or not at all for long periods, so
that the living things in the ocean that provide their food have a chance to increase by a large
amount. If it were not for this faculty, they would swallow all the food available in a short time
and would probably starve themselves out of existence.
But the most impressive thing about the sea cucumber is the way it defends itself. Its major
enemies are fish and crabs. When attacked, it forces all its internal organs in its body out into
water. It also casts off attached structures such as tentacles. The sea cucumber will get rid of its
organs and grow itself again if it is attacked or even touched; it will do the same if the
surrounding water temperature is too high or if the water becomes too polluted.

21. What does the passage mainly discuss?


a.The reason for the sea cucumber's name
b.What makes the sea cucumber unusual
c.How to identify the sea cucumber
d.Places where the sea cucumber can be found
22.In line 3,the word "bizarre"is closest meaning to
a.odd b.marine c.simple d.rate
23. According to the passage, why is the shape of sea cucumbers important?
A. It makes them attractive to fish.
B. It helps them to digest their food.
C. It helps them to protect themselves from danger.
D It makes it easier for them to move through the mud.
24. The words "this faculty" in paragraph 3 refer to the sea cucumber's ability to
a. aqueeze into crevices b. devour all available food in a short time
c. suck up mud or sand d. live at a low metabolic rate
25. The fourth paragraph of the passage mainly discusses _____.
A. the reproduction of sea cucumbers B. the food sources of sea cucumbers
C. the eating habits of sea cucumbers D. the danger to sea cucumbers? existence
26. The phrase "casts off" in paragraph 4 is closest in meaning to
a. grows again

78
Downloaded by Nhi An (annhinguyendang@gmail.com)
lOMoARcPSD|15007663

TRAN VAN HAU (ERIC TRAN) VSTEP – READING B1 UPDATED 2019


b. grabs
c. gets rid of
d. uses as a weapon
27. Of all the characteristics of the sea cucumber,which of the following seems to fascinate the
author most?
a.What it does when threatened
b.where it lives
c.How it hides from predators
d.What it eats
28. Compared with other sea creatures the sea cucumber is very__________________
a. dangerous b. intelligent
c. strange d. fat
29. What can be inferred about the defense system of the sea cucumber?
A. They are similar to those of most sea creatures.
B. They are very sensitive to the surroundings.
C. They require group cooperation.
D. They are almost useless.
30. Which of the following would NOT cause a sea cucumber to give away its internal organs
into the water?
A. Touch B. Food C. Temperature D. Pollution
PASSAGE 4 – Questions 31 – 40
One of the most important social developments that helped to make possible a shift in thinking
about the role of public education was the effect of the baby boom of the 1950's and 1960's on
the schools. In the 1920's, but especially in the Depression conditions of the 1930's, the United
States experienced a declining birth rate-- every thousand women aged fifteen to forty-four gave
birth to about 118 live children in 1920, 89. 2 in 1930, 75. 8 in 1936, and 80 in 1940. With the
growing prosperity brought on by the Second World War and the economic boom that followed
it, young people married and established households earlier and began to raise large families
than had their predecessors during the Depression. Birth rate rose to 102 per thousand in 1946,
106.2 in 1950, and 118 in 1955. Although economics was probably the most important factor, it
is not the only explanation for the baby boom. The increased value placed on the idea of the
family also helps to explain this rise in birth rates. The baby boomers began streaming into the
first grade by the mid-1940's and became a flood by 1950. The public school system suddenly
found itself overtaxed. While the number of school children rose because of wartime and
postwar conditions, these same conditions made the schools even less prepared to cope with the
flood. The wartime economy meant that few new schools were built between 1940 and 1945.
Moreover, during the war and in the boom times that followed, large numbers of teachers left
their profession for better- paying jobs elsewhere in the economy.

79
Downloaded by Nhi An (annhinguyendang@gmail.com)
lOMoARcPSD|15007663

TRAN VAN HAU (ERIC TRAN) VSTEP – READING B1 UPDATED 2019


Therefore, in the 1950'S and 1960's, the baby boom hit an old- fashioned and inadequate school
system. Consequently, the "custodial rhetoric" of the 1930's and early 1940's no longer made
sense; that is, keeping youths aged sixteen and older out of the labor market by keeping them in
school could no longer be a high priority for an institution unable to find space and staff to teach
younger children aged five to sixteen. With the baby boom, the focus of educators and of laymen
interested in education inevitably turned toward the lower grade and back to basic academic
skills and discipline. The system no longer had much interest in offering nontraditional, new,
and extra services to older youths.
31. What is the passage mainly concerned with?
A.The impact of the baby boom on public education.
B.Birth rates in the United States in the 1930's and 1940's.
C.The teaching profession during the baby boom.
D.The role of the family in the 1950's and 1960's.
32. The word "it" in paragraph 1 refers to____
A. the economic boom B. the Second World War
C. the 1930s D. the United States
33: The word "overtaxed" in paragraph 1 is closest in meaning to____.
A. charged too much B. plentifully supplied
C. heavily burdened D. well prepared
34. The public school of the 1950s and 1960s faced all of the following problems EXCEPT
A. an inadequate number of school buildings
B. old-fashioned facilities
C. a shortage of teachers
D. a declining number of students
35: According to the passage, why did teachers leave the teaching profession after the outbreak
of the war?
A. Teaching positions were scarce.
B. They were dissatisfied with the curriculum.
C. Other jobs provided higher salaries.
D. They needed to be retrained.
36. The word <inadequate= in paragraph 2 is closet in meaning to
A. deficient B. expanded
C. innovative D. specialized
37. The <custodial rhetoric= mentioned in the paragraph 2 refers to
A. raising a family
B. keeping older individuals in school
C. running an orderly household
80
Downloaded by Nhi An (annhinguyendang@gmail.com)
lOMoARcPSD|15007663

TRAN VAN HAU (ERIC TRAN) VSTEP – READING B1 UPDATED 2019


D. maintaining discipline in the classroom
38: The word "inevitably" in paragraph 2 is closest in meaning to____
A. unavoidably B. impartially C. irrationally D. unwillingly
39. Where in the passage does the author refer to the attitude of Americas toward to raising a
family in the 1950?s and 1960?s?
A. Lines 1 – 3
B. Lines 11 – 14
C. Lines 22 – 24
D. Lines 26 - 27
40: Which of the following best characterizes the organization of the passage?
A. The second paragraph provides a fictional account to illustrate a problem presented in the first
paragraph.
B. The second paragraph argues against a point made in the first paragraph.
C. The second paragraph introduces a problem not mentioned in the first paragraph
D. The second paragraph presents the effect of circumstances described in the first paragraph.

SESSION 12
PRACTICE TEST 10
PASSAGE 1 – Questions 1 – 10
Large animals that inhabit the desert have evolved a number of adaptations for reducing the
effects of extreme heat. One adaptation is to be light in color, and to reflect rather than absorb
the Sun?s rays. Desert mammals also depart from the normal mammalian practice of
maintaining a constant body temperature. Instead of trying to keep down the body
temperature deep inside the body, which would involve the expenditure of water and energy,
desert mammals allow their temperatures to rise to what would normally be fever height, and
temperatures as high as 46 degrees Celsius have been measured in Grant?s gazelles. The
overheated body then cools down during the cold desert night, and indeed the temperature may
fall unusually low by dawn, as low as 34 degrees Celsius in the camel. This is an advantage
since the heat of the first few hours of daylight is absorbed in warming up the body, and an
excessive buildup of heat does not begin until well into the day.
Another strategy of large desert animals is to tolerate the loss of body water to a point that
would be fatal for non-adapted animals. The camel can lose up to 30 percent of its body weight
as water without harm to itself, whereas human beings die after losing only 12 to 13 percent of
their body weight. An equally important adaptation is the ability to replenish this water loss at
one drink. Desert animals can drink prodigious volumes in a short time, and camels have been
known to imbibe over 100 liters in a few minutes. A very dehydrated person, on the other
hand, cannot drink enough water to rehydrate at one session, because the human stomach is not
sufficiently big and because a too rapid dilution of the body fluids causes death from water
81
Downloaded by Nhi An (annhinguyendang@gmail.com)
lOMoARcPSD|15007663

TRAN VAN HAU (ERIC TRAN) VSTEP – READING B1 UPDATED 2019


intoxication. The tolerance of water loss is of obvious advantage in the desert, as animals do
not have to remain near a water hole but can obtain food from grazing sparse and far-flung
pastures. Desert-adapted mammals have the further ability to feed normally when extremely
dehydrated, it is a common experience in people that appetite is lost even under conditions of
moderate thirst.
1. What is the main topic of the passage?
A. Weather variations in the desert. B. Adaptations of desert animals.
C. Diseased of desert animals. D. Human use of desert animals.
2. According to the passage, why is light coloring an advantage to large desert animals?
A. It helps them hide from predators.
B. It does not absorb sunlight as much as dark colors.
C. It helps them see their young at night.
D. It keeps them cool at night.
3. The word <maintaining= is closest in meaning to _________.
A. measuring B. inheriting C. preserving D. delaying
4. The author uses of Grant?s gazelle as an example of
A. an animal with a low average temperature
B. an animal that is not as well adapted as the camel
C. a desert animal that can withstand high body temperatures
D. a desert animal with a constant body temperature
5. When is the internal temperature of a large desert mammal lower?
A. Just before sunrise B. In the middle of the day
C. Just after sunset D. Just after drinking
6. The word <tolerate= is closest in meaning to _________.
A. endure B. replace C. compensate D. reduce
7. What causes water intoxication?
A. Drinking too much water very quickly. B. Drinking polluted water.
C. Bacteria in water. D. Lack of water.
8. Why does the author mention humans in the second paragraph?
A. To show how they use camels.
B. To contrast them to desert mammals.
C. To give instructions about desert survival.
D. To show how they have adapted to desert life.
9. The word <obtain= is closest in meaning to _________.
A. digest B. carry C. save D. get
10. Which of the following is NOT mentioned as an adaptation of large desert animals?
A. Variation in body temperatures. B. Eating while dehydrated.
82
Downloaded by Nhi An (annhinguyendang@gmail.com)
lOMoARcPSD|15007663

TRAN VAN HAU (ERIC TRAN) VSTEP – READING B1 UPDATED 2019


C. Drinking water quickly. D. Being active at night.
PASSAGE 2 – Questions 11 – 20
Rent control is the system whereby the local government tells building owners how much they
can charge their tenants in rent. In the United States, rent controls date back to at least World
WarII.
In 1943 the federal government imposed rent controls to help solve the problem of housing
shortages during wartime. The federal program ended after the war, but in some locations,
including New York City, controls continued. Under New York's controls, a landlord generally
cannot raise rents on apartments as long as the tenants continue to renew their leases. In places
such as Santa Monica, California, rent controls are more recent. They were spurred by the
inflation of the 1970's, which, combined with California's rapid population growth, pushed
housing prices, as well as rents, to record levels. In 1979 Santa Monica's municipal
government ordered landlords to roll back their rents to the levels charged in 1978. Future
rents could only go up by two-thirds as much as any increase in the overall price level.

In any housing market, rental prices perform three functions: (1) promoting the efficient
maintenance of existing housing and stimulating the construction of new housing, (2)
allocating existing scarce housing among competing claimants, and (3) rationing use of
existing housing by potential renters.
One result of rent control is a decrease in the construction of new rental units. Rent controls
have artificially depressed the most important long-term determinant of profitability — rents.
Consider some examples. In a recent year in Dallas, Texas, with a 16 percent rental vacancy
rate but no rent control laws, 11,000 new housing units were built. In the same year, in San
Francisco, California, only 2,000 units were built. The major difference? San Francisco has
only a 1.6 percent vacancy rate but stringent rent control laws. In New York City, except for
government-subsidized construction, the only rental units being built are luxury units, which
are exempt from controls. In Santa Monica, California, new apartments are not being
constructed. New office rental space and commercial developments are, however. They are
exempt from rent controls.
11. What does the passage mainly discuss?
(A) The construction of apartments in the United States.
(B) Causes and effects of rent control
(C) The fluctuations of rental prices
(D) The shortage of affordable housing in the United States.
12. The word "They" in line 9 refers to
(A) the tenants (B) their leases
(C) places (D) rent controls.
13. Which of the following was NOT a reason for the introduction of rent controls in Santa
Monica, California?
(A) rapid population growth
(B) inflation
(C) economic conditions during wartime
(D) record-high housing prices
83
Downloaded by Nhi An (annhinguyendang@gmail.com)
lOMoARcPSD|15007663

TRAN VAN HAU (ERIC TRAN) VSTEP – READING B1 UPDATED 2019


14. The phrase "roll back" in line 11 is closest in meaning to
(A) credit (B) measure
(C) vary (D) reduce
15. The word "stimulating" in line 15 is closest in meaning to
(A) experimenting with (B) identifying
(C) estimating (D) encouraging
16. It can be inferred that the purpose of rent control is to
(A) protect tenants (B) promote construction
(C) increase vacancy rates (D) decrease sales of rental units
17. The word "depressed" in line 19 is closest in meaning to
(A) saddened (B) created
(C) lowered (D) defeated
18. The information in the last paragraph supports which of the following statements?
(A) San Francisco has eliminated its rent control laws.
(B) Rent control leads to a reduction in the construction of housing units
(C) Luxury apartments are rarely built when there is rent control
(D) There is a growing need for government-subsidized housing.
19. According to the passage,which of the following cities does NOT currently have rent
controls?
(A) Santa Monica (B) Dallas
(C) San Francisco (D) New York City
20. According to the passage,which of the following is NOT exempt from rent control?
(A) Luxury apartments
(B) Commercial development
(C) Moderately priced apartments
(D) Office space.
PASSAGE 3 – Questions 21 – 30
In the United States in the early 1800's, individual state governments had more effect on the
economy than did the federal government. States chartered manufacturing, banking, mining,
and transportation firms and participated in the construction of various internal improvements
such as canals, turnpikes, and railroads. The states encouraged internal improvements in two
distinct ways ; first, by actually establishing state companies to build such improvement ;
second, by providing part of the capital for mixed public-private companies setting out to make
a profit.
In the early nineteenth century, state governments also engaged in a surprisingly large amount
of direct regulatory activity, including extensive licensing and inspection programs. Licensing
targets reflected both similarities in and differences between the economy of the nineteenth
century and that of today: in the nineteenth century, state regulation through licensing fell
especially on peddlers, innkeepers, and retail merchants of various kinds. The perishable
commodities of trade generally came under state inspection, and such important frontier
staples as lumber and gunpowder were also subject to state control. Finally, state governments
experimented with direct labor and business regulation designed to help the individual laborer

84
Downloaded by Nhi An (annhinguyendang@gmail.com)
lOMoARcPSD|15007663

TRAN VAN HAU (ERIC TRAN) VSTEP – READING B1 UPDATED 2019


or consumer, including setting maximum limits on hours of work and restrictions on price-
fixing by businesses.
Although the states dominated economic activity during this period, the federal government
was not inactive. Its goals were the facilitation of western settlement and the development of
native industries. Toward these ends the federal government pursued several courses of action.
It established a national bank to stabilize banking activities in the country and, in part, to
provide a supply of relatively easy money to the frontier, where it was greatly needed for
settlement. It permitted access to public western lands on increasingly easy terms, culminating
in the Homestead Act of 1862, by which title to land could be claimed on the basis of
residence alone. Finally, it set up a system of tariffs that was basically protectionist in effect,
although maneuvering for position by various regional interests produced frequent changes in
tariff rates throughout the nineteenth century.
21. What does the passage mainly discuss?
(A) States's rights versus federal rights
(B) The participation of state governments in railroad, canal, and turnpike construction
(C) The roles of state and federal governments in the economy of the nineteenth century
(D) Regulatory activity by state governments
22. The word <effect= in line 1 is closest in meaning to
(A) value (B) argument
(C) influence (D) restraint
23. All of the following are mentioned in the passage as areas that involved state governments
in the nineteenth century EXCEPT
(A) mining (B) banking
(C) manufacturing (D) higher education
24. The word <distinct= in line 5 is closest in meaning to
(A) separate (B) innovative
(C) alarming (D) provocative
25. It can be inferred from the first paragraph that in the nineteenth century canals and
railroads were
(A) built with money that came from the federal government
(B) much more expensive to build than they had been previously
(C) built predominantly in the western part of the country
(D) sometimes built in part by state companies
26. The regulatory activities of state governments included all of the following EXCEPT
(A) licensing of retail merchants
(B) inspecting materials used in turnpike maintenance
(C) imposing limits on price-fixing
(D) control of lumber

85
Downloaded by Nhi An (annhinguyendang@gmail.com)
lOMoARcPSD|15007663

TRAN VAN HAU (ERIC TRAN) VSTEP – READING B1 UPDATED 2019


27. The word <setting= in line 17 is closest in meaning to
(A) discussing (B) analyzing
(C) establishing (D) avoiding
28. The word <ends= in line 20 is closest in meaning to
(A) benefits (B) decisions
(C) services (D) goals
29. According to the passage, which of the following is true of the Homestead Act of 1862 ?
(A) It made it increasingly possible for settlers to obtain land in the West.
(B) It was a law first passed by state governments in the West.
(C) It increased the money supply in the West.
(D) It established tariffs in a number of regions.
30. Which of the following activities was the responsibility of the federal government in the
nineteenth century?
(A) Control of the manufacture of gunpowder
(B) Determining the conditions under which individuals worked
(C) Regulation of the supply of money
(D) Inspection of new homes built on western lands
PASSAGE 4 – Questions 31 – 40
The history of clinical nutrition, or the study of the relationship between health and how the body
takes in and utilizes food substances, can be divided into four distinct eras: the first began in the
nineteenth century and extended into the early twentieth century when it was recognized for the
first time that food contained constituents that were essential for human function and that different
foods provided different amounts of these essential agents. Near the end of this era, research studies
demonstrated that rapid weight loss was associated with nitrogen imbalance and could only be
rectified by providing adequate dietary protein associated with certain foods.
The second era was initiated in the early decades of the twentieth century and might be called "the
vitamin period. "Vitamins came to be recognized in foods, and deficiency syndromes were
described. As vitamins became recognized as essential food constituents necessary for health, it
became tempting to suggest that every disease and condition for which there had been no previous
effective treatment might be responsive to vitamin therapy. At that point in time, medical schools
started to become more interested in having their curricula integrate nutritional concepts into the
basic sciences. Much of the focus of this education was on the recognition of deficiency symptoms.
Herein lay the beginning of what ultimately turned from ignorance to denial of the value of
nutritional therapies in medicine. Reckless claims were made for effects of vitamins that went far
beyond what could actually be achieved from the use of them.
In the third era of nutritional history in the early 1950's to mid-1960's, vitamin therapy began to
fall into disrepute. Concomitant with this, nutrition education in medical schools also became less
popular. It was just a decade before this that many drug companies had found their vitamin
sales skyrocketing and were quick to supply practicing physicians with generous samples of
vitamins and literature extolling the virtue of supplementation for a variety of health-related
86
Downloaded by Nhi An (annhinguyendang@gmail.com)
lOMoARcPSD|15007663

TRAN VAN HAU (ERIC TRAN) VSTEP – READING B1 UPDATED 2019


conditions. Expectations as to the success of vitamins in disease control were exaggerated. As is
known in retrospect, vitamin and mineral therapies are much less effective when applied to health-
crisis conditions than when applied to long-term problems of under nutrition that lead to chronic
health problems.
31. What does the passage mainly discuss?
a. The effects of vitamins on the human body
b. The history of food preferences from the nineteenth century to the present
c. The stages of development of clinical nutrition as a field of study
d. Nutritional practices of the nineteenth century
32. It can be inferred from the passage that which of the following discoveries was made
during the first era in the history of nutrition.
a. Protein was recognized as an essential component of diet
b. Vitamins were synthesized rom foods.
c. Effective techniques of weight loss were determined.
d. Certain foods were found to be harmful to good health.
33. The word "tempting"in line 11 is closest in meaning to
a.necessary b.attractive c.realistic d.correct
34. The word"Reckless"in line 16 is closest in meaning to
a.recorded b.irresponsible c.informative d.urgent
35. The word"them"in line 17 refers to
a.therapies b.claims c.effects d.vitamins
36. Why did vitamin therapy begin losing favor in the 1950's ?
a.The public lost interest in vitamins.
b.Medical schools stopped teaching nutritional concepts.
c.Nutritional research was of poor quality
d.Claims for effectiveness of vitamin therapy were seen to be exaggerated.
37. The phrase"concomitant with"in line 19 is closest in meaning to
a.in conjunction with b.prior to c.in dispute with d.in regard to
38. The word"skyrocketing"in line 21 is closest in meaning to …………
a.internationally popular b.increasing rapidly c.acceptable d.surprising
39. The word"extolling"in line 22 is closest in meaning to
a.analizing b.questioning c.praising d.promising
40. The paragraph following the passage most probably discusses……………
a.the fourth era of nutrition history
b.problems associated with under nutrition
c.how drug companies became successful
d.why nutrition education lost its appeal

SESSION 13
PRACTICE TEST 11

87
Downloaded by Nhi An (annhinguyendang@gmail.com)
lOMoARcPSD|15007663

TRAN VAN HAU (ERIC TRAN) VSTEP – READING B1 UPDATED 2019


PASSAGE 1 – Questions 1 – 10
Insects' lives are very short and they have many enemies, but they must survive long enough to
breed and perpetuate their kind. The less insect-like they look, the better their chance of
survival. To look "inedible" by imitating plants is a way frequently used by insects to survive.
Mammals rarely imitate plants, but many fish and invertebrates do.

The stick caterpillar is well named. It is hardly distinguishable from a brown or green twig.
This caterpillar is quite common and can be found almost anywhere in North America. It is
also called "measuring worm" or "inchworm." It walks by arching its body, then stretching out
and grasping the branch with its front feet then looping its body again to bring the hind feet
forward. When danger threatens, the stick caterpillar stretches its body away from the branch
at an angle and remains rigid and still, like a twig, until the danger has passed.

Walking sticks, or stick insects, do not have to assume a rigid, twig-like pose to find
protection; they look like inedible twigs in any position. There are many kinds of walking
sticks, ranging in size from the few inches of the North American variety to some tropical
species that may be over a foot long. When at rest their front legs are stretched out,
heightening their camouflage. Some of the tropical species are adorned with spines or ridges,
imitating the thorny bushes or trees in which they live.

Leaves also seem to be a favorite object for insects to imitate. Many butterflies can suddenly
disappear from view by folding their wings and sitting quietly among the plants that they
resemble.
1. What is the main idea of the passage?
A. The feeding habits of insects
B. Insects that are threatened with extinction
C. How some insects imitate plants to survive?
D. Caterpillars that live in trees
2. Which of the following does the word <enemies= in line 1 refer to?
A. extreme weather conditions B. creatures that eat insects
C. plants looking like insects D. insects looking like plants
3. According to the passage, how does the stick caterpillar make itself look like a twig?
A. By laying its body flat against a branch
B. By changing the color of its skin
C. By holding its body stiff and motionless
D. By looping itself around a stick
4. Which of the following is true of stick insects?
A. They make themselves look like other insects.
B. They change color to make themselves invisible.
C. They are camouflaged only when walking.
D. They resemble their surroundings all the time.
5. Which of the following is the antonym of the word <inedible= in paragraph 3?
A. eatable B. colorful C. moving D. beautiful
88
Downloaded by Nhi An (annhinguyendang@gmail.com)
lOMoARcPSD|15007663

TRAN VAN HAU (ERIC TRAN) VSTEP – READING B1 UPDATED 2019


6. Which of the following are NOT mentioned in the passage as objects that are imitated as a
means of protection?
A. leaves B. flowers C. thorns D. sticks
7. In which paragraph does the author describe the way in which stick caterpillars move?
A. Paragraph 1
B. Paragraph 2
C. Paragraph 3
D. Paragraph 4
8. According to the passage, which of the following is NOT TRUE about the stick caterpillar?
A. It is not popular in North America
B. It changes its color to avoid danger
C. It can have different sizes
D. The tropical stick caterpillars can have parts of their body looking like thorns.
9. How can butterflies make themselves invisible?
A. by hiding under the leaves
B. By disappearing from the view
C. By folding wings and sitting still among the leaves with similar colors
D. By flying among colorful flowers
10. Which of the following best paraphrases the sentence in italics? The stick caterpillar is well
named.
A. The caterpillar has a good name.
B. The caterpillar is stuck to a popular name.
C. The caterpillar is named just like the way it looks.
D. The caterpillar is named after a well-known name.
PASSAGE 2 - Questions 11 – 20
Fungi, of which there are over 100,000 species, including yeasts and other single-celled
organisms as well as the common molds and mushrooms, were formerly classified as members
of the plant kingdom. However, in reality they are very different from plants and today they are
placed in a separate group altogether. The principal reason for this is that none of them
possesses chlorophyll, and since they cannot synthesize their own carbohydrates, they obtain
their supplies either from the breakdown of dead organic matter or from other living organisms.

Furthermore the walls of fungal cells are not made of cellulose, as those of plants are, but of
another complex sugar like polymer called chitin, the material from which the hard outer
skeletons of shrimps, spiders, and insects are made. The difference between the chemical
composition of the cell walls of fungi and those of plants is of enormous importance because it
enables the tips of the growing hyphae, the threadlike cells of the fungus, to secrete enzymes
that break down the walls of plant cells without having any effect on those of the fungus itself.
It is these cellulose-destroying enzymes that enable fungi to attack anything made from wood,
wood pulp, cotton, flax, or other plant material.

The destructive power of fungi is impressive. They are a major cause of structural damage to
building timbers, a cause of disease in animals and humans, and one of the greatest causes of
agricultural losses. Entire crops can be wiped out by fungal attacks both before and after

89
Downloaded by Nhi An (annhinguyendang@gmail.com)
lOMoARcPSD|15007663

TRAN VAN HAU (ERIC TRAN) VSTEP – READING B1 UPDATED 2019


harvesting. Some fungi can grow at +50°C, while others can grow at -5°C, so even food in cold
storage may not be completely safe from them. On the other hand, fungi bring about the
decomposition of dead organic matter, thus enriching the soil and returning carbon dioxide to
the atmosphere. They also enter into a number of mutually beneficial relationships with plants
and other organisms. In addition, fungi are the source of many of the most potent antibiotics
used in clinical medicine, including penicillin.

11. What does paragraph 1 mainly discuss?


(A) differences between simple and complex fungi
(B) functions of chlorophyll in plants
(C) functions of sugar in the walls of fungal cells
(D) differences between fungi and plants
12. Which of the following is mentioned as a major change in how scientists approach the study
of fungi?
(A) Fungi are no longer classified as plants
(B) Some single-cell organisms are no longer classified as fungi.
(C) New methods of species identification have been introduced
(D) Theories about the chemical composition of fungi have been revised.
13. The word <principal= in line 4 is closest in meaning to
(A) true
(B) main
(C) logical
(D) obvious
14.According to the passage , how do fungi obtain carbohydrates?
(A) The absorb carbohydrates from their own cell walls.
(B) They synthesize chlorophyll to produce carbohydrates.
(C) They produce carbohydrates by breaking down chitin.
(D) They acquire carbohydrates from other organic matter, both living and dead.
15. The passage mentions shrimps, spiders, and insects in line 9 because their skeletons
(A) can be destroyed by fungi
(B) have unusual chemical compositions
(C) contain a material found in the walls of fungal cells
(D) secrete the same enzymes as the walls of fungal cells do
16. The word <those= in line 13 refers to
(A) tips
(B) hyphae
(C) enzymes
(D) walls
17. Fungi have all of the following characteristics EXCEPT
(A) They grow hyphae. (B) They secrete enzymes.
(C) They synthesize cellulose. (D) They destroy crops.
18. The word <Entire= in paragraph 2 is closest in meaning to
(A) certain (B) whole
(C) mature (D) diseased

90
Downloaded by Nhi An (annhinguyendang@gmail.com)
lOMoARcPSD|15007663

TRAN VAN HAU (ERIC TRAN) VSTEP – READING B1 UPDATED 2019


19. The passage describes the negative effects of fungi on all the following EXCEPT
(A) buildings (B) animals
(C) food (D) soil
20. The phrase <bring about= in paragraph 2 is closest in meaning to
(A) cause (B) join
(C) take (D) include
PASSAGE 3 – Questions 21 – 30
Diseases are a natural part of life on Earth. If there were no diseases, the population would grow
too quickly, and there would not be enough food or other resources. So in a way, diseases are
nature's way of keeping the Earth in balance. But sometimes they spread very quickly and kill
large numbers of people. For example, in 1918, an outbreak of the flu spread across the world,
killing over 25 million people in only six months. Such terrible outbreaks of a disease are called
pandemics.

Pandemics happen when a disease changes in a way that our bodies are not prepared to fight. In
1918, a new type of flu virus appeared. Our bodies had no way to fight this new flu virus, and
so it spread very quickly and killed large numbers of people. While there have been many
different pandemic diseases throughout history, all of them have a few things in common.

First, all pandemic diseases spread from one person to another very easily. Second, while they
may kill many people, they generally do not kill people very quickly. A good example of this
would be the Marburg virus. The Marburg virus is an extremely infectious disease. In addition,
it is deadly. About 70-80% of all the people who get the Marburg virus die from the disease.
However, the Marburg virus has not become a pandemic because most people die within three
days of getting the disease. This means that the virus does not have enough time to spread to a
large number of people. The flu virus of 1918, on the other hand, generally took about a week
to ten days to kill its victims, so it had more time to spread.

While we may never be able to completely stop pandemics, we can make them less common.
Doctors carefully monitor new diseases that they fear could become pandemics. For example,
in 2002 and 2003, doctors carefully watched SARS. Their health warnings may have prevented
SARS from becoming a pandemic.
21. This passage is mainly about ____.
A. how to prevent pandemic diseases.
B. pandemic diseases.
C. pandemic diseases throughout history.
D. why pandemics happen.
22. According to paragraph 1, how are diseases a natural part of life on Earth?
A. They prevent pandemics.
B. They help control the population.
C. They help the world grow quickly.
D. They kill too many people.
23. Based on the information in the passage the term pandemics can best be explained as ___.

91
Downloaded by Nhi An (annhinguyendang@gmail.com)
lOMoARcPSD|15007663

TRAN VAN HAU (ERIC TRAN) VSTEP – READING B1 UPDATED 2019


A. diseases with no cure
B. diseases that spread quickly and kill large numbers of people
C. a deadly kind of flu
D. new diseases like SARS or the Marburg virus
24. According to the passage, what causes pandemics?
A. Changes in a disease that the body cannot fight
B. Careless doctors who do not watch the spread of diseases
C. Population growth that the world cannot support
D. The failure to make new medicines
25. According to the passage, all of the following are true of the 1918 flu pandemic EXCEPT
that ____.
A. it involved a new kind of flu virus
B. it killed over 25 million people
C. it was the last pandemic in history
D. it took a little over a week to kill its victims
26. The word <it= in the passage 2 refers to ____.
A. disease B. flu virus
C. pandemics D. bodies
27. Which of the following is mentioned as a common feature of all pandemic diseases?
A. They spread from people to people very slowly.
B. They may kill many people very quickly.
C. They do not kill people very quickly.
D. They kill all the victims.
28. According to paragraph 3, why hasn't Marburg virus become a pandemic?
A. It is not a deadly disease.
B. It does not spread from person to person easily.
C. Doctors have prevented it from becoming a pandemic.
D. It kills people too quickly.
29. The word <monitor= in the last passage is closest in meaning to ____.
A. fight B. prevent
C. watch D. avoid
30. The author mentions SARS in order to ____.
A. give an example of a highly dangerous disease.
B. suggest that SARS will never become a pandemic.
C. give an example of the successful prevention of a pandemic.
D. suggest that there may be a new pandemic soon.
PASSAGE 4 – Questions 31 – 40
According to sociologists, there are several different ways in which a person may become
recognized as the leader of a social group in the United States. In the family traditional cultural
patterns confer leadership on one or both of the parents. In other cases, such as friendship
groups, one or more persons may gradually emerge as leaders, although there is no formal
process of selection. In larger groups, leaders are usually chosen formally through election or
recruitment .

92
Downloaded by Nhi An (annhinguyendang@gmail.com)
lOMoARcPSD|15007663

TRAN VAN HAU (ERIC TRAN) VSTEP – READING B1 UPDATED 2019


Although leaders are often thought to be people with unusual personal ability, decades of
research have failed to produce consistent evidence that there is any category of <natural
leaders.= It seems that there is no set of personal qualities that all leaders have in common;
rather, virtually any person may be recognized as a leader if the person has qualities that meet
the needs of that particular group.

Furthermore, although it is commonly supposed that social groups have a single leader, research
suggests that there are typically two different leadership roles that are held by different
individuals. Instrumental leadership is leadership that emphasizes the completion of tasks by a
social group. Group members look to instrumental leaders to <get things done.= Expressive
leadership, on the other hand, is leadership that emphasizes the collective well-beings of a
social group?s members. Expressive leaders are less concerned with the overall goals of the
group than with providing emotional support to group members and attempting to minimize
tension and conflict among them. Group members expect expressive leaders to maintain stable
relationships within the group and provide support to individual members.

Instrumental leaders are likely to have a rather secondary relationship to other group
members. They give others and may discipline group members who inhibit attainment of the
group?s goals. Expressive leaders cultivate a more personal or primary relationship to others in
the group. They offer sympathy when someone experiences difficulties or is 21 30 subjected to
discipline, are quick to lighten a serious moment with humor, and try to resolve issues that
threaten to divide the group. As the difference in these two roles suggest, expressive leaders
generally receive more personal affection from group members; instrumental leaders, if they are
successful in promoting group goals, may enjoy a more distant respect.
31.What does the passage mainly discuss?
(A)The problems faced by leaders.
(B)How leadership differs in Small and large groups.
(C)How social groups determine who will lead them.
(D)The role of leaders in social groups.
32.The passage mentions all of the following ways by which people can become leaders
EXCEPT
(A) recruitment (B) formal election process
(C) specific leadership training (D) traditional cultural patterns
33.In mentioning <natural leaders= in line 9, the author is making the point that
(A) few people qualify as <natural leaders=.
(B) there is no proof that <natural leaders= exist.
(C) <natural leaders= are easily accepted by the members of a group.
(D) <natural leaders= share a similar set of characteristics
34.Which of the following statements about leadership can be inferred from paragraph 2?
(A) A person who is an effective leader of a particular group may not be an effective leader in
another group
(B) Few people succeed in sharing a leadership role with another person

93
Downloaded by Nhi An (annhinguyendang@gmail.com)
lOMoARcPSD|15007663

TRAN VAN HAU (ERIC TRAN) VSTEP – READING B1 UPDATED 2019


(C)A person can best learn how to be an effective leader by studying research on leadership.
(D)Most people desire to be leaders but can produce little evidence of their qualifications.
35.The passage indicates that instrumental leaders generally focus on
(A)ensuring harmonious relationships.
(B)sharing responsibility with group members.
(C)identifying new leaders.
(D)achieving a goal.
36.The word <collective= in line17 is closest in meaning to
(A)necessary (B)typical
(C)group (D)particular
37.The word <them= in line 19 refers to
(A) expressive leaders (B) goals of the group
(C) group members (D) tension and conflict
38.A <secondary relationship= mentioned in line 27 between a leader and the members of a
group could best be characterized as
(A)distant (B)enthusiastic
(C)unreliable (D) personal
39.The word <resolve= in line 27 is closest in meaning to
(A) avoid repeating (B) talk about
(C) avoid thinking about (D)find a solution for
40.Paragraphs 3 and 4 organize the discussion of leadership primarily in terms of
(A) examples that illustrate a problem
(B) cause and effect analysis
(C) narration of events
(D) comparison and contrast

ANSWER KEY – READING THEORY


SESSION 1
I.Reading for topics, Example: Practice 1:A
main ideas 1.C 2. B Practice 3: B
Practice 4: C
II.Reading for A. Example: B Practice 1: 1. C 2. D 3. B 4. A 5.A
Information Practice 2: 1. C 2. B 3. D 4. A 5. A
Practice 3: 1. B 2. D
Practice 4: 1. D 2. C 3. D
SESSION 2
III.Reading – Recognize Example: B Practice 1: B
referents Practice 2: A
Practice 3: C
Practice 4: 1. B 2. D
IV. Reading – Getting Example: D Practice 1: 1. C 2. A
meaning from context Practice 2: 1. C 2. C 3. D 4. B
V. Make Inference Example: 1. D 2. A Practice 1: 1. A 2. A
VI. Infer Rherical purpose Practice 1: 1. D 2. C
Practice 2: 1. D
94
Downloaded by Nhi An (annhinguyendang@gmail.com)
lOMoARcPSD|15007663

TRAN VAN HAU (ERIC TRAN) VSTEP – READING B1 UPDATED 2019


Practice 3: 1. B 2. A
VII. Determine the tone, Example: Practice 1: 1. C 2. A 3. B
purpose, course 1.D 2. D 3. C Practice 2: D
VIII. Recognize organization Dạng 1: D Practice 1: B Practice 2: B
Dạng 2: C Practice 3: A
Practice 4: 1. C 2. B
Practice 5: 1. C 2. A
Practice 6: A
Practice 7: D
Practice 8: 1. C 2. D
XI. Essential Example: A Practice 1: C
Information
questions
XII. Insert sentence Example: C Practice 1: C
into the passage Practice 2: D

ANSWER KEY – PRACTICE TESTS


SESSION 3 - Practice test 1
1D 2B 3C 4B 5D 6A 7C 8D 9A 10B
11A 12C 13B 14B 15A 16C 17.B 18.A 19.D 20.C
21.B 22.A 23. B 24.B 25.D 27.A 28. C 29. C 30. B
31A 32 D 33C 34D 35B 36B 37D 38D 39A 40.B
SESSION 4 - Practice test 2
1A 2A 3B 4B 5D 6D 7D 8B 9D 10A
11C 12B 13A 14C 15D 16A 17D 18A 19B 20A
21B 22B 23C 24B 25C 26D 27B 28B 29D 30C
31B 32A 33C 34D 35C 36C 37A 38C 39B 40D
SESSION 5 – Practice test 3
1B 2A 3D 4D 5A 6D 7D 8D 9D 10A
11C 12B 13D 14D 15C 16A 17B 18C 19A 20D
21 22B 23C 24B 25A 26B 27A 28A 29B 30D
31C 32A 33B 34D 35C 36C 37D 38A 39B 40B
SESSION 6 – Practice test 4
1A 2C 3A 4B 5C 6B 7D 8B 9D 10A
11D 12C 13C 14D 15A 16A 17B 18B 19D 20B
21D 22C 23D 24D 25B 26C 27B 28B 29B 30C
31B 32A 33C 34D 35A 36B 37A 38C 39C 40D
SESSION 7 – Practice test 5
1A 2A 3D 4B 5D 6C 7B 8D 9C 10B
11B 12B 13C 14D 15A 16C 17A 18D 19B 20A
21A 22C 23A 24B 25C 26D 27C 28A 29D 30C
95
Downloaded by Nhi An (annhinguyendang@gmail.com)
lOMoARcPSD|15007663

TRAN VAN HAU (ERIC TRAN) VSTEP – READING B1 UPDATED 2019


31B 32D 33B 34B 35A 36C 37D 38A 39A 40D
SESSION 8 – Practice 6
1D 2A 3A 4A 5C 6B 7B 8D 9B 10D
11B 12D 13A 14A 15C 16A 17C 18D 19D 20A
21A 22B 23D 24C 25A 26B 27D 28B 29B 30C
31D 32D 33C 34C 35D 36A 37B 38B 39A 40A
SESSION 9 – Practice test 7
1B 2D 3B 4A 5D 6D 7A 8B 9D
11A 12D 13A 14A 15B 16D 17A 18B 19C 20B
21B 22A 23B 24C 25D 26B 27C 28C 29C 30B
31A 32C 33A 34B 35C 36D 37C 38A 39D 40C
SESSION 10 – Practice test 8
1A 2A 3C 4B 5D 6D 7B 8B 9D 10C
11A 12C 13A 14D 15B 16A 17C 18D 19D 20A
21C 22B 23D 24A 25D 26B 27D 28C 29C 30A
31D 32C 33B 34C 35B 36B 37D 38B 39A 40D
SESSION 11 – Practice test 9
1B 2D 3C 4A 5B 6D 7B 8C 9A 10D
11D 12C 13A 14B 15C 16B 17A 18A 19D 20B
21B 22A 23C 24D 25C 26C 27A 28C 29B 30B
31A 32A 33C 34D 35C 36A 37B 38A 39B 40D
SESSION 12 – Practice test 10
1B 2B 3C 4C 5A 6A 7A 8B 9D 10D
11B 12D 13C 14D 15D 16B 17C 18B 19B 20C
21C 22C 23D 24A 25D 26B 27C 28D 29A 30C

31C 32A 33C 34B 35D 36D 37A 38B 39C 40A
SESSION 13 – Practice Test 11
1C 2B 3C 4D 5A 6B 7B 8A 9C 10C
11D 12A 13B 14D 15C 16D 17D 18B 19A 20A
21B 22B 23B 24A 25C 26C 27C 28D 29C 30C
31C 32C 33B 34A 35D 36C 37C 38A 39D 40D

REFERENCES
1 – Reading, VSTEP preparation kit, B1, complied and updated by Phan Thi My Khanh and Vo
Phuong Quyen, January 2017.
2 – Reading, VSTEP preparation kit B2 – C1, complied and updated by Phan Thị Mỹ Khanh
and Vo Phuong Quyen, January 2017.
3 – 20 bộ đề đọc hiểu, thi năng lực ngoại ngữ 6 bậc VIỆT NAM (vstep), trình độ B1 – C1 ( NXB
ĐHSPTPHCM

96
Downloaded by Nhi An (annhinguyendang@gmail.com)

You might also like